Vnmath Ky Yeu TraiHeHungVuong Toan 2009

You might also like

Download as pdf or txt
Download as pdf or txt
You are on page 1of 223

TRI H HNG VNG LN TH 6

=============================

Nguyn Vn Mu (Ch bin)

K YU TRI H HNG VNG


MN TON HC

VIT TR, 02-04/08/ 2009

Mc lc
Li ni u . . . . . . . . . . . . . . . . . . . . . . . . . . . . . . . 1 thi Olympic Ton Hng vng 6 10

1.1 Olympic Ton Hng vng ln th 1, nm 2005 . . . . . . . . . 10 1.2 Olympic Ton Hng vng ln th 2, nm 2006 . . . . . . . . . 11 1.3 Olympic Ton Hng vng ln th 3, nm 2007 . . . . . . . . . 12 1.4 Olympic Ton Hng vng ln th 4, nm 2008 . . . . . . . . . 13 1.5 Olympic Ton Hng vng ln th 5, nm 2009 . . . . . . . . . 15 1.6 p n Olympic Ton Hng vng ln th 5-2009 . . . . . . . . 16 2 i cng v lch s mn gii tch ton hc 2.1 Tm lc lch s mn gii tch 2.1.1 2.1.2 2.1.3 2.1.4 21

. . . . . . . . . . . . . . . . . . 21

Hy Lp v Ma m c i . . . . . . . . . . . . . . . . . . 21 Trung c . . . . . . . . . . . . . . . . . . . . . . . . . . . 21 Cn i . . . . . . . . . . . . . . . . . . . . . . . . . . . 22 Hin i . . . . . . . . . . . . . . . . . . . . . . . . . . . 23

2.2 i cng v lch s mn gii tch ton hc thi Hy Lp v Ma m c i . . . . . . . . . . . . . . . . . . . . . . . . . . . . . . 24 2.2.1 2.2.2 2.2.3 2.2.4 Pythagoras (580-500 trc Cng nguyn) . . . . . . . . . 24 Euclid (300 trc Cng nguyn) . . . . . . . . . . . . . . 32 Archimedes (287 - 212 trc Cng nguyn) . . . . . . . 36 Papus (th k th 4 sau Cng nguyn) . . . . . . . . . . 49
3

MC LC

Cc chuyn chuyn ton 3.1 Mt s k thut nh gi v c lng khi gii phng trnh i

51

s . . . . . . . . . . . . . . . . . . . . . . . . . . . . . . . . . . . 51 3.1.1 3.1.2 3.1.3 3.1.4 3.2 K nng s dng bt ng thc . . . . . . . . . . . . . . 51 K nng nh gi da vo "gi thit tm" . . . . . . . . 54 K nng nhm nghim kt hp nh gi . . . . . . . . . 55 Bi tp rn luyn . . . . . . . . . . . . . . . . . . . . . . 56

Ti liu tham kho . . . . . . . . . . . . . . . . . . . . . . . . . 56 p dng nh l Burnside-Frobenius vo bi ton t mu trong t hp . . . . . . . . . . . . . . . . . . . . . . . . . . . . . . . . 56 3.2.1 3.2.2 3.2.3 3.3 Mt s kin thc b tr v nhm v nh l BurnsideFrobenius . . . . . . . . . . . . . . . . . . . . . . . . . . 57 p dng vo bi ton t mu trong t hp . . . . . . . . 61 Bi tp tham kho . . . . . . . . . . . . . . . . . . . . . 65

Ti liu tham kho . . . . . . . . . . . . . . . . . . . . . . . . . 67 Chuyn chn lc v bt ng thc . . . . . . . . . . . . . . . 67 3.3.1 3.3.2 3.4 M u . . . . . . . . . . . . . . . . . . . . . . . . . . . 67 Ni dung . . . . . . . . . . . . . . . . . . . . . . . . . . . 68

Ti liu tham kho . . . . . . . . . . . . . . . . . . . . . . . . . 85 Mt s nhn xt v ging dy chuyn ng dng nguyn l Dirichlet . . . . . . . . . . . . . . . . . . . . . . . . . . . . . . . 86 3.4.1 3.4.2 3.4.3 3.4.4 3.4.5 Phn m u . . . . . . . . . . . . . . . . . . . . . . . . 86 Phn ni dung . . . . . . . . . . . . . . . . . . . . . . . 87 Bi tp vn dng . . . . . . . . . . . . . . . . . . . . . . 98 Hng dn cch gii . . . . . . . . . . . . . . . . . . . . 99 Kt lun . . . . . . . . . . . . . . . . . . . . . . . . . . . 104

Ti liu tham kho . . . . . . . . . . . . . . . . . . . . . . . . . 104

MC LC

3.5 S dng tnh n iu ca hm s tm gii hn . . . . . . . . 105 3.5.1 3.5.2 Cc tnh cht . . . . . . . . . . . . . . . . . . . . . . . . 105 Cc v d . . . . . . . . . . . . . . . . . . . . . . . . . . 108

Ti liu tham kho . . . . . . . . . . . . . . . . . . . . . . . . . 113 3.6 S phc v ng dng trong hnh hc . . . . . . . . . . . . . . . 114 3.6.1 3.6.2 3.6.3 3.6.4 M u . . . . . . . . . . . . . . . . . . . . . . . . . . . 114 M t mt s kt qu ca hnh hc phng bng ngn ng s phc . . . . . . . . . . . . . . . . . . . . . . . . . 114 Mt s v d p dng . . . . . . . . . . . . . . . . . . . . 120 Bi tp . . . . . . . . . . . . . . . . . . . . . . . . . . . 132

Li ni u

Ton hc l mt mn hc c bit quan trng trong chng trnh bc ph thng. Trong nhng nm gn y, cc thy gio, c gio v hc sinh cc trng trung hc ph thng chuyn v nng khiu c iu kin hi nhp vi cc chng trnh, cc chuyn ton quc t v khu vc thng qua cc hot ng hp tc, tham d cc k thi olympic v cc phng tin mng vin thng quc t. Nhiu dng ton mi hnh thnh, nhiu chuyn ton ph thng cp nht vi trnh tin tin ca cc nc pht trin, c bit nhiu chuyn ton hc gn vi ng dng v cc m hnh thc tin ngy cng lm cho cc ni dung ging dy v hc tp mn Ton hc trong trng ph thng ngy cng phong ph v a dng.

Ton hc khng nhng nhm gip trang b cho hc sinh nhng kin thc c th p dng trong cuc sng thng ngy m iu quan trng hn l cn cung cp, rn luyn cho hc sinh cc k nng, phng php t duy cht ch v logic, iu m cc em s cn thit trong c cuc i hot ng thc tin sau ny.

Nm nay, Tri h Hng Vng bc sang nm th 6, t chc ti trng


6

MC LC

THPT Chuyn Thi Nguyn. Cc cun K yu tri h Hng Vng ln th 2-5 ra i p ng c s mong i v k vng ca cc thy, cc c v cc em hc sinh trong khi cc trng trung hc ph thng chuyn khu vc min ni v trung du pha bc. Ngoi cc thi Olympic Ton Hng Vng, Olympic Ton H Ni m rng v Olympic quc t Singapore m rng, cun K yu cn gii thiu mt s phng php gii ton, cc k nng vn dng logic ton hc trong cuc sng ca cc gio s, cc nh khoa hc qua nhiu nm tm huyt vi chin lc o to ti nng tr ca t nc.

Nm nay, khi cc trng tham gia Tri h Hng Vng c bc tin di trn con ng hi nhp. Nhiu kin thc cp nht c cc thy c vit thnh cc chuyn , cc bi hc kinh nghim v cc trao i semina v hc thut thuc nhiu lnh vc l th ca ton hc. Ngoi ra cun K yu ln ny cn b sung cc thi thi Olympic Ton Hng Vng nm 2009, Olympic Ton H Ni m rng v Olympic quc t Singapore m rng ca nm 2010 v cc ton d tuyn do chnh cc trng ngh. Cun K yu ny gm cc chuyn t chn c sc theo chng trnh dnh cho cc lp chuyn Ton, l s kt tinh t kinh nghim ging dy v bi dng hc sinh ca cc thy gio, c gio cc trng THPT Chuyn cc tnh thnh Bc Giang, in Bin, Sn La, Ph Th, Vnh Phc, Lng Sn, Ha Bnh, H Giang, Tuyn Quang, Lo Cai, Qung Ninh, Yn Bi, Cao Bng, Bc Ninh, Bc cn v Thi Nguyn. Hy vng rng cun K yu ny s cung cp thm cho cc em hc sinh mt s kin thc b sung, gip cc em hiu su hn Sch gio khoa v chun b tt cho cc k thi hc sinh gii, Olympic, cc k thi tt nghip THPT, thi tuyn sinh vo i hc.

MC LC

Ngoi ra, trong cun sch cn trnh by hai ph lc c vit bng tin Anh cc em c iu kin lm quen vi cc ngn t, thut ng c bn, tip cn v tm hiu su thm cc kin thc cp nht qua mng internet v cc sch chuyn ca cc nc. Thay mt hi ng c vn khoa hc, xin chn thnh cm n cc thnh vin seminar ca Tri h Hng Vng, cc ng nghip, cc thy gio, c gio c v c nhng ng gp cho bn tho K yu c hon chnh.

Mi kin ng gp xin c gi v Ban T Chc Tri h Hng Vng ln th V, Trng THPT Chuyn Hng Vng Vit tr, Ph Th.

H Ni-Thi Nguyn, ngy 1-3 thng 8 nm 2010 Thay mt Hi ng c vn khoa hc GS Nguyn Vn Mu

MC LC

Chng 1

thi Olympic Ton Hng vng


1.1 Olympic Ton Hng vng ln th 1, nm 2005

Cu 1. Cc s nguyn dng a1 , a2 , a3 , a4 , a5 lp thnh mt cp s cng tng. Hi lp c bao nhiu cp s cng tho mn iu kin a1 > 50 v a5 < 100? Cu 2. Cc s nguyn dng a1 , a2 , a3 , a4 , a5 lp thnh mt cp s nhn tng. Hi lp c bao nhiu cp s nhn tho mn iu kin a5 < 100? Cu 3. Cc s dng a1 , a2 , a3 , a4 , a5 tho mn cc iu kin (i) 2a1 , 2a2 , 2a3 , 2a4 , 2a5 l cc s nguyn dng, (ii) a1 + a2 + a3 + a4 + a5 = 99. Tm gi tr ln nht ca tch P = a1 a2 a3 a4 a5 . Cu 4. Gi s tam thc bc hai f (x) lun lun dng vi mi x. Chng minh rng f (x) vit c di dng tng bnh phng ca hai nh thc bc nht. Cu 5. Gi s hm trng phng g(x) = x4 + bx2 + c lun lun dng vi mi x. Chng minh rng g(x) vit c di dng tng bnh phng ca hai tam thc bc hai.
10

1.2. Olympic Ton Hng vng ln th 2, nm 2006

11

Cu 6. Cho hnh vung ABCD. Tm qu tch cc im M thuc hnh vung (phn bn trong v bin ca hnh vung) sao cho din tch cc tam gic MAB v MAC bng nhau. Cu 7. Cho hnh vung ABCD. Gi s E l trung im cnh CD v F l mt im bn trong hnh vung. Xc nh v tr im Q thuc cnh AB sao cho AQE = BQF .

1.2

Olympic Ton Hng vng ln th 2, nm 2006

Cu 1. S o cc gc trong ca mt ng gic li c t l 2 : 3 : 3 : 5 : 5. S o ca gc nh nht bng [(A)] 200 , [(B)] 400 , [(C)] 600 , [(D)] 800 [(E)] 900 . Cu 2. Cho a = 0. Gii h phng trnh 2005 + y 2005 + z 2005 = a2005 x x2006 + y 2006 + z 2006 = a2006 2007 x + y 2007 + z 2007 = a2007 . Cu 3. Xc nh b s dng a, b, c sao cho ax9 y 12 + by 9 z 9 + cz 11 x8 15x4 y 8z 7 , x > 0, y > 0, z > 0.

Cu 4. Cho tam gic ABC v im M thuc BC. Xt hnh bnh hnh AP MN, trong P thuc AB v N thuc AC v hnh bnh hnh ABDC vi ng cho AD v BC. O l giao im ca BN v CP . Chng minh rng P MO = NMO khi v ch khi BDM = CDM. Cu 5. Cho s dng M. Xt cc tam thc bc hai g(x) = x2 + ax + b c nghim thc x1 , x2 v cc h s tho mn iu kin max{|a|, |b|, 1} = M.

12

Chng 1. thi Olympic Ton Hng vng

Tm gi tr ln nht ca biu thc (1 + |x1 |)(1 + |x2 |).

1.3

Olympic Ton Hng vng ln th 3, nm 2007

Cu 1. Mt a gic li c nhiu nht l bao nhiu gc nhn? (A) 2; (B) 3; (C) 4; (D) 5; (E) 6. Cu 2. Mt a gic li c nhiu nht l bao nhiu gc khng t? (A) 2; (B) 3; (C) 4; (D) 5; (E) 6. Cu 3. Xc nh hai ch s tn cng ca s sau M = 23 + 202006 + 2002007 + 20062008 ? (A) 04; (B) 34; (C) 24; (D) 14; (E) Khc cc p s nu. Cu 4. C n vin bi trong hp c gn nhn ln lt l 1, 2, . . . , n. Ngi ta ly ra mt vin bi th tng cc nhn ca s bi cn li l 5048. Hi vin bi c gn nhn l s no? (A) 1; (B) 2; (C) 3; (D) 4; (E) 5. Cu 5. Cho s t nhin abc chia ht cho 37. Chng minh rng cc s bca v cab cng chia ht cho 37. Cu 6. Cho 0 < a 2. Gii h phng trnh sau x + 1 = ay x 1 y + = az y 1 z + = ax. z

1.4. Olympic Ton Hng vng ln th 4, nm 2008

13

Cu 7. Cho hnh bnh hnh ABCD c AB < BC. ng phn gic BP ca gc ABC ct AD P . Bit rng P BC l tam gic cn, P B = P C = 6cm v P D = 5cm. Tnh di cc cnh ca hnh bnh hnh. Cu 8. Chng minh rng tam thc bc hai g(x) = 3x2 2ax + b c nghim a=++ b = + + . Cu 9. Cho ba s dng a1 , a2 , a3 . Cc s nguyn 1 , 2 , 3 v 1 , 2 , 3 cho trc tho mn cc iu kin a1 1 + a2 2 + a3 3 = 0 a1 1 + a2 2 + a3 3 = 0. Tm gi tr nh nht ca biu thc M = a1 x1 y 1 + a2 x2 y 2 + a3 x3 y 3 , x > 0, y > 0. Cu 10. Tnh M= 1 1 . + cos 5 cos 3 5

khi v ch khi tn ti b s , , sao cho

1.4

Olympic Ton Hng vng ln th 4, nm 2008

Cu 1. Hai ch s tn cng ca s M = 22008 l (A) 16, (B) 36, (C) 56, (D) 76, (E) khng phi l cc p s trn Cu 2. Cho m, n l cc s nguyn dng sao cho s A = m2 + 5mn + 9n2 c ch s tn cng bng 0. Khi hai ch s tn cng ca A l (A) 00, (B) 20, (C) 40, (D) 60, (E) khng phi l cc p s trn Cu 3. Hi c bao nhiu s nguyn t 1 n 2008 ng thi khng chia ht cho 2, 3 v 5?

14

Chng 1. thi Olympic Ton Hng vng

Cu 4. Gii h phng trnh sau x + xy + y = 5 y + yz + z = 11 z + zx + x = 7 Cu 5. C th tm c hay khng nm s nguyn sao cho cc tng ca tng cp trong nm s lp thnh mi s nguyn lin tip? Cu 6. Chng minh rng tn ti s t nhin A c 4 ch s tn cng l 2008 v chia ht cho 2009. Cu 7. Xt hnh thoi ABCD cnh bng a. Gi r1 , r2 ln lt l bn knh cc ng trn ngoi tip cc tam gic ABD, ABC. Chng minh rng gi tr ca biu thc a r1 lun lun khng i. Cu 8. Gii phng trnh sau 3 4x2 + 2 = 3 4x3 + x
2

a r2

Cu 9. Cho ba s thc x, y, z tha mn iu kin x2 + y 2 + z 2 + xy + yz + zx = 25. Tm gi tr nh nht ca biu thc T = x2 + 3y 2 + 9z 2 .

1.5. Olympic Ton Hng vng ln th 5, nm 2009

15

1.5

Olympic Ton Hng vng ln th 5, nm 2009

Cu 1. Chng minh rng t 2009 s t nhin ty u c th chn c mt hoc mt s s m tng ca n chia ht cho 2009. Cu 2. Tm b ba s nguyn t lin tip (lin k) sao cho tng bnh phng ca chng cng l mt s nguyn t. Cu 3. Trong 100 hc sinh h chuyn c 29 em gii ton, 30 em gii vn, 42 em gii nhc. Trong s c 8 em va gii ton, va gii vn, 10 em va gii nhc va gii ton, 5 em va gii nhc va gii vn, c ba em gii c ba mn. Hi c bao nhiu em ch gii ton, ch gii vn, ch gii nhc v bao nhiu em khng gii mn no? Cu 4. Cho f, g xc nh v tha mn h thc f (x + 6) + 2g(2x + 15) = 1 (x + 2) 2 x + 2 + g(x + 5) = x + 4. f 2 Cu 5. Tm tt c cc cp s (x, y) tha mn ng thc 2(x2 + 1)(y 2 + 1) = (xy + 1)(x + 1)(y + 1).

Hy xc nh f (x) v g(x).

Cu 6. Cho hnh vung ABCD c di cnh bng 2cm, M l mt im di ng trn mt phng cha hnh vung sao cho MA2 + MB 2 = MC 2 . Tnh khong cch ln nht t im M ti im D. Cu 7. Cho tam gic ABC khng cn ni tip trong ng trn tm O bn knh R. Tm qu tch nhng im M trong tam gic ABC sao cho MA MB MC + + = 3, MA MB MC

16

Chng 1. thi Olympic Ton Hng vng

trong A , B , C ln lt l giao ca MA, MB, MC vi ng trn cho. Cu 8. Tng ca mt s cc s nguyn dng l 2009. Tm gi tr ln nht ca tch cc s nguyn dng cho.

Cu 9. Tm tt c cc a thc f (x) vi h s l cc s nguyn khng m nh hn 8 v tho mn iu kin f (8) = 2009.

1.6

p n Olympic Ton Hng vng ln th 5-2009

Cu 1. Gi 2009 s cho l a1 ; a2 ; a3 ; . . . ; a2009 . Xt 2009 tng sau: S1 = a1 S2 = a1 + a2 S3 = a1 + a2 + a3 ...... S2009 = a1 + a2 + a3 + cdots + a2009 Nu tn tai mt trong cc tng trn chia ht cho 2009 lun th ta c lun iu phi chng minh. Nu trong cc tng trn khng tn ti tng no chia ht cho 2009. Ta xt ng d ca cc tng trn khi chia cho 2009. Lc ny tp s d khi chia 2009 ca Theo nguyn l Drichlet ta c t nht 2 trong s cc tng trn c cng s d . khi chia cho 2009. Gi s 2 tng l Si v Sj . |Si Sj |. .2009. Ta c iu tng ny l: S = {1; 2; 3; ...; 2008} .

phi chng minh.

Cu 2. Gi 3 s nguyn t lin tip l p, q, r vi 2 p < q < s. l s nguyn t nn khng tha mn.

B ba s nguyn t lin tip u tin l 2,3,5 c 22 + 32 + 52 = 38 khng

1.6. p n Olympic Ton Hng vng ln th 5-2009

17

B ba s nguyn t lin tip tip theo l 3,5,7 c 32 + 52 + 72 = 83 l s nguyn t nn l b ba tha mn bi. Xt p > 3, th hin nhin q, r > 3. Nhn xt rng cc s nguyn t ny phng ca chng lun chia ht cho 3, khng phi l s nguyn t. nht tha mn bi. Cu 3. Dng s Ven ta thu c: - S em ch gii Ton l 14. - S em ch gii Vn l 20. - S em ch gii Nhc l 30. - S em khng gii mn no l 19. u c dng 1( mod 6) v khng chia ht cho 2 v 3. V th nn tng bnh Vy b ba s nguyn t lin tip tip (3,5,7) l b ba s nguyn t duy

Cu 4. Ta c f (x + 6) + 2g(2x + 15) = 1 (x + 2) 2 x + 2 ) + g(x + 5) = x + 4. f( 2 (1) (2)

Trong (2) thay x bi 2x + 10 ta c f (x + 6) + g(2x + 15) = 2x + 14. T ta c h 1 f (x + 6) + 2g(2x + 15) = (x + 2) 2 f (x + 6) + g(2x + 5) = 2x + 14.

Gii h ny ta tm c f (x + 6) = 7x + 54 (x + 2) (3) 2 g(2x + 15) = 3x 26 . (4) 2 7x + 12 , trong (4) thay x bi Trong (3) thay x bi x 6 ta tm c f (x) = 2 3x 7 x 15 ta tm c g(x) = . 2 4

18

Chng 1. thi Olympic Ton Hng vng

Cu 5. Theo bt ng thc Cauchy (Bunhiacopski), ta c 2(x2 + 1) (x + 1)2 , 2(y 2 + 1) (y + 1)2 , (x2 + 1)(y 2 + 1) (xy + 1)2 . Du ng thc xy ra khi v ch khi x = y = 1. Suy ra [2(x2 + 1)(y 2 + 1)]2 [(x + 1)(y + 1)(xy + 1)]2 , hay 2(x2 + 1)(y 2 + 1) |(x + 1)(y + 1)(xy + 1)| (x + 1)(y + 1)(xy + 1). Vy c ng thc, ta phi c (x, y) = (1, 1). Cu 6. Khng gim tnh tng qut ta gi thit hnh vung ABCD c cc nh A, B, C, D theo th t ngc chiu kim ng h. Lp h trc ta Oxy c nh O(0; 0), A(2; 0), C(0; 2), B(2; 2), gi M(x; y). Theo gi thit ta c MA2 + MB 2 = MC 2 (x 2)2 + y 2 + (x 2)2 + (y 2)2 = x2 + (y 2)2 x2 8x + 8 + y 2 = 0 (x 4)2 + y 2 = 8. Phng trnh (1) l phng trnh ca ng trn c tm I(4; 0) thuc trc Ox bn knh R = 2 2. Suy ra khong cch ln nht t M ti D l d = MI + R = 4 + 2 2.

Cu 7. Ta c MA.MA = MB.MB = MC.MC = R2 MO 2 . Suy ra = MA2 MB 2 MC 2 MA MB MC + + = + + MA MB MC MA.MA MB.MB MC.MC = M MA2 + MB 2 + MC 2 . R2 MO 2

MA2 + MB 2 + MC 2 = 3MG2 + GA2 + GB 2 + GC 2

1.6. p n Olympic Ton Hng vng ln th 5-2009

19

= 3MG2 + OA2 + OB 2 + OC 2 3GO 2 = 3MG2 + 3R2 3GO 2 . Do vy = 3 v MG2 + MO 2 = OG2 , tc qu tch M l ng trn ng knh OM.

Cu 8. Ta c mt s nhn xt sau: - Nhn xt 1: vi x1 , x2 , , xk l cc s nguyn dng th x1 +x2 + +xk +1 = x1 +x2 + +(xk +1) v x1 .x2 xk .1 < x1 .x2 (xk +1). Do tch ca cc s nguyn c tng bng 2009 l ln nht khi cc s nguyn ln hn hoc bng 2. - Nhn xt 2: vi s n > 4, ta c 2(n 2) > n, do trong cc s phi tm

khng th c s ln hn 4, v nu c s n nh th th ta tch thnh hai s 2 v thun vi iu kin ln nht ca tch. n 2 th tng ca chng vn l 2009, trong khi tch ca chng ln hn, mu

- Nhn xt 3: Do 23 < 32 , nn trong cc s cn tm khng th c nhiu hn hai s 2, v khi ta thay ba s 2 bi hai s 3 c mt tch ln hn. - Nhn xt 4: Trong cc s cn tm khng th va c s 2 va c s 4, v khi ta c th thay s 2 v s 4 bi hai s 3 thu c mt tch ln hn. T cc nhn xt trn ta suy ra cc s cn tm s gm cc ch s 3 v mt hoc hai s 2 hoc mt s 4. Nhng ta c 2009 = 669.3 + 2, do cc s cn tm c mt s 2 v 669 s 3, khi tch ca chng t gi tr ln nht l 2.3669 .

Cu 9. Ta c MA.MA = MB.MB = MC.MC = R2 MO 2 .

20

Chng 1. thi Olympic Ton Hng vng

Suy ra MB MC MA2 MB 2 MC 2 MA + + = + + MA MB MC MA .MA MB .MB MC .MC MA2 + MB 2 + MC 2 R2 MO 2

m Do vy qu tch ca M l ng trn ng knh OM. Cu 10. Xt a thc f (x) = a0 xn + a1 xn1 + + an , trong a0 , a1 , . . . , an

l cc s nguyn khng m v nh hn 8. Do f (8) = 2009 nn a0 8n + a1 8n1 + + an = 2009. Thc hin php chia 2009 cho 8 c d a0 = 1. Li ly thng ca php chia ny cho 8 ta c a1 = 3, lin tip thc hin php chia nh th ta c a thc cn tm l: f (x) = 3x3 + 7x2 + 3x + 1.

Chng 2

i cng v lch s mn gii tch ton hc


2.1
2.1.1

Tm lc lch s mn gii tch


Hy Lp v Ma m c i

Pythagoras (580-500 trc cng nguyn) nh l Pythagoras v tam gic vung; s v t Euclid (300 trc Cng nguyn)

2.

C quyn lc nht trong cc nh ton hc cng thi vi ng. nh l Euclid v s hon ho v v hn cc s nguyn t. Arcgimedes (287-212 trc Cng nguyn) Xc nh c tip tuyn, din tch v th tch ch yu bng php tnh vi phn; tm th tch v din tch mt ca mt hnh cu; trng tm i vi trng lc; ng xon c Arcgimedes; tnh c s. Pappus (Th k th t sau Cng nguyn) Trng tm ca trng lc i vi cc vt th v mt cong trn xoay. 2.1.2 Trung c

Descartes (1596-1650)
21

22

Chng 2. i cng v lch s mn gii tch ton hc

c coi l ng t ca hnh hc gii tch; a ra mt vi khi nim tuyt vi. Mersenne (1588-1648) Chng minh li cc tng; ng cycloid; s nguyn t Mersenne. Fermat (1601-1665) Thc s tm ra hnh hc gii tch; tnh ton v s dng o hm v tch phn; sng lp ra gii tch s hin i; xc sut. Pascal (1623-1662) Php quy np ton hc; h s nh thc; cycloid; nh l Pascal trong hnh hc; xc sut; c nh hng t Leibnitz. Huygens (1629-1695) Dy s, cycloid; s vn ng vng trn; Dy hc ton ca Leibnitz (ai l gio vin; ai l hc sinh). 2.1.3 Cn i

Newton (1642-1727) ng sng to ra php tnh vi phn; tm ra nh l c bn; s dng chui s; gn nh l ngi sng to ra thin vn hc v vt l nh l mt ngnh khoa hc Ton. Leibnitz Cc sng to ca ng l cc dng tt nht ca php tnh vi phn; tm ra nh l c bn; sng to ra mt vi khi nim qu; dy anh em nh Bernoulli.

Anh em nh Bernoulli (James 1654-1705, John 1667-1748) Hc c php tnh vi phn t Leibnitz v pht trin p dng n mt cch tng qut; chui s; John l thy gio ca Euler

2.1. Tm lc lch s mn gii tch

23

Euler (1707-1783) Lm vic trn php tnh vi phn v pht trin n rt tng qut; h thng ho hnh hc gii tch v lng gic; a ra cc k hiu e, , i, f (x), sin x, cos x; chui v cc tnh cht; php tnh vi phn i vi s bin thin. Lagrange (1736-1813) Php tnh vi phn i vi s bin thin; c hc gii tch. Laplace (1749-1827) C hc v tr, l thuyt xc sut v s tin b ca con ngi. Fourier (1768-1830) Chui Fourier; phng trnh truyn nhit. 2.1.4 Hin i

Gauss (1777-1855) Khi xng ton hc chnh xc vi chng minh hi t ca chui; l thuyt s; s phc trong gii tch; i s v l thuyt s; hnh hc vi phn; hnh hc phi Euclid; v.v. . . Cauchy (1789-1857) X l mt cch k lng v gii hn, lin tc, o hm, tch phn, chui, gii tch phc. Abel (1802-1829) Chui nh thc, phng trnh bc nm; php tnh tch phn; hm elliptic. Dirichlet (1805-1859) Mt ngi c rt nhiu ng gp trong vic xy dng nhng gi tr bn vng cho gii tch v l thuyt s. Liouville (1809-1882) Tch phn ca nhng hm c bn, s siu vit.

24

Chng 2. i cng v lch s mn gii tch ton hc

Riemann (1826-1866) Tch phn Rimann; nh l hon v Riemann; hnh hc Riemann; hm zeta Riemann; gii tch phc.

2.2
2.2.1

i cng v lch s mn gii tch ton hc thi Hy Lp v Ma m c i


Pythagoras (580-500 trc Cng nguyn)
Ba phn nm thin ti v hai phn nm l nhng iu v vn J.R.Lowell

Nn vn minh phng Ty nh mt dng sng ln chy theo thi gian, c nui dng v lm giu bi nhiu cng hin phong ph t cc nn vn ha khc. Hy cho tr tng tng ca chng ta ngc dng thi gian quay li vi ngn nm trc, u ngun ca nn vn minh Hy Lp c i. Ni y, ti u ngun ca dng sng, ng trong my m bc tng Pythagoras hin ln huyn o. Cho n by gi hu ht mi ngi u ngh Pythagoras l mt nh ton hc nhng vi nhng ngi cng thi, ng c coi nh mt ngi thy ca s thng thi, mt nh tn ngng, mt v thnh. Mt thy ph thu, mt lang bm, hay mt nh chnh tr tin phong tu theo tng quan im. Trong cc t chc sng bi ng, cc mn ca ng pht trin cc tng ca ng trong sut thi k vn minh Hy Lp. Ton hc bt u vi ng bng quan nim u tin ca ng rng n l mt h thng c t chc v c th lin kt vi nhau bi s chng minh cht ch. ng l ngi u tin s dng t mathemtike c ngha l ton hc. Trc ng ch c t mathemata ngha l kin thc hoc vic hc ni chung. Trong cm nhn ca ng, mi th trong khoa hc u c th d on c, c th hiu c v c th a ra cc bng chng cht ch. ng l ngi u tin p dng t kosmos - hi ho theo mt trt t - cho hu ht cc lnh vc.

2.2. i cng v lch s mn gii tch ton hc thi Hy Lp v Ma m c i

25

Cm nhn u tin ca ng v trit hc phng Ty l nhng kin ca ng v t nhin m hai th k sau chnh l ci ngun cho hc thuyt Plato v tt c cc t tng ca ng c nhc li rt nhiu mt cch c h thng trong hc thuyt. Thm tr ng c coi nh l ng t ca nn trit hc. ng tng dng t philosophia - tnh yu i vi khoa hc thay cho Sophia (s thng thi) ging nh s khoe khoang nhng hiu bit ca con ngi. Bt c ai bt u s nghip ca mnh cng mun c nhng thnh cng cng b vi mi ngi. Liu chng ta c nn tin rng 3 phm cht sau y cng tn ti trong mt con ngi? Hy xem chng din ra nh th no. u tin c th ni g v cuc i ng? ng l ngi cng thi vi Confucius, Budda v Zoroaster. Cng nh nhng nhn vt ni ting ny, t thi s khai ca loi ngi, Pythagoras c chng ta bit n ch qua truyn thuyt v nhng ghi chp cn li hng trm nm sau khi ng cht. Theo truyn thuyt, ng sinh ra o Samos, ngoi khi b bin pha Ty Tiu . Thi thanh nin, ng l mt ngi rt ham hc v i chu du sut 30 nm Ai Cp, Babylon, Phoenicia, Syria v c l cua Persia v n . Trong sut cuc hnh trnh ca mnh, ng thu c nhng kinh nghim ban u v thin vn hc v ton hc nguyn thu. Khi tr v Samos ng khng hi lng vi nhng g chng kin y - mt bo cha c ti nhng thiu s ng cm - v tui 50 ng c tr Hy Lp - thuc a ca Crotana pha nam nc . y cuc i chnh tr ca ng bt u. ng lm thy gio v lp ra trng Pythagorean ni ting trong kt hp hng trm mn hc vi nhng i hi danh d nh mt trng i hc u tin trn th gii. Ban u trng hc ny dng nh l mt gio hi vi mc tiu ci tin o c x hi v l ni tp trung cc hot ng tr thc. Tuy nhin x hi khng phi lc no cng hoan nghnh nhng ci tin o c, v nhng ngi khc coi hi

26

Chng 2. i cng v lch s mn gii tch ton hc

Pythagorean nh l mt ng chnh tr xc phm n cc nguyn t o c v tn gio. Thm tr nhng hot ng chnh tr ngy cng tng ca h khuy ng s gin d ca cng chng, n mt chng mc no h b n p mnh m, trng hc b t ph. Pythagoras chy trn n gn thuc a ca Metapontum, ng cht y khi tui cao. Nhng ngi trong hi Pythagorean cn sng st d sng di rc khp a Trung Hi vn gi lng trung thnh v tip tc trng phi trit hc y hn mt th k sau. y l s trung thnh vi ci g vy? Quan im m u l thuyt Pythagoras v linh hn v thc th vt cht - lng tin c c kt t nhng kinh nghim ca ng khi Ai Cp v chu . ng tin vo thuyt lun hi hay s u thai ca mi linh hn sau ci cht t th xc ny sang th xc khc ca con ngi cng nh loi vt. Mi linh hn tip tc qu trnh u thai mt cch khng hn nh, ln hoc xung thnh ng vt cao hn hoc thp hn tu theo nhng phm cht xng ng c khen thng hay nhng li lm khuyt im ca mnh. Ch c mt cch duy nht thot khi gung quay ca s phn ny c s siu thot l thng qua s sm hi c th xc v tm hn. Nhng kin ny, d l k qui i vi suy ngh hin thi vn lan rng trong ngi i xa ng vai tr nghi thc trong nhiu gio gii. Cc mn Pythagorean c gn b vi nhau bi li th trung thnh vi ngi khc trong hi v tun theo mt th lnh, s sm hi c th hin theo nhiu cch khc nhau. H chia s vi nhau mi th v vt cht. H n mc gin d, hnh x khim tn, khng ci hoc th thm. H b cm n ht v tht. Lnh cm n c l l iu k quc nht trong iu cm k nguyn thu v ch ngha n chay l mt bin php phng nga t nhin chng li nhng iu gh tm trong n ung ca t tin. Cng nh vy, ung nc thay ru c khuyn khch - cng l li khuyn ca nhng nh thng thi hoi nghi min nam nc hin nay.

2.2. i cng v lch s mn gii tch ton hc thi Hy Lp v Ma m c i

27

Truyn thuyt miu t rng Pythagoras hn hn tt c cc hc tr ca ng v s thnh cng v hon ho trong cuc sng theo nhng tiu chun ny. Uy tn v tri thc uyn thm v o c ca ng ln n ni nhm t m ng s dng autuspha - t chu trch nhim vi chnh hnh ng ca mnh tr thnh khu hiu cho quyt nh cui cng trong bt k vn no ca h. cng l thi quen a ra tt c cc tng v khm ph cho ngi th lnh, chnh iu lm cho chng ta kh c th phn bit nhng thnh qu ca ng vi nhng thnh cng ca ng c s ng gp ca cc mn . Nh chng ta ni trn, hi Pythagoras th hin s sm hi ca th xc qua s khc kh, s tit ch v s iu . y l s ph bin v n by gi vn ph bin nhiu vng ca min ng. iu c bit Pythagoras nm trong khoa hc m ng nghin cu nhm t ti s sm hi v tinh thn thng qua vic nghin cu tch cc v mn ton hc v khoa hc khc. y l s chng i kch lit vic b ng trong suy ngh b chi phi bi hu ht nhng s th cng thn b. Khoa hc ca Pythagoras to ngun cho nh hng to ln ca ng ti nn vn minh phng Ty v ghi du mt phn trong c trng ring bit ca nn vn minh ny nh l n pht trin sut 2500 nm qua. Kho hc m Pythagoras yu cu bn mn hc: hnh hc, s hc, nhc v thin vn hc. Trong thi k trung i, nhm cc mn hc ny c bit n nh l quadrivium v sau c m rng thm thnh trivium gm ng php, tu t hc v logic. l by mn ngh thut rng ri c coi l phn ch yu ca gio dc bt k mt con ngi c vn ho no. Ton hc Hy Lp gn nh l mt trong nhng thnh tu tri thc ln nht ca lch s nhn loi. Pythagoras bt u tt c, khng phi ch l trong suy on thc t ca vin th k ngi Babylon hay vin kim sot ngi Ai Cp, m l chnh ng, theo nh mt mn thn cn, ng l ngi c ti nng ngh ln cao hn mc bnh thng. Trc ng ch c mt vi quy tc

28

Chng 2. i cng v lch s mn gii tch ton hc

tch bit v hnh hc c c nh vo kinh nghim thc tin v dng nh l ngi sng to ra cc m hnh v nh ngha, tiu , nh l v chng minh, theo cc cu trc phc tp ca hnh hc c sinh ra t mt s t cc gi thit c t ra mt cch r rng t nhng suy din cht ch. Truyn thuyt cho rng ng ngh ra cc tng chng minh ton hc. ng pht minh ra nhiu nh l: tng cc gc trong mt tam gic bt k bng hai gc vung v nh l Pythagoras ni ting v bnh phng cnh huyn ca mt tam gic vung. Theo truyn thuyt k li rng ng rt vui khi pht minh ra nh l tuyt vi ny n ni ng hin dng mt con b c cm t, m y l mt hnh ng vi phm ln n c tin ca hi Pythagorean. Ngi cng hi ng cng bit nhng tnh cht ca nhng ng thng song song v nhng tam gic ng dng v sp xp tt c nhng iu ny trong mt h thng logic c gn kt cht ch gn nh tng ng vi hai cun sch u tin C s ca Euclid (300 nm trc Cng nguyn). iu chng t rng, bt u t nhng iu u tin m h pht minh ra mn hnh hc nhiu bng chng trnh hc ca na u chng trnh hc ph thng hin nay. Hi Pythagorean cng m li cho mn S hc - khng nhng cho kh nng tnh ton hu ch m cn v l thuyt s tru tng. C l h l nhng ngi u tin chia cc s thnh cc lp chn v l, nguyn t v khng nguyn t. . . Biu din bng hnh nh cc s l nim say m ca h, vic ny c ny sinh bng s sp xp cc du chm theo mt m hnh hnh hc thng dng. Chng ta hy xem tam gic s: 1, 3, 6, 10 . . . l s lng cc du chm dy hnh tam gic sau:

din cc s chnh phng 1, 4, 9, 16, . . . nh sau:

Hin nhin y l nhng s c dng 1 + 2 + 3 + + n. Cng c th biu

2.2. i cng v lch s mn gii tch ton hc thi Hy Lp v Ma m c i

29

Tm li, mi s chnh phng c to ra t nhng s trc bng cch thm vo ng bin hnh L gi l glomon ngh l thc vung ca ngi th mc. Hi Pythagorean a ra nhiu thc t th v v k hiu s ch dng hnh nh. V d, t nhng thc vung ca ngi th mc t lin tip, lp tc suy ra mt cch r rng rng tng ca n s l u tin bng n2 : 1 + 3 + 5 + + (2n + 1) = n2 . Hon ton tng t, cng thc: 1 1 + 2 + 3 + + n = n(n + 1). 2 Suy t tam gic s c th chng minh cho cng thc hin nhin: 2 + 4 + 6 + + (2n) = n(n + 1). V tri ca ng thc l tng ca n s chn u tin v ng thc c hnh dung ngay khi tng ny biu din dng dnh ch nht vi n du chm. Trn mt cnh v n+1 du chm trn cnh kia nh sau:

C kin cho rng hi Pythagorean coi ton hc l cha kho gii thch v cc cu trc t nhin, v c l vi bn thn Pythagoras cng vy. Pht hin ny ny sinh ra t mt th nghim thng thng vi m nhc. Pythagoras ko cng dy cho cy n lia gia hai ci mc trn con thuyn. Khi dy n c gy ln n pht ra m thanh rt chun. ng nhn ra rng khi dy n b chn bi mt vt di ng c gi vo gia dy v thuyn th nu phn dy dng gy gim ch cn mt na so vi di ban u ca n th n pht ra m thanh c trng bng 1/8 (qung 8) m thanh ban u; v nu gim 2/3

30

Chng 2. i cng v lch s mn gii tch ton hc

di dy th m thanh pht ra bng 1/5 m thanh ban u; v nu gim 3/4 di dy th m thanh pht ra bng 1/4 m thanh ban u. Qung 8,5,4 l nim m u v s du dng m sau ny chng ta quen thuc. Trng phi Pythagorean gy n tng su sc trong vic nhn mnh mi lin h r rt gia cc phn s 1/2, 2/3, 3/4 v trng ca nt nhc m ngi sng tc da trn nhng suy xt hon ton mang tnh thm m. Hn na, nh l h qu tip theo, h cho rng mi ngi di chuyn trong khng gian pht ra mt m thanh c cng t l vi tc di chuyn. Do vy cc hnh tinh chuyn ng vi nhng tc khc nhau trong cc qu o ring ca chng xung quanh Tri t pht ra bn ho m ca bu tri, cn gi l m nhc ca bu kh quyn. ng gp thm cho thin vn hc, Pythagoras cng xc nhn rng Tri t hnh cu - c l v l do n gin bi hnh cu l mt khi cht rn p nht. Quy lut v trng m nhc c miu t y l s nh lng u tin c khm ph v th gii t nhin. Cng vi n trit l hin nhin c m rng trn cc hnh tinh, iu khin Pythagoias tin chc rng cc s, gm cc s nguyn v phn s u i din cho tt c mi th. Mi th u l s tr thnh khu hiu ca h, khng ch c ngha c bn m cn l bn cht bt bin ca bt k mt s vt no. Nhng hc thuyt ny tr nn i lp vi hnh hc. Bi v mi th u l s - ngha l cc s hu t, v khng c s no khc - bng chng l chiu di ca bt k mt on ct no cng phi l chiu di ca bt k mt on ct no khc vi mt s hu t. Khng may rng iu ny l sai, v ngay sau h pht hin ra rng t inh l Pythagoras suy ra hnh vung c cnh bng mt c di ng cho l 2, v theo nhng g bit Pythagoras chng minh rng khng c s hu t no bnh phng ln bng 2. Sai lm ny n n s i u gia hai nhm hc tr trong hi: mt bn khng tin v

2.2. i cng v lch s mn gii tch ton hc thi Hy Lp v Ma m c i

31

bn kia l khng chp nhn sai lm ny. Mt bn ngh rng ng cho hnh vung cnh bng 1 th khng c d di, cn bn kia cho l khng ng v mi th u l s. S sp ca cc hc thuyt truyn thng v mt s khng hu t l mt c sc i vi h v h gi kn iu ny. Tuy nhin vic pht hin ra s v t l mt thnh tu xut sc nht ca ton hc Hy Lp c i. D vi sai lm trn, Pythagoras v cc mn ca ng vn gi c tin vi s. Nu thc s ph nhn cc s th ton b cng lao ca h s mt. H cm tt c cc mn nghin cu v vn ny v gi cc vn ny trong bc mn huyn b. Ging nh bt k mt gio l no, trong c tin ca trng phi Pythagoras tht kh nhng iu khng quen tr nn ng tin. Khi nim ct li trong h thng ca h l b t linh thing, gm cc s 1, 2, 3, 4 m tng 10 s linh thing - linh thing bi 1 l im, 2 l ng thng, 3 l mt, 4 l khi v do 1 + 2 + 3 + 4 = 10 l tt c, l s ca vn vt. iu khng nh rng tt c cc phn s m h c hc 1/2, 2/3, 3/4 u l t s lin tip ca cc s 1, 2, 3, 4 v c lin kt cht ch vi s ho m trong m nhc, k c h thng thp phn ca chng ta cng l s hu t. H cng ch ra rng s l (tr 1) l ging c v s chn l ging ci. Hn na h tin rng mt s u c du hiu ring ca chng, v nh s 1 l s to ra tt c cc s, l c Cha tri, 2 th a dng v l s ging ci u tin, 3 = 1 + 2 l s ging c u bng, 5 = 3 + 2 l s ca tic ci, l s kt hp ca mt nam v mt n, 6 = 1 + 2 + 3 l s hon ho, bi n l tng ca cc c s ca n, v nhng c ny l thng nht, a dng v l b ba ca thnh v c ngha lan truyn trong Thin cha gio thi k c i. i vi chng ta, tm quan trng ca m hn n nhng s th phng k tin, l s kt hp thng nht v a dng, 4 = 2 + 2 = 2 2 l s ca s cng

32

Chng 2. i cng v lch s mn gii tch ton hc

cc l n vt ln c t tng ca Plato (428 - 438 trc Cng nguyn) v lm ny sinh s thay i mnh m nh mt dng chy ca c tin qua cc trng phi Thin cha gio c i, trung i v thi k phc hng v n vn c nh hng ln cho ti ngy nay. Plato l ngi c tr tu phi thng trong nn vn minh nhn loi. Hng chc tc phm ln ca ng c lu gi vi s yu mn v khm phc ca ton nhn loi vi nhng gi tr su sc v y cht th v v nhn vt chnh trong tc phm ca ng Socrates. Hnh tng Socrates trong suy ngh ca Plato rt quan tm n s cng bng trong x hi, vi o c tt p, s khn ngoan v s trn tr cho mt cuc sng ngy cng tt p hn. Ngoi tnh yu v s ca tng i vi Socrates, Plato cn rt say m ton hc. Trong nhng nm trung nin ng dnh thi gian ng k min nam nc lin h vi gio phi Pythagorean - nhng ngi m trit l ca h l ton hc nhng uy lc li l tn gio v s thn b.

2.2.2

Euclid (300 trc Cng nguyn)


B sch C s ca Euclid l mt trong nhng b sch v i nht tng c vit Bertrand Rusell

l mt trong nhng iu i lp vi bt c tiu chun gio dc no tng c bit n i vi vic ging dy v o to trong sut 23 th k qua. Cun Element (C s) m u bng phn hnh hc m khng yu cu ngi c phi c hiu bit v kinh nghim trc khi c n. N khng a ra mt s gii thch km theo v khng a ra mt nhn xt c th no. N khng c ni dung lin quan trc tip ti khoa hc v thm tr n khng gi ti mt s ng dng no. Cun sch ny cng khng c sc cc vn nu ra theo bi cnh lch s v ton hc v cng khng nu tn ca bt k mt

2.2. i cng v lch s mn gii tch ton hc thi Hy Lp v Ma m c i

33

ngi no. S ra i ca cun sch c Bible so snh nh Cha tri to ra c Thin ng v Tri t - Cun Element bt u vi nh ngha Mt im l th khng c b phn. Cun sch gm 13 cun v 465 mnh khng c tho lun theo mt cch no c. Hu ht mi ngi u ngc nhin bi cun C s dng nh ch c mt tc gi. Vy Euclid l ai m tn ng ng ngha vi hnh hc n tn th k 20 vy? Ch c 3 iu thc t sau y chng ta s bit v ng. Nhng thc t ny l: ng tr hn Plato (428 trc Cng nguyn), ng gi hn Archimedes (287 trc Cng nguyn) v ng dy hoc Alexandria. Khi vua Alexander cht nm 323 trc Cng nguyn, ch chu Phi do Ptolemy tha k. Ptolemy a Euclid t Athen v Alexandria tham gia m trung tm gio dc Hellenistic s - c bit n nh mt Vin bo tng, vi th vin ni ting - ni cc ti liu ca ng c tm thy y. Tc truyn rng: mt ln Ptolemy hi Euclid liu c con ng no ngn hn n vi hnh hc hn l cun Element khng, ng tr li ngay rng trong hnh hc khng c con ng dnh ring cho vua cha. C ngi bt u c cun hnh hc ca Euclid, khi c mnh u tin hi ng: ti c th hc c g t nhng th ny?. Euclid gi ngi n l ca ng v p: hy a cho ng ny mt xu, ng ta s ni ci li m ng ta nhn c t cun sch ny. Ngoi vic tnh ton c h thng ca mn hnh hc c s, cun Element cng bao gm tt c nhng g c bit n thi by gi v l thuyt c s. Vai tr ca Euclid nh l mt tc gi chnh t chc v sp xp li cc pht minh ri rc ca cc bc tin bi. C th ng ch gp thm mt s kin v chng minh ca mnh trong mt s nh l quan trng nhng iu cng lm tng thm uy tn cho ng. Cun I ca b Element bt u vi 23 nh ngha (im, ng thng,

34

Chng 2. i cng v lch s mn gii tch ton hc

ng trn, . . . ) 5 mnh v 5 tin hoc khi nim chung. Trong trit hc Hy Lp, tin c hiu nh mt s cng nhn chung cho tt c cc lnh vc nghin cu, trong khi mnh c coi l s gi nh (gi thuyt) ch c ngha trong phm vi mt mm khoa hc v cn phi bn bc (VD: qua 2 im c th xc nh c mt ng thng). S khc nhau ny c b qua trong ton hc hin i, v hin nay t mnh v tin c th c s dng thay th cho nhau. Ni chung quyn I n quyn VI vit v hnh hc phng, quyn VII n quyn IX vit v l thuyt s, quyn X vit v s v t v quyn XI n quyn XVI vit v hnh hc khng gian. nh ngha th 47 trong quyn I (thng k hiu l I.47) l nh l Pythagoras. Sau y l mt vi chnh gy c s quan tm c bit: VII.1 v VII.2 a ra thuc ton Euclid tm c chung ln nht ca 2 s nguyn dng, VIII.30 l b Euclid khng nh rng mt s l tch ca 2 s nguyn dng s chia ht cho mt trong 2 tha s; IX.20 l nh l ca Euclid v s v hn ca s nguyn t; IX.36 l nh l ca Euclid v s hon chnh v XII.10 a ra cng thc tnh th tch hnh nn. Chng ta hy nh li cc kin thc v hnh hc: mt a gic u n cnh l a gic c tt c n cnh bng nhau v n gc bng nhau. Hnh B1 cho thy mt a gic u 3 cnh, 4 cnh, 5 cnh v 6 cnh, d nhin thng c gi l tam gic u, hnh vung, ng gic u v lc gic u. Cun IV trong b Element a ra cch dng a gic u 3, 4, 5, 6 v 15 cnh ch vi thc v compa. Cch dng ny cho bit Pythagoras sng trc Euclid rt nhiu nm v Plato cng cc hc tr gi thc v compa l dng c ca Euclid. Theo cch chia i gc, c th d dng dng a gic u 2n t a gic u n cnh. Trc ngi Hy Lp c th dng a gic u n cnh m n l cc gi tr sau y: 3, 6, 12, 24, . . . . . .

2.2. i cng v lch s mn gii tch ton hc thi Hy Lp v Ma m c i

35

4, 8, 16, 24, . . . . . . 5, 10, 20, 40, . . . . . . 15, 30, 60, 120, . . . . . . D nhin bc tip theo Euclid tm cch dng cc a gic u 7, 9, 11, 13, . . . cnh. Sau nhiu c gng khng thnh cng, vn ny nh li cho dn 2100 nm sau - 30/3/1976 mi c gii quyt. Vo mt ngy ng nh nht c ghi li trong lch s, mt ngi c tr tui - Carl Friedrich Gauss chng minh rng a gic u 17 cnh c th dng c. Khi Gass mi 18 tui, khm ph y lm ng vui sng n ni lm ng quyt nh i theo con ng ton hc thay cho mn trit hc m ng la chn. ng tip tc nhng pht minh ca mnh v nhanh chng gii quyt hon ton vn dng hnh y. ng chng minh bng phng php c phn kh hiu ca i s v l thuyt s rng mt a gic u n cnh l dng c khi v ch khi n l tch ca mt lu tha ca 2 (trong 20 = 1) v mt s nguyn t nht nh c dng pk = 22 + 1. c bit khi k = 0, 1, 2, 3 th mi s tng ng
k

pk = 3, 5, 17, 257 l s nguyn t, v vy a gic u vi s cnh nh trn. S nguyn t 7 l dng c. S nguyn t 7 thuc dng ny nn a gic u 7 cnh l dng c. Cun XIII trong b Element dnh trn cho vic dng a din u nh mi ngi nhm tng. Mt khi a din l khi b mt gm mt s cc mt a gic, n c gi l u nu cc mt ca n l cc a gic u bng nhau v nu cc gc khi nh bng nhau. R rng c v hn cc a gic u, nhng li ch c 5 a din u. Chng c t tn theo s mt ca chng: t din u (4 mt tam gic), hnh lp phng (6 mt vung), khi 8 mt u (8 mt tam gic), khi 12 mt (12 mt ng gic) v khi 20 mt (20 mt tam gic). D dng chng minh c rng ch c 5 hnh a din u ni trn.

36

Chng 2. i cng v lch s mn gii tch ton hc

Gi s m l s cnh ca mi mt a gic u v n l s a gic cng cha 1 3600 . Mt khc nh. S o (bng ) ca mi gc trong mi mt l 1800 m tng cc gc mi nh ca a din nh hn 3600 , do : n 1800 hn 2. 3600 m < 3600 hay n 1 2 < 2, m

d dng suy ra bt ng thc (m 2)(n 2) < 4, trong m v n u ln Nu m = 3, n c th l 3, 4 hoc 5; nu m = 4, n ch c th l 3 v nu

m = 5, n ch c th l 3, l tt c 5 trng hp c th. i vi ton hc, cc nh l trong b sch ca Euclid va quan trng va rt th v. Trong hn 2000 nm cng trnh kin trc ca tr tu Element c so snh vi cng trnh Parthenon nh l mt biu tng ca thi Hy Lp hong kim. n nay, c hai b h hng cht t nhng c l chng c gi gn v tn to nhiu hn nhng g chng b mt.

2.2.3

Archimedes (287 - 212 trc Cng nguyn)


Tr tng tng ca Archimedes cn ln hn Homer nhiu. Voltaire Archimedes s lun c nh n cn Aeschylus th s ri vo qun lng bi v ngn

ng c th khng tn ti nhng cc tng ton hc th s cn sng mi vi thi gian. G.H.Hardy

Archimedes l mt nh ton hc, nh vt l hc v l nh pht minh ln nht ca th gii c i, mt tr tu siu phm ca nn vn minh phng Ty. Khng ai c th so snh vi ti nng v s sng to ca ng tr Newton ca th k 17. Archimedes sinh ra ti thnh ph Syracuse ca Hy Lp nm trn o Sicyly. ng c quan h thn thuc vi gia nh ca Hong gia v c l cn c quan

2.2. i cng v lch s mn gii tch ton hc thi Hy Lp v Ma m c i

37

h h hng vi vua Hieron II. Thi tr ng hc ti trung tm gio dc ln Alexandria. Sut thi gian ny ng chi thn vi Eratosthenes m sau ny l Gim c th vin Alexandria, ngi truyn t li cc pht minh ca ng. Khi tr li thnh ph qu hng, ng nh c lun v dnh ton b qung i cn li nghin cu ton hc. ng b lnh Roman git tui 75 khi Syracuse b qun La M tn cng trong i chin th gii th hai. Archimedes l ngi lng danh khp th gii Hy Lp trong sut cuc i v tr thnh hnh tng i vo truyn thuyt, khng ch bi nhng pht minh ton hc ca ng m cn bi nhng thnh tu chi li v ng ghi nh ca ng, bi nhng pht kin mu tr v c bi cch hy sinh ca ng, iu c ghi chp li bi nhng tc gi ngi Roman, Hy Lp, Byzantine v ngi Rp qua nhiu th k. ng xc nh th hng ca mnh trn th gii v th gii khng bao gi qun ng. C l cu chuyn truyn thuyt ni ting nht l khi vua Hireon yu cu ng cc nh xem chic vng min mi lm bng vng nguyn cht hay ngi th kim hon thay bt bng bc la ng. Archimedes rt bi ri cho n ngy hm sau ng bc vo mt phng tm cng cng v ch n s trn ra ca nc. Bt ng ng nhn ra rng vng nng bng nc v vng cng chim ch trong nc t hn. ng vui sng vi pht hin ny n ni qun l mnh ang trn trung. ng chy ra ng khng mt tc vi trn ngi, ming la ln: Eureka! Eureka! ngha l Ti tm ra ri! Ti tm ra ri!. Ngay lp tc ng xc nh c rng chic vng min ca vua chim ch nhiu hn trong nc so vi s vng cng khi lng v th ngi th kim hon gian ln b kt n. Cu chuyn ny thng c gn lin vi khm ph ca ng v nguyn l thu tnh: mt vt ni chim ch bng khi lng ca cht lng. T s khi u ny, ng chng minh nhiu nh l v s cn bng v tr ca vt ni c hnh dng khc nhau. Hn na, mt trong nhng pht minh ni ting nht

38

Chng 2. i cng v lch s mn gii tch ton hc

ca ng l bm nc hnh xon c gi l inh c Archimedes. Dng c ny hin nay vn c ngi dn dc sng Nile s dng nng nc ln ti cho ng rung. Trong C hc, ng pht minh ra nguyn l n by, xy dng khi nim trng tm v tm ra trng tm ca mt v khi. Theo mt nh ghi chp, vic nghin cu v n by khin ng tht ra cu ni ni ting: Hy cho ti mt im ta, ti s nng bng c Tri t. S chng minh y khng c xc thc1 . Tuy nhin, nhng sai lm c xem xt tng i t v d dng sa cha. Tt c ngh ca con ngi tip tc tin rng h Euclid trong hnh hc l ng, theo ngha n miu t trc tip hnh hc ca th gii thc m chng ta sng, v cn thit, theo ngha l n c th c dn ra bi cc lp lun khng th b bc b t cc tin nhng iu t chng c xem l hin nhin v lun lun ng. Tnh hung may mn ca cc vn trong hnh hc a n vic hy vng rng theo cch tng t cc chn l xa vi nht ca khoa hc v x hi c th c khm ph v c chng minh n gin bng cch ch ra rng chng l hin nhin v sau lp lun t cc c s . Khng cn xut hin cc tng kh qun hay hp dn trong lch s vn minh ca th gii phng Ty. Uy th ca hnh hc rt v i, c bit trong cc th k 17 v 18, m cc kin thc trong cc lnh vc hu ht u cn n hnh hc Euclid nh mt xc nhn v tnh hp php. Rt nhiu lnh vc ln xn ca tri thc, cc hnh mu b lng trnh, c t coi trng bi mt l do no , mt hoc hai giai on, cc k cng qu tc khng c ch . Do o c hc ca Spinoza, cc mn hc l thn thnh, l nhng ni am m ca con ngi trong nhn loi, gm c cc khi nim, cc tin
1 Nh li nh ngha ca mt im trch dn y. Ngoi ra: Mt ng nm ngang c di : Mt on thng l mt ng m ni cc im ca n; Mt n v c tng tng vi mt ci g c gi l mt; Mt s l tch hp ca mt n v. Nhc im trong s chng minh thng bao gm vic s dng cc gi nh thm m khng c cng nhn r rng.

2.2. i cng v lch s mn gii tch ton hc thi Hy Lp v Ma m c i

39

v cc mnh ca hnh hc Euclid chim mt v tr u tin trong tm tr ca chng ta, v cc mnh m ng ta c gng ng h bng cc chng minh theo phong cch Euclid2 . Trit hc Kant dy rng cc mnh ca hnh hc Euclid chim mt v tr u tin trong tm tr chng ta v do l phng thc cn thit quan st khng gian; v ng xy dng ton b h thng trit hc trn nguyn tc . Principia ca Newton, vi cc ni dung theo li kinh nghim ca chng tp trung vo cc quy lut chuyn ng v thin vn hc trong h thng mt tri, b chi phi hon ton bi s sp xp theo h thng ca Euclid v cc nh ngha, tin , b , mnh , h qu v chng minh, vi mt cht t do vi Q.E.D.s. Hc thuyt th k 17 v t nhin c cng b bi Locke l mt s c gng dn n cc quy lut ca chnh tr v chnh quyn t cc tin ca mt kiu Euclid3 . Thm tr bn tuyn ngn c lp ca nc M, c ni Chng ta c c nhng iu tht s l hin nhin, tip tc tm thy s r rng v ng tin bi cc kiu ca Euclid. Tht khng may mn, s tht hin nhin by gi cng ngy cng khan him hn chng tat ta s dng. T l thuyt ca thuyt tng i v v tr hc, thy rng Hnh hc Euclid khng thch hp vi khun kh ton hc i vi v tr rng ln, v theo ngha khng cn ng na do l thuyt ca hnh hc phi Euclid, cho thy rng cc tin ca hnh hc Euclid khng cn hin nhin lun ng na; Ngc li, chng c th c thay bng nhng iu khc tri ngc vi chng v c chng t r rng chp nhn t cc lp lun logic. Cc tin trong nh nc v cc hot ng ca con ngi by gi c tha nhn hi vng v din t s thch hn l s thc khng th thay i c.
Ti s xem xt cc hot ng ca con ngi v thc s mong mun nu ti hc c ng thng, mt phng v cc vt th - Ethics, phn II, M u. 3 hiu v sc mnh tht s ca chnh tr, v phn tch n t bn cht, chng ta cn phi xem xt cc tnh hung ca con ngi mt cch t nhin, v ngha l trng thi ngu nhin hon ho sp xp cc hnh ng ca h, v sp t ti sn ca ci v con ngi ng nh h ngh - Second Treatise of Government, Part 2.
2

40

Chng 2. i cng v lch s mn gii tch ton hc

Bt chp rt nhiu cc o tng , phng php tin u tin ca Euclid vn cn c s dng rng ri trong nhiu phn l thuyt ca ton hc cao cp nh mt iu hin nhin cho phc ha r rng h thng ton hc tm ra chn l. Khng phi l qu ng khi ni rng l thuyt ton hc tru tng c th kh tn ti nu khng c phng php . Ni chung, i vi hn 2000 nm kiu kin trc tr tu ca C s cnh tranh vi Parthenon nh mt du hiu ca thin ti Hy Lp. C hai phn no gim gi tr trong cc th k gn y, song c th quyn sch cn mang li gi tr xy dng hn l thit hi. Trong C hc, ng pht minh ra nguyn l n by, xy dng khi nim trng tm v tm ra trng tm ca mt v khi. Theo mt nh ghi chp, vic nghin cu v n by khin ng tht ra cu ni ni ting: Hy cho ti mt im ta, ti s nng bng c Tri t 4 . Mt ngy Archimedes qu quyt vi vua Hieron, ngi bn v ngi b con ca ng, rng vi mt lc cho trc ng c th nhc bng bt k vt c trng lng no v hn na ng ta cn chng t rng nu mt Tri t khc, ng c th i khp quanh n v nhc bng Tri t. Khi Hieron kinh ngc n tt , ng vi ng khi ng a ra s chng minh i vi mt vi vt nng c th di chuyn khi tc ng bi mt lc rt nh. Archimedes bt mt trong s thuyn ca vua c ko ln mt b bin bi rt nhiu ngi n ng v nhn cng tuyt vi; v biu din iu vi nhiu ngi khch v rt nhiu hng ho, ng t t mt khong cch, v khng cn qu c gng, ch di chuyn my mc bng cnh tay n ca ng, n gm cc dy cp v rng rc khc nhau, ng ko chic tu mt cch nh nhng v an ton nh l n c di chuyn trn nc vy. Do Hieron rt kinh ngac vi iu thn k m ng lm, T nay v
4

Xem chuyn lun On the Equilibtium of Planes, Works, tr. 198 - 220

2.2. i cng v lch s mn gii tch ton hc thi Hy Lp v Ma m c i

41

sau mi iu ng ni u phi c tin tng. Danh ting lng ly ca Archimedes c ngi i xa ghi li trong nhng cu chuyn v my mc chin tranh m ng sng ch bo v Syracuse chng li qun i c hi qun La M. Plutarch dnh trn nhng trang chi li ca mnh miu t s tn cng ca qun La M v hiu qu ca cc my mc bo v ca Archimedes. l cc my bn c th iu chnh c tm bn ca cc tng khng l bi cong ca gy, c th di chuyn c chnh xc nh ra trn khp cc tng thnh v bn cc vt nng vo qun th mt cch chnh xc nht, v cc cn trc v hm mc phi thng gi tu. Nng chng ln v dm chng xung y bin. Thm tr cng c nhng tm gng t chy i qun trn tu t mt khong cch xa5 . Nh Plutarch vit: Qun La M, trong tnh trng kit sc v cng khi khng th nhn thy qun th, h bt u ngh rng ang chng li cha tri. Marcellus tht ra v tnh, v ci vi sn phm thit k ca ng ni rng: Chng ti phi t b trn chin vi hnh hc Briareus [mt trm v kh khng l hoang tng], ngi ngi trn b bin v lm nhim v ch nh mt tr chi, th v nm ti thuyn ca chng, v tn cng ti thi im cn thit vi v s mi tn, thm ch vt hn hn hng trm cnh tay khng l trong thn thoi. Cui cng qun La M cm thy kinh hi, nu h ch thy cc dy cp hay cy gy trn khp thnh tr, h go tht rng Archimedes san bng i qun ca h bng c my c ng c no , m quay lng li v b chy. Do d Marcellus t b nh ca ng ta tn cng thnh ph v ng
i vi khun kh ca mt th nghim c bi lc lng hi qun Hy Lp nhn thy rng vic s dng nng lng mt tri trong chin tranh c th thc s kh thi, xem Newsweek, 26 thng 11.1973, tr.64. Danh ting lng ly ca Archimedes c ngi i xa ghi li trong nhng cu chuyn v my mc chin tranh m ng sng ch bo v Syracuse chng li qun i v hi qun La M, Plutarch dnh trn nhng trang chi li ca mnh miu t s tn cng ca qun La M v hiu qu ca cc my mc bo v ca Archimedes.
5

42

Chng 2. i cng v lch s mn gii tch ton hc

hy vng vo s vy hm. S vy hm ko di sau 3 nm v kt thc nm 212 trc Cng nguyn vi s sp ca thnh ph. Tt c n khi Archimedes cht, s cng hin c cuc i ca ng, mit mi trong cc nh ton hc. Trong mt s hn n v tn st di s sp ca thnh ph, ng vn tp trung vo cc biu m ng v ra trn ct, v b git cht bi mt tn lnh cp ph khng bit ng l ai. Trong mt on ca cu chuyn ng ni vi k xm lc, khi h n qu gn Khng c quy ry nhng ng trn ca ti, sau s ni gin tn lnh rt kim v m vo ngi ng. Marcellus rt bun bi iu , bi v ng a ra mt lnh rt nghim khc cho qun lnh phng nh v ngi thn ca Archimedes, v ng c ho tng xng ng nh trit hc thng thi N.N Whotehead tm thy ngha ln trong s hin din ny hn c ci cht ca mt ngi n c. Ci cht ca Archimedes di bn tay ca mt lnh La M l du hiu ca mt thay i ln lao ca nhn loi. Ngi La M l mt chng tc ln nhng h ang c nguy c tuyt chng. H khng c sc tng tng dn ti mt ci nhn nhn mi. Quan im mi, nhng ci nhn c th dn ti s chinh phc c thin nhin. Khng ngi La M no git cht c ng, v ng ang say sa m chm vi nhng biu Ton hc. Archimedes ni vi cc bn ca ng rng hy t ln bia m ca ng mt hnh biu din mt hnh tr ngoi tip mt hnh cu, v nh ti thnh 3 tu Ton hc ln nht ca ng hy khc ln bia m phn s cha trong 2 vt th ba chiu. Vic ny c thc hin theo lnh ca Marcellus. Nh hng bin ngi La M Cicero, khi ng ta cn lm quan coi ngn kh ti Sieily vo nm 75 trc Cng nguyn, tm thy k vt ny trong mt bi cy mm xi, ng lau sch v t nguyn li v tr c vi lng knh trng i vi nh

2.2. i cng v lch s mn gii tch ton hc thi Hy Lp v Ma m c i

43

Ton hc v i nht6 . Cicero cng xem v m t mt pht minh ca Archimedes, pht minh to ra mt du n su sc trong th gii c i iu c nhiu tc gi quan tm. Hnh v ny thc s l mt m hnh v tr thu nh, mt hnh cu m bng ng v knh t quay bi dng nc trong mi mt vng quay mt tri, mt trng v 5 hnh tinh chuyn ng trn cng mt qu o tng i i vi v tr ca cc ngi sao c nh nh chng chuyn ng trn bu tri mt ngy, v trong cng c th quan st c chu k nguyt thc. Qu cu ng tng trng cho Tri t quay u v m phng chuyn ng hng ngy ca 5 ngi sao c nh bit, nhng khng phi v th m Archimedes c th m t c c ch s c lp v nhng chuyn ng khc ca mt tri, mt trng v cc hnh tinh khc cng vi s xoay vng ca 5 ngi sao c nh, thi m ng ang sng l mt thi k vi nhng kh nng khng tng. Cicero vit: Khi Gallus t tri t trong trng thi chuyn ng s thy mt tri mc trn ng chn tri ca Tri t sau mt tri xut hin trn bu tri hng ngy; v khi chng ta cm nhn c mt tri ln ra sao v mt trng lan to khp bng ca Tri t nh th no vi Mt tri na kia ca Tri t7 . C ch chy bng sc nc c nhng ngi La M nm gi nh mt phn ca chin li phm ca Syracuse, v n c trn trng trong hng trm nm sau nh l mt iu k diu ca nhn loi. Archimedes tr thnh mt nh pht minh ti ba v tinh t, nhng Plutarch cho rng: nhng pht minh ca ng ch nh mt tr chi hnh hc. Trong mt on vn ni ting ng c ng ni vi chng ta v quan im ca Archimedes i ci cuc sng ni chung v i vi nhng pht minh ca ng
Xem Ciceros Tusculan Disputation Loeb Classical Library, p.491. Ngi La M khng quan tm ti ton hc v do hnh ng sa sang bia m Acsimet ca Cicero c l l mt ng gp ng ghi nhn ca ngi La M trong lch s Ton hc. 7 Xem Cicero s De Re Pubblica, Leob Classical Library, p.43.
6

44

Chng 2. i cng v lch s mn gii tch ton hc

ni ring: Archimedes s hu mt tr tu tuyt vi do c mt tm hn cao thng v c nhng hiu bit su sc v khoa hc, mc d nhng pht minh ca ng a ng tr thnh con ngi phi thng nhng ng cng khng quan tm ti vic ghi li nhng bi vit, nhng cng vic lin quan ti nhng vn , m coi cng vic nh mt hnh ng khng tt p v ch l s trc li v c tnh cht li ch c nhn. ng cng hin ton b tm tr sc lc v c gng khng mt mi nghin cu tm ra nhng nt p tinh t nhng khng c lin quan ti nhng iu cn thit cho cuc sng. Mc d c ti hng bin nhng s tht m Plutarch nu ra y rt ng nghi ng, v c bit rng Archimedes vit mt lun n m by gi tht lc (V s hnh thnh Tri t - On Sphere - making) trong c l c lin quan ti nhng chi tit k thut c i hi i vi vic xy dng m hnh thin vn hc ca ng. Plutarch hon ton b nhim bnh v coi nh Archimedes khng tn ti thng qua nhng l thuyt sung coi thng i vi cng c v nhng o lng khoa hc l mt trong nhng di sn tha k ngu xun ca trit hc Plato li. Tuy nhin, mt iu r rng l trong ton hc thun tu Archimedes c th hi lng v nhng m c su sc ca mnh. Plutarch tr nn thuyt phc hn khi ng ta ni vi chng ta rng mt s t ngi tng sng trong lo u vi Ton hc nh chnh bn thn ng. V th chng ta khng chp nhn nhng th khng th tin c, nhng th l tm thng theo cch ni ca ng ta, v chng tr nn bt dit bi gia nh ca ng ta Siren, tc l hnh hc ca ng ta th vi vic n ung v khng t chm sc con ngi ca ng; rng ng thng quan tm ti lc trong bn tm v khi ng ta c th vch ra nhng hnh v bng nhng

2.2. i cng v lch s mn gii tch ton hc thi Hy Lp v Ma m c i

45

ming than ci, v vi ngn tay ca mnh ng v nhng ng thng trn ngi ca ng cng vi hng v ca tinh du, tr nn mt trng thi khoa hc tuyt diu khoa hc ca ng. Nhng thnh tu ca ng trong Ton hc liu c hon ton ng? Hu ht nhng bt tch tuyt vi ca ng vn ang hng hc sng, mt cch cng bng th l nhng cng trnh ca thin ti. Hu ht nhng vn ca ng a ra trong lun n l hon ton nguyn bn v cha ng hon ton nhng khm ph mi ca ng. Mc d ng nghin cu trn phm vi rng bao gm hnh hc v mt phng lp th, s hc, thin vn, thy tnh hc v c hc nhng ng khng bin son thnh sch nhng khm ph trc nh Euclid. Mc ch ca ng l lun trang b cho mnh nhng kin thc mi. Bng cng vic ca mnh, ng li nhng n tng kh qun, Heath ni: Bn lun n, khng phi ngoi l, l mt cng trnh Ton; s khm ph tng bc mt trong k hoch tn cng, s sp xp bc thy nhng mnh , s la chn nghim khc v hp l cho mc ch chnh, mt kt thc trn vn gy mt n tng su sc i vi s hon ho ca n, bn cnh n cng lm cho ngi c cm thy va gn gi nhng cng rt i mnh mng. Nh Plutarch ni (vi mt s cng iu) Ngay lc u khng th tm trong hnh hc nhng cu hi hoc nhng chng minh kh hn trong nhng mnh r rng v n gin hn. Chnh ti thi im mt con ng huyn diu m ra v a ng t ti kt qu ca mnh. R rng, chng khng th c khm ph bi nhng bc i m li dn chng ti mt ng ct nguy him8 . Do vy, mt trong nhng quan im vit ca ng ta a ra mt cu trc mc mc d hiu nhng hon ho. Mt khc, trong hu ht cc lun n Ton hc ( mc d khng lin quan ti vt l) u c nhng li ta trong ghi nhn nhng ngi bn, gii thch mc ch ca mnh v tp hp tng qut
8

T.L. Heath, Lch s Ton hc Hy Lp, Oxford University Press, 1921, Vol. II, p.20.

46

Chng 2. i cng v lch s mn gii tch ton hc

nhng phm vi m cun sch c lin quan. So snh vi Euclid, nhng bi vit ca ng mang hi th thi i. Phm vi nh hng v tm quan trng ca cc cng trnh Ton hc ca Archimedes c l c bit n nhiu nht t bi tng thut (hoc) bn vn tt trong lun n ca ng: ba lun n lin quan ti hnh hc phng, hai lun n v hnh hc lp th v mt lun n v phng php nghin cu ca ng. 1. Php cu phng mt parabol: lun n ny gm 24 mnh , 2 nh l v chng minh v vic chia nh parabol: min b ct t mt parabol bi bt k ng thng nm ngang bng 4/3 min tam gic vi y v chiu cao bng nhau. nh l c a trong mc 6.2 ca cun sch ny vi chng minh chi tit mc A.2. Hai nh ngha cui ca Archimedes v tng v hn dng chui 1+ 1 1 + 4 4
2

chng t rng ng l ngi nhn thc rt su sc v khi nim gii hn. Vn ny cho n tn th k 19 mi c cc nh ton hc khc nhn thc r rng. 2. ng xon c: ch ca 28 mnh trong lun n ny l ng cong m by gi c bit nh l inh c Archimedes. ng nh ngha nh sau: nu mt ng thng vi mt v tr c nh c quay trn theo mt vn tc u trong mt mt phng n khi n tr li v tr ban u v nu cng thi im y, mt im chuyn ng theo vn tc u dc theo dng thng, bt u t mt v tr c nh th im y s v ln mt ng xon c trong mt phng. Thnh tu chnh ca ng l xc nh tip tuyn ti im bt k v tm din tch min ng bi php quay u tin (mnh 24) sau l din tch hnh trn bn knh bng khong cch t 1 im di chuyn dc theo mt ng thng chuyn ng. ng xon c v nhng nh ngha ny c trnh by

2.2. i cng v lch s mn gii tch ton hc thi Hy Lp v Ma m c i

47

mc 16.3 (bi tp 6), 16.4 (bi tp 7), 16.5 (bi tp 3). Nh nhng nh ton hc sau ny dng ng xon c nh mt ng cong b tr cho vic chia ba mt gc (bi tp 16.3, bi tp 23) v din tch hnh trn (Chng 16, vn 8). 3. Php o ng trn: Phn ny gm ba mnh c ng chng minh rt cht ch m trc ng khng ai lm c v nh ta bit trong mc 6.2; Din tch hnh trn bng din tch mt tam gic c y bng chu vi ng trn 1 v chiu cao bng bn knh ca n, A = cr vi c = 2r v xc nh s 2 1 10 2 ta da vo cng thc A = r . ng thit lp bt ng thc 3 < < 3 , 71 7 bng s tnh ton k lng chu vi ca a gic u 96 cnh ni tip v ngoi tip ng trn. 4. Hnh cu v hnh tr: y l lun n c nh hng su sc nht v n gm nhng chng minh cht ch trong cc pht minh ln ca ng v th tch v din tch hnh cu ( mnh 33 v 34). bit ng pht minh nhng vn ny nh th no ta hy xem mc 6. 5. Hnh nm v hnh ta cu: Phn ny vit v khi trn xoay to nn khi quay parabol, hyperbol v ellips quanh bn trc ca chng. ng tnh th tch mt phn ca cc khi ny v t chng minh cc cng thc: n(n + 1) 2 n(n + 1)(2n + 1) 12 + 22 + + n2 = 6 cho tng ca n s t nhin v bnh phng ca chng (xem mnh 162 v 1+2++n = 105 - 109). ng cng chng minh cng thc: Din tch ca mt ellips vi cc bn trc a v b bng ab. 6. Phng php: iu th v nht trong tt c cc bn lun n l trong mt bc th gi Eratosthenes, Archimedes trnh by phng php nghin

48

Chng 2. i cng v lch s mn gii tch ton hc

cu ca ng trong cc pht minh hnh hc v minh ho cc tng vi 15 mnh . Vic ny tnh c c pht hin trong mt bn vit trn giy da cu Constraninople nm 1906 sau gn mt ngn nm tht lc. Cun Phng php th v nht bi nhng l do sau y. Cc nh hnh hc ln ca Hy Lp ch ni v vic c trng hoc s gian kh trong cng vic nghin cu ca mnh ch khng ch dn cc bc trong cch pht minh ra cc nh l ln ca h. H li cho chng ta cc nh l c hon thnh trong cc tc phm ln nhng khng mt li mch bo v phng php m h dng chng minh. V vy chng ta khng th chc rng ngi Hy Lp c cc phng php km hn so vi phng php ca gii tch hin i hay khng. Mt trong nhng minh ha cho phng php ca Archimedes l ng ch ra cch ng pht minh ra nh l ni ting v th tch hnh cu. Chi tit ton b kin ca ng c trnh by trong mc A.5, cch ngh ca ng dn dn n s khm ph ra cng thc tnh din tch mt cu vi quan nim mt cu l mt nn c qun quanh nh ca n. T nh l ny ch ra rng hnh cu bng 4 ln hnh nn trong ng trn ln ca hnh cu l y ca hnh nn v chiu cao bng bn knh hnh cu din tch mt cu bng 4 ln ng trn ln ca n; din tch hnh trn bng din tch tam gic c y bng chu vi ng trn v chiu cao bng bn knh ng trn. Cng nh vy, hnh cu bng hnh nn c y bng din tch mt cu v chiu cao bng bn knh. Trong lun n cn c hai pht minh thng c s dng trong tnh ton hin nay: xc nh trng tm ca khi bn cu (mnh 6) v th tch phn chung ca hai hnh tr bng nhau v c cc trc vung gc vi nhau (mnh 15). Ngoi ra cn c 6 mnh v hnh hc v 2 mnh vt l trong mt mnh c cp . iu ny c lin quan n s hc v thin vn hc v

2.2. i cng v lch s mn gii tch ton hc thi Hy Lp v Ma m c i

49

c goi l ng h ct trong ng xy dng mt h thng thit k cc so rt ln nh N N trong N = 108 . ng p dng pht minh tm ra gii hn di ca cc s ht ct dy vo mt hnh cu m bn knh bng khong cch t mt tri ti ni m Archimedes gi l hnh cu cha y cc v sao vi khong cch l 1063 v iu ny c lin quan n thin vn hc. Trong tt c nhng thnh cng ca con ngi v ton hc v vt l trn mi luc a v trong mi nn vn minh t tha s khai cho n th k 17 phng Ty, thnh tu ca Archimedes c coi l ln nht. ng chnh l mt nn vn minh v i nht.

2.2.4

Papus (th k th 4 sau Cng nguyn)

Papus l ging vin ca trng Alexandria, l mt nh ton hc thng thi, y ti nng v c nhiu tng hay. Tuy vy, ng khng may mn sinh ra khi nn ton hc Hy Lp b xo trn trong 900 nm k t khi Thalet v Pythagoras cht. Tc phm chnh ca ng - cun Tuyn tp ton hc l s kt hp gia mt bch khoa th, bi bnh lun v l sch hng dn cho mn hnh hc Hy Lp c cho n thi ca ng, ng thi cng m rng v lm phong ph hn cho cc kt qu ca nhng ngi i trc bng cc nh l v chng minh mi. Khng may cho ng, cun Tuyn tp ton hc khi xut bn khng p ng c nn ton hc Ly Lp ang cn mt hi th ca cuc sng mi. Sau Papus ton hc Hy Lp gn nh bin mt v phi i n 1300 nm sau mi c hi sinh vo th k 17. Papus ni ting nht vi nh l v hnh hc lin h gia trng tm khi v mt trn xoay. Trc tin nh l khng nh th tch to ra bi s quay ca mt min bao bi mt bin phng, kn nm hon ton trn mt ng thng

50

Tri h Hng Vng ln th 6-2010

l trc quay bng tch ca din tch min v lhong cch gia trng tm v ng bin ca n. Papus rt t ho v tnh ph bin ca nh l ny, ng ni: Chng bao gm mt s bt l cc nh l v tt c cc loi: bin, mt, khi uc chng t bi mt s chng minh duy nht. ng a ra nhn xt u tin v chng minh v tnh cht tiu im, ng chun, tm sai ca ba ng cnic (mc 15.5) ng l ngi rt k lng trong vic chon ngun ti liu v khng c ngun ti liu no c ni n y, l l do suy ra rng y l nhng khm ph ca chnh ng. ng nu ra s m rng ca nh l Pythagoras sau y (xem hnh B3): cho ABC l tam gic bt k; ACDE v BCF G l hnh bnh hnh c dng pha ngoi trn cc cnh AC v BC; nu DE v F G ct nhau ti H; AJ v BI bng v song song vi HC th din tch hnh bnh hnh ABIJ bng tng din tch cc hnh bnh hnh ACDE v BCF G (chng minh ACDE = ACHR = AIUJ v BCF G = BCHS = BIUT ). D thy rng nh l Pythagoras l trng hp c bit ca bi ton ny, khi gc C l gc vung v cc hnh bnh hnh dng l hnh vung. Cui cng chng ti mun nhn mnh n mt kt qu quan trng ca hnh hc c gi l nh l Papus. Nu cc nh ca mt hnh lc gic ln lt nm trn mt cp ng thng ct nhau (hnh B.4) th 3 giao im ca cc cnh i din ca lc gic thng hng (cnh i din c xc nh t s ca biu lc gic c ch ra trong hnh). K hiu y ca nh l c in ny cui cng mi c tit l vo nm 1899 bi nh ton hc c David Hirbert trong chng trnh lm sng t nhng c s hnh hc ca ng.

Chng 3

Cc chuyn chuyn ton


3.1 Mt s k thut nh gi v c lng khi gii phng trnh i s
Cao Xun Nam, THPT Chuyn H Giang

Tm tt ni dung 1. C th ni trong chng trnh ton ph thng, phng trnh l vn c bn v trng tm, h thng cc phng php gii phng trnh kh phong ph v a dng. nh gi l mt trong nhng phng php . Bi vit nh ny nhm cung cp mt s k nng nh gi khi gii phng trnh. 3.1.1 K nng s dng bt ng thc

V d 3.1. Gii phng trnh x4 + 7x2 + 1 + x4 x2 + 1 = 4x. (3.1)

Gii. Do v tri ca phng trnh dng nn iu kin cn phng trnh c nghim l x > 0. Chia hai v ca phng trnh cho x, ta c x2 + Ta c x2 +
1 x2

1 +7+ x2
1 x2

x2 +

1 1 = 4. x2

(3.2)

+7+

x2 +

1 4, ng thc xy ra khi x=1 (v x > 0)


51

52

Chng 3. Cc chuyn chuyn ton

Do phng trnh (3.2) c nghim khi x = 1. Th li ta thy x = 1 l nghim ca phng trnh cho. V d 3.2. Gii phng trnh Gii. iu kin (3.3) x+ 3x 2 + 6 4x x2 = x2 3x + 5. (3.3)

2 3

x x+

10 2. Ta c 6 4x x2 = x2 2x + 5. (3.4)

3x 2 + x +

Theo bt ng thc Cauchy-Bunhiacpxki, ta c x+ 3x 2 2 2x 1,

6 4x x2 2 3 2x. T x + 3x 2 + x + 6 4x x2 2( 2x 1 + 3 2x). M 2x 1 + 3 2x 2, do vy x+ x+ 3x 2 + x + 6 4x x2 4,

du bng xy ra khi v ch khi x = 1. x = 1. Phng trnh (3.3) c nghim khi V T (3.4) = V P (3.3) = 4 khi v ch khi x = 1. Th li ta thy x = 1 tho mn. Vy phng trnh cho c nghim x = 1. V d 3.3. Gii phng trnh x2 + 9x 13 + x2 + 7x 1 = x2 4x + 6. (3.5) Mt khc x2 2x + 5 = (x 1)2 + 4 4, du = xy ra khi v ch khi

Gii. p dng bt ng thc gia trung bnh cng v trung bnh nhn, ta c x2 + 9x 13 x2 + 9x 13 + 1 , 2

3.1. Mt s k thut nh gi v c lng khi gii phng trnh i s

53

suy ra

2 2(x 2)2 2. Do VT (3.5) 2, ng thc xy ra khi x = 2. x = 2. Th li ta thy x = 2 l nghim ca phng trnh cho. V d 3.4. Gii phng trnh (x 1)4 1 + (x2 3)4 + = 3x2 2x 5 2 3)2 (x (x 1)2 Gii. iu kin: x = 3 v x = 1. Ta vit phng trnh di dng (x 1)4 1 + (x2 3)4 + = (x 1)2 + 2(x2 3). 2 3)2 (x (x 1)2 t u = (x 1)2 ; v = x2 3, phng trnh cho tr thnh 1 u2 + v 4 + = u + 2v. v2 u p dng bt ng thc Cauchy-Bunhicpxki, ta c ( u2 1 + v 4 + )(v 2 + u + 1) (u + v 2 + 1)2 2 v u u2 1 + v 4 + u + v 2 + 1 u + 2v. v2 u = v2 = v=1 u = v = 1. Khi :
u v 1 u

x2 + 9x 13+ x2 + 7x 1 2x2 +8x6. M 2x2 +8x6 =

x2 + 7x 1

3x2 + 7x 1 + 1 , 2

Mt khc VP (3.5) = x2 4x + 6 = (x 2)2 + 2 2, ng thc xy ra khi

(3.6)

(3.7)

T (3.6) v (3.7), suy ra

x = 2. Vy phng trnh cho c nghim duy nht x = 2.

(x 1)2 = 1 x2 3 = 1

54

Chng 3. Cc chuyn chuyn ton

3.1.2

K nng nh gi da vo "gi thit tm"

V d 3.5. Gii phng trnh: x2 + x + 2 2x2 + x + 1 = (x2 1) 3x2 + 2x + 3. (3.8) Gii. t VT (3.8) = x2 + x + 2 2x2 + x + 1; VP (3.8) = (x2 1) 3x2 + 2x + 3. Gi s x2 + x + 2 2x2 + x + 1 x2 1. Khi : VT (??) 0; VP (3.8) 0, do phng trnh (3.8) c nghim Nu x2 + x + 2 < 2x2 + x + 1 x2 < 1. Khi VT (3.8) < 0, VP (3.8) > 0, Th li ta thy phng trnh (3.8) c nghim x = 1 v x = 1. V d 3.6. Gii phng trnh: x2 x + 2 x2 + x = x2 1. 2 +x+2 2 x+4 1 + x 1 + x (1) (x2 x) + 2 (x2 + x 2) + 2

khi VT (3.8) = VP (3.8) = 0, khi v ch khi x = 1 v x = 1. do phng trnh (3.8) v nghim.

(3.9)

Gii. iu kin: 0 x 2. Ta c : 1+ 2 (x2 x) 1+

Xt hm s f (t) =

t+2 . 1+ 2t

2 (x2 + x 2)

= (x 1)(x + 1). (3.10)

D dng chng minh c hm s f (t) l hm s tng. Ta c phng trnh (3.10) tr thnh: f (x2 x)f (x2 +x2) = (x1)(x+1). Gi s x2 x x2 + x 2 x 1. Khi VT (3.10) 0, VP (3.10) 0 (v x + 1 > 0, x [0; 2]) do Nu x2 x < x2 + x 2 x > 1, ta c: VT (3.10) < 0, VP (3.10) > 0,

phng trnh (3.10) c nghim x = 1. phng trnh (3.10) v nghim.

Th li ta thy x = 1 tho mn. Vy phng trnh cho c nghim duy nht x = 1.

3.1. Mt s k thut nh gi v c lng khi gii phng trnh i s

55

3.1.3

K nng nhm nghim kt hp nh gi

V d 3.7. Gii phng trnh: 4x x2 = 3


74 Gii. iu kin: 27 x 10 . 3

4 3 10 3x.

(3.11)

3, do VT (3.11) > VP (3.11). 3, do VT (3.11) < VP (3.11). * Nu 3 < x


10 3

Ta thy x = 3 l mt nghim ca phng trnh (3.11): * Nu 74 x < 3 th 3 4 3 10 3x < 3; (x1)(x3) < 0 4xx2 > 27 th 3 4 3 10 3x > 3; (x1)(x3) > 0 4xx2 <

Vy x = 3 l nghim duy nht ca phng trnh cho. V d 3.8. Gii phng trnh x x x x 5 = 5. (3.12)

(T 4/332 Tp ch ton hc v tui tr) Gii. iu kin: x 5. t x x 5 = t(x t 0 th phng trnh

(3.12) tr thnh: x x t = 5(x t 0).

* Nu t < 5, th x t > x 5 0 x t > x 5 x x t < x x 5 x x t < x x 5 t < 5, v l . * Nu t > 5, th 0 x t < x 5 x t < x 5 x x t > x x 5 x x t > x x 5 t > 5, v l . Vy t = 5, do : x x 5 = 5 x 25 = x 5 x 25 x2 51x + 630 = 0 x 25 (x 25)2 = x 5

x = 30.

Vy x = 30 l nghim duy nht ca phng trnh cho.

56

Tri h Hng Vng ln th 6-2010

3.1.4

Bi tp rn luyn

Gii cc phng trnh sau: 1. x2 + x 1 + x x2 + 1 = x2 x + 2 2. 3. 4. 5. 4 1 x2 + 4 1+x+ 4 1x =3


3 x

25x(2x2 + 9) = 4x + 4

x+3
5 2+7 x+1

1 + 2x = x 2

+ 4x = 3 2 1

Ti liu tham kho 1. Nguyn Vn Mu (1993), Phng php gii phng trnh v bt phng trnh, NXB Gio dc. 2. Nguyn Vn Mu (2002), a thc i s v phn thc hu t, NXB Gio dc.

3.2

p dng nh l Burnside-Frobenius vo bi ton t mu trong t hp


Nguyn Don Ph, Nguyn Th Ngc nh Trng THPT Chuyn Thi Nguyn

Tm tt ni dung 2. Bi vit ca chng ti xin trnh by ba ni dung chnh: *a ra mt s kin thc b tr v nhm nhm chng minh c nh l

3.2. p dng nh l Burnside-Frobenius vo bi ton t mu trong t hp

57

Burnside- Frobenius . *Nu v d minh ha cho ng dng ca nh l Burnside- Frobenius vo mt s bi ton t mu ca t hp. Trong s c v d l bi ton t mu xut hin trong k thi Hc sinh gii ton ton quc nm hc 2009 2010. *Mt s bi tp tham kho.

3.2.1

Mt s kin thc b tr v nhm v nh l Burnside- Frobenius

Nhm v nhm con Nhm l mt tp G cng vi mt php ton k hiu bi du tho mn

cc iu kin:

(ii) e G sao cho e x = x e = x, x G.

(i) a (b c) = (a b) c, a, b, c G.

(iii) Vi mi x G, tn ti x1 G sao cho x x1 = x1 x = e. hiu | G |.

Nu G c hu hn phn t th s phn t ca G c gi l cp ca G, k Tp con H ca mt nhm G c gi l nhm con ca G nu e H v a1 H, a b H vi mi a, b H.

nh l Lagrange Cho H l mt nhm con ca mt nhm G. Vi mi a G, gi {h a |

ghp tri th s cc lp ghp tri ca H gi l ch s ca H trong G, k hiu l (G : H) .

h H} = Ha l mt lp ghp tri ca H trong G. Khi H ch c hu hn lp

nh l Lagrange: Trong mt nhm hu hn, cp v ch s ca mt nhm con l c ca cp ca ton nhm.

58

Tri h Hng Vng ln th 6-2010

Nhm i xng

Gi s X l tp hp c n phn t. Khi nhm i xng ca X l tp hp bao gm cc hon v ca X c k hiu bi S(X) hoc Sn . Mi phn t f ca ton trong nhm Sn l php nhn cc nh x thng thng. Ta cng c th coi f l mt hon v ca tp {1, 2, .., n}. Gi s i l phn t no thuc tp {1, 2, .., n}. Ta k hiu f 1 (i) = f (i), f 2 (i) = f (f 1 (i)), ..., f t (i) = f (f t1 (i)), ... V tp X l hu hn nn tn ti mt s nguyn dng r sao cho f r (i) = i. Khi dy (i, f 1 (i), f 2 (i), ..., f r1 (i)) gi l mt vng xch c di r . vit f thnh tch cc vng xch c lp ta lm theo cc bc sau: Bc 2: Chn mt phn t ca {1, 2, .., n} cha xut hin trong cc vng xch Bc 1: Chn i {1, 2, .., n} (thng l i = 1 ). Tm vng xch (i, f 1 (i), f 2 (i), ..., f r1 (i)). Sn c th ng nht vi mt song nh t tp {1, 2, . . . , n} n chnh n. Php

hon thnh. S dng phn t ny nh phn t i bc 1 v thu c mt vng xch mi. Mun biu din f ta t cc vng xch ca f lin tip cnh nhau. Vng xch c di bng 1 c th khng vit ra. Khi ta ni f c phn tch thnh tch cc vng xch c lp V d: Cc phn t ca nhm i xng S3 c biu din nh sau Bc 3: Lp li bc 2 ti khi tp {1, 2, .., n} c vt kit.

S3 = {I, (1, 2, 3), (1, 3, 2), (1, 2), (1, 3), (2, 3)}.

trong I l nh x ng nht.

3.2. p dng nh l Burnside-Frobenius vo bi ton t mu trong t hp

59

nh l Burnside- Frobenius Cho X l mt tp hp c n phn t v G l mt nhm con ca nhm i xng S(X) . Vi x X, t Gx = {g G : g(x) = x}. Gx = {g(x) : g G}. F (g) = {z X : g(z) = z}. Khi Gx l nhm con ca G. Nhm Gx c gi l nhm con ng hng ca G ng vi phn t x. Tp Gx gi l qu o ca x trong X. B 1.4.1: (i) Gx = vi mi x X. (ii) X = Gx.
xX

(iii) Gx = Gy hoc Gx Gy = vi mi x, y X.

Chng minh: (i), (ii). V x = I(x) Gx nn Gx = . Suy ra X =

Gx.
xX

(iii). Gi s GxGy = . Khi tn ti g1 , g2 G sao cho g1 (x) = g2 (y). Suy ra


1 1 1 x = I(x) = g1 g1 (x) = g1 g2 (y). Cho g(x) Gx. Ta c g(x) = g(g1 g2 (y))

Gy. Do Gx Gy. Tng t Gy Gx, v v th Gx = Gy. hoch trn X. B 1.4.2:

B trn ch ra rng tp cc qu o trong X lm thnh mt php phn

| Gx || Gx |=| G | Hn na, nu Gx1 , . . . , Gxt l cc qu o i mt ri nhau trong X th


t t

| X |=

Gxi =
i=1 i=1

| Gxi |

Chng minh: Gi L l tp cc lp ghp tri ca Gx trong G. Xy dng mt nh x f : L Gx bin mi phn t gGx L thnh g(x) Gx. Ta chng

60

Tri h Hng Vng ln th 6-2010

minh c f l mt song nh. Suy ra | Gx |=| L |. Theo nh l Lagrange b c chng minh. nh l Burnside- Frobenius . Cho X l mt tp hp hu hn. G l mt g(x) X. Khi s qu o ca tc ng l nhm con ca tp i xng S(X). G tc ng ln tp X bi g G, x X th 1 F (g). | G | gG Chng minh: Gi T l tp cc cp sp th t (g, x) sao cho g G, x X v cp ca nhm con ng hng Gx ca x. V th ta c | T |= | Gx | g(x) = x. Vi mi x X, s cc phn t g G sao cho (g, x) T chnh l

xX

Vi mi g G, s phn t x X sao cho (g, x) T chnh l F (g). V th | T |= T hai ng thc trn ta c | Gx | 1 F (g). = |G| | G | gG F (g).
gG

xX

Gi t l s qu o. Gi Gx1 , . . . , Gxt l cc qu o. V cc qu o l i mt ri nhau v X l hp ca cc qu o nn ta c | Gx | | Gx | | Gx | = + ...+ . |G| |G| |G| xGx xGx


1 t

xX

Vi mi i = 1, . . . , t, theo B 1.4.2 tng


xGxi

1 . hng, mi s hng u bng | Gxi | V th


xGxi

| Gx | bao gm | Gxi | s |G|

| Gx | =1 |G|

3.2. p dng nh l Burnside-Frobenius vo bi ton t mu trong t hp

61

vi mi i = 1, . . . , t. Suy ra
xX

| Gx | = t. |G|

3.2.2

p dng vo bi ton t mu trong t hp

Xt mt a gic u n cnh, tm O v cc nh 1, 2, . . . , n sp th t ngc Ta gi nhm nh din D2n l nhm con ca Sn sinh bi hai phn t R v T, 3600 , trong R l php quay tm O ngc chiu kim ng h vi gc quay n v T l php i xng qua ng thng ni tm O vi mt nh (chng hn gic, trong c n php quay I, R, R2 , . . . , Rn1 v n php i xng T, RT, . . ., Rn1 T. Nu n l th mi php i xng c xc nh bi ng thng ni tm O vi mt nh (i qua trung im ca cnh i din vi nh ). Nu n chn th n/2 php i xng c xc nh bi n/2 ng thng, mi ng ni hai nh i din nhau (ng thng ny i qua tm O); v n/2 php i xng c xc nh bi n/2 ng thng, mi ng ni hai trung im ca hai cnh i din nhau. V d 2.1: Xt mt hnh vung tm O vi cc nh l 1, 2, 3, 4 sp th t ngc chiu kim ng h. I l php ng nht, R l php quay tm O vi gc quay 900 v T l php i xng qua ng thng ni hai nh 1 v 3 th nhm D8 gm 8 phn t sau: I; R = (1, 2, 3, 4); R2 = (1, 3)(2, 4); R3 = (1, 4, 3, 2); T = (2, 4); RT = (1, 2)(3, 4); R2 T = (1, 3); R3T = (1, 4)(2, 3) Gi s rng, vi k mu cho sn, chng ta t mu cc nh ca hnh vung trong v d trn (khng yu cu phi dng tt c cc mu). Th th c bao nhiu cch t mu? Gi s chng ta c hai mu, mu trng (T) v mu xanh chiu kim ng h. K hiu Sn l nhm cc php th ca tp nh {1, 2, . . . , n}.

nh 1). Nu I l nh x ng nht th D2n gm ng 2n php ng c ca a

62

Tri h Hng Vng ln th 6-2010

(X). Khi qua php R2 trong cch t mu C1 = bin thnh cch t mu C2 = 1 2 3 4 , T X T X 1 2 3 4 T X T X

Hai cch t mu C1 v C2 nn c xem l nh nhau. Nhm D8 l nhm con ca S(X), trong X l tp cc nh ca hnh vung. D8 tc ng ln X v v th n tc ng ln tp cc cch t mu ca cc C = g C l tc ng ca g ln C th ta c th coi cc cch t mu C v C nh. Vi mi g D8 nu C l mt cch t mu cc nh ca hnh vung v

l nh nhau. Do cc cch t mu trong cng qu o {g C : g D8 } phn bit (khng tng ng) chnh l s cc qu o.

ca C nn c xem l tng ng. Trong trng hp ny, s cch t mu nh l Burnside- Frobenius gip ta tnh c s qu o ca mt tc ng nu ta bit s cch t mu c nh qua tc ng ca mt hon v cho trc. V th ta cn mnh sau y. Mnh 2.2 : Gi G l nhm con ca S(X) vi X l tp cc nh ca mt a gic u n cnh. Cho g G. Gi s g l tch ca c vng xch c lp, tnh c cc xch c di 1. Nu ta c k mu th s cch t mu cc nh ca a gic c nh qua tc ng ca g l k c . Chng minh: Cho C l mt cch t mu cc nh ca a gic. Gi s C c nh

ca g, v g(a1) = a2 , g(a2 ) = a3 , . . . , g(an ) = a1 nn cc nh a1 , . . . , ap phi c cng mu. Ngc li, gi s vi mi vng xch (a1 , . . . , ap ) ca g, cc nh a1 , . . . , ap c cng mu. Khi r rng C l c nh qua tc ng ca g. V th s cch t mu c nh qua tc ng ca g l s cch chn mu cho cc xch ca g, k c cc xch c di 1, mi xch chn mt mu. V vy c ng

qua tc ng ca g (tc l C = g C), khi vi mi vng xch (a1 , . . . , ap )

3.2. p dng nh l Burnside-Frobenius vo bi ton t mu trong t hp

63

k c cch t mu c nh qua tc ng ca g. Nhm D8 gm c: 1 hon v 4 vng xch, l I 2 hon v 3 vng xch, l T v R2 T 3 hon v 2 vng xch, l R2 , RT v R3 T 2 hon v 1 vng xch, l R v R3 . Theo nh l Burnside- Frobenius v mnh 2.2 ta c s cch t mu phn bit cc nh hnh vung ni trn l: 1 4 (k + 2k 3 + 3k 2 + 2k). 8 y cng chnh l s cch t mu phn bit. V d 2.3: Cho s nguyn dng n. Cho bng vung kch thc 3 3. Ngi ta dng n mu t tt c cc vung con ca bng sao cho trong mi cch t, mi vung con c t bi mt mu. Hai cch t c coi l nh nhau nu cch t mu ny c th nhn c t cch t mu kia nh mt php quay quanh tm ca bng vung. Hi c tt c bao nhiu cch t mu i mt khng nh nhau? (Lu : Trong mi cch t khng nht thit phi dng n mu). Gii: nh s cc vung bin theo th t t 1 n 8 theo chiu ngc chiu kim ng h. Theo gi thit chnh gia bng c n cch t mu. Ta xt nhm G cc php quay quanh tm ca bng trn bin bng thnh bng c v tr trng vi n. G bao gm 4 phn t I, R = (1, 3, 5, 7)(2, 4, 6, 8), R2 = (1, 5)(2, 6)(3, 7)(4, 8), R3 = (1, 7, 5, 3)(2, 8, 6, 4). Theo nh l Burnside- Frobenius v mnh 2.2 ta c s cch t mu cc bin tha mn yu cu bi ton l: 1 8 (n + n4 + 2n2 ). 4

64

Tri h Hng Vng ln th 6-2010

Theo quy tc nhn, s cch t mu tha mn yu cu bi ton l: 1 9 (n + n5 + 2n3 ). 4 V d 2.4: Cho trc mt hnh lp phng. Ta nh s bn nh mt trn bi cc s 1, 2, 3, 4. nh s bn nh mt di nh sau: nh 5 di nh 1, nh 6 di nh 2, nh 7 di nh 3, nh 8 di nh 4. Xt nhm G gm 24 php quay bin hnh lp phng thnh chnh n nh sau: Php ng nht I = (1)(2)(3)(4)(5)(6)(7)(8) 1800 , v d (1, 3)(2, 4)(5, 7)(6, 8) 900 , v d (1, 2, 3, 4)(5, 6, 7, 8) Ba php quay quanh ng thng ni tm ca hai mt i din, gc quay Ba php quay quanh ng thng ni tm ca hai mt i din, gc quay Ba php quay quanh ng thng ni tm ca hai mt i din, gc quay

2700 , v d (1, 4, 3, 2)(5, 8, 7, 6)

quay 1800, v d (1, 2)(3, 5)(4, 6)(7, 8) v d (1)(2, 4, 5)(3, 8, 6)(7) v d (1)(2, 5, 4)(3, 6, 8)(7)

Su php quay quanh ng thng ni trung im hai cnh i din, gc Bn php quay quanh ng cho ca hnh lp phng, gc quay 1200 ,

Bn php quay quanh ng cho ca hnh lp phng, gc quay 2400 ,

Vi k mu cho trc, hi c bao nhiu cch t mu phn bit cc nh ca hnh vung bit hai cch t mu c coi l nh nhau nu cch ny c th nhn c t cch t mu kia nh mt php quay thuc G. Gii: Nhm G c mt hon v tm vng xch l I. Mi by hon v bn vng xch. Su hon v hai vng xch. Do kt qu cn tm l: 1 8 (k + 17k 4 + 6k 2 ). 24

3.2. p dng nh l Burnside-Frobenius vo bi ton t mu trong t hp

65

3.2.3

Bi tp tham kho

Bi 1: Ngi ta cn t mu cc nh hnh ca mt hnh vung vi 2 mu trng v xanh. C bao nhiu cch t mu phn bit tha mn yu cu bi ton bit hai cch c coi l nh nhau nu cch t mu ny l tc ng ln cch t mu kia qua php hon v trong nhm D8 Bi 2: Gi s mt ci cy gy c t trn trc honh t x = 1 n x = 1 v ti trung im (0, 0) ca gy. Ngi ta mun t 3 ht bng n mu, v hai cch t mu c coi l nh nhau nu cch t mu ny l tc ng ln ng nht v l php quay quanh trc tung mt gc 1800 . Chng minh rng khi s cch t mu phn bit l 1 2 (n + n3 ). 2 (Ch rng nu gi thit thm rng trong n mu c mu en. Khi s 1 cch t mu phn bit sao cho ht gia gy lun c mu en l (n + n2 )). 2 Bi 3: Vi 2 mu v xanh ta cn t mu cc nh ca mt lc gic u. Gi thit rng 2 cch t mu l tng ng nu cch t mu ny l tc ng ln cch t mu kia qua mt hon v trong nhm D12 . C bao nhiu cch t mu phn bit sao cho 3 nh ca lc gic c mu v 3 nh cn li c mu xanh. Bi 4: Gi G l nhm gm 12 php quay mt khi t din u bao gm: hon v ng nht I v 08 php quay ngc chiu kim ng h quanh trc ni 1 nh vi trng tm ca mt i din vi gc quay 1200 v 2400 , l: (1)(2,3,4), (1,3,4)(2), (1,2,4)(3), (1,2,3)(4), (1)(2,4,3), (1,4,3)(2), (1,4,2)(3), (1,3,2)(4) v cch t mu kia qua php hon v trong nhm {I, }, trong I l php th

vi 3 ht c nh trn gy. Cc ht ny c nh ti im (1, 0) v (1, 0))

66

Tri h Hng Vng ln th 6-2010

03 php quay quanh 03 trc ni cc trung im ca 2 cnh cho nhau vi gc quay 1800. Gi thit rng 2 cch t mu l tng ng nu cch ny l tc ng ln cch kia qua mt hon v no trong nhm G. Chng minh rng, vi k mu cho trc t mu cc nh ca khi t din ni trn, c 1 4 (n + 11n2 ) 12 cch t mu phn bit. Bi 5: Gi s ta c 4 mu, trong c mu xanh t mu cc nh ca khi t din u. Gi G l nhm php th gm 12 phn t xc nh nh trong bi 4 . Gi thit rng 2 cch t mu c xem l nh nhau nu cch t ny l tc ng ca cch kia qua mt php th trong G. C bao nhiu cch t mu phn bit sao cho c ng 2 nh c t mu xanh. Bi 6: Cho p l s nguyn t. Xt mt a gic u p cnh vi tm O. Gi I l php ng nht v R l php quay tm O ngc chiu kim ng h vi gc quay 3600/p. K hiu G = {I, R, R2, . . . , Rp1 }. l nhm cc php quay ca a gic. Gi thit rng hai cch t mu l tng ng nu cch t ny l tc ng ln cch kia qua mt php quay trong G. Chng minh rng vi n mu cho trc, t mu cc nh ca a gic, s 1 cch t mu phn bit l (np + (p 1)n). p Bi 7: Gi thit rng 2 cch t mu cc nh ca mt hnh lc gic u l tng ng nu cch t mu ny l tc ng ca mt hon v trong nhm

Chuyn chn lc v bt ng thc

67

D12 ln cch t mu kia. Chng minh rng vi k mu cho trc, c 1 6 (k + 3k 4 + 4k 3 + 2k 2 + 2k). 12 cch t mu phn bit. nh l Burnside- Frobenius cn c ng dng trong mt s bi ton mc kh hn, v d nh bi ton t mu cc mt ca khi lp phng, khi 12 mt u, khi 20 mt u...Tuy nhin, trong phm vi mt bi ton nh chng ti khng nu c ra y. Rt mong cc bn ng nghip c, cho gp v pht trin tip tng m chng ti nu ra. Chng ti xin chn thnh cm n.

Ti liu tham kho 1. Nguyn Tuyt Nga (2009), Mt s ng dng ca l thuyt nhm vo cc bi ton s cp, Lun vn cao hc, i hc Khoa Hc - i hc Thi Nguyn, Thi Nguyn nm 2009. 2. V.K. Balakrishnan, Ph.D (1995), Theory and problems of combinatorics, McGraw-Hill, INC, Singapore.

3.3

Chuyn chn lc v bt ng thc


T Ton, THPT Chuyn Cao Bng

3.3.1

M u

I- L do chn ti. - Bt ng thc v chng minh bt ng thc l mt phn quan trng ca chng trnh ton ph thng. Trong cc thi tuyn sinh vo cc trng i

68

Tri h Hng Vng ln th 6-2010

hc v cao ng, cc k thi chn hc sinh gii cc cp, bi ton bt ng thc l bi ton khng th thiu. Bt ngt thc cng l mt trong nhng vn kh ca chng trnh ton ph thng, khng ch i vi hc sinh trung bnh m ngay c i vi hc sinh gii. V vy vic dy hc bt ng thc v gii quyt cc bi ton l mt ni dung ca chng trnh v mi gio vin, nht l i vi nhng gio vin n luyn hc sinh gii cn quan tm. l l do chng ti chn ti nghin cu ca mnh l: ng dng ca bt ng thc AM-GM v bt ng thc Cauchy-Bunhiacovski vo chng minh bt ng thc. II- C s l lun. - Trong chng minh bt ng thc, vic s dng cc bt ng thc c chng minh chng minh cc bt ng thc khc l mt trong cc phng php hay c dng. Cc bt ng thc thng c dng l bt ng thc AM-GM v bt ng thc Cauchy-Bunhiacovski. - Vic hng dn hc sinh s dng 2 bt ng thc ny mt cch hp l v hiu qu l rt cn thit v cn c quan tm ng mc 3.3.2 Ni dung

I. Bt ng thc AM-GM Bt ng thc AM-GM ch p dng cho cc s dng hoc bng 0. Khng p dng c cho s m. Vi a1 , a2 , . . . , an l cc s thc khng m ta lun c a1 + a2 + + an n n a1 .a2 . . . an , du ng thc xy ra khi v ch khi a1 = a2 = = an . Khi s dng bt ng thc AM-GM c th phi cng thm nhiu s 1 hoc

tch mt s hng thnh nhiu s hng c c gi tr n (s cc s hng) em li s m mong mun.

3.3. Chuyn chn lc v bt ng thc

69

Khi chng minh bt ng thc c ba s hng nn cn nhc s dng mt ln bt ng thc AM-GM cho ba s hng hoc ba ln, mi ln cho hai s hng. chng minh mt bt ng thc dng phn thc, thng phi p dng hai ln bt ng thc AM-GM: Mt ln cho ton th cc phn thc, mt ln cho ring mt phn thc (hoc cho ring biu thc mu ca mt phn thc). Nhng gi trn y c p dng trong vic gii cc bi ton sau: Bi 1. Cho cc s dng a1 , a2 , . . . , an sao cho a1 a2 a3 . . . an = 1. Chng minh rng (1 + a1 ).(1 + a2 ).(1 + a3 ) . . . (1 + an ) 2n . Bi gii. Theo bt ng thc AM-GM: V 1 + a1 2 a1 , 1 + a2 2 a2 , 1 + an 2 an nn nhn cc bt ng (1 + a1 ).(1 + a2 ) . . . (1 + an ) 2n a1 .a2 . . . an = 2n . C du ng thc xy ra khi v ch khi 1 = a1 = a2 = = = an . Bi 2. Cho x, y, z > 0, Chng minh rng x2 1 1 1 x+y+z + 2 + 2 . + zy y + zx z + xy 2xyz (1)

thc cng chiu, cc v u dng ny li s c

Trng hp no xy ra du ng thc? Bi gii. Theo bt ng thc AM-GM th x2 + yz 2x yz x2 x2 1 1 + zy 2x yz

1 1 1 1 1 1 + 2 + 2 + + + zy y + zx z + xy 2x yz 2y zx 2z xy yz + zx + xy (y + z)/2 + (z + x)/2 + (x + y)/2 x+y+z = . = 2xyz 2xyz 2xyz Du ng thc xy ra khi v ch khi x = y = z.

70

Tri h Hng Vng ln th 6-2010

Bi 3. Cho ba s dng a, b, c tho mn iu kin a2 + b2 + c2 = 1. Chng minh rng b c 3 3 a + + . b2 + c2 c2 + a2 a2 + b2 2 (1)

Bi gii. Ta c b2 c2 3 3 2 a2 + + .(a + b2 + c2 ). (1) a(1 a2 ) b(1 b2 ) c(1 c2) 2 Nhn xt rng (2) s c chng minh nu x2 3 3 2 .x khi 0 < x < 1 x(1 x2 ) 2 2 x(1 x2 ) khi 0 < x < 1. 3 3 Tht vy theo bt ng thc AM-GM th 1 1 2x2 + (1 x2 ) + (1 x2 ) (1 x ) = .(2x2 ).(1 x2 ).(1 x2 ) 2 2 3
2 2 3

(2)

(3)

4 . 27

T y c (3) nn suy ra pcm. C du ng thc a = b = c = c (3).

3 . 3

Cch khc. Kho st hm bc ba f (x) (1 x2 ) vi x (0, 1) cng thu

Bi 4. Cho a, b > 0. Chng minh bt ng thc 1 a3 1 a + 3 + b3 + + b. a3 b a b Bi gii. V (1)

3 a3 3a 1 1 +1+1 3 3 3 .1.1 = , tng t: 3 +1+1 , b3 +1+1 3 a a a b b 3b, cng ba bt ng thc ny s c 1 a3 + 3 + b3 + 6 3 a3 b 1 a + +b a b 1 a 3 1 2 + + b + 2.3 . .b (1). a b a b

C du ng thc khi v ch khi a = b = c = 1. a3 b3 c3 a b c Tng qut. Nu a, b, c > 0 th 3 + 3 + 3 + + . C du ng thc b c a b c a a = b = c.

3.3. Chuyn chn lc v bt ng thc

71

Bi 5. Cho x, y > 0. Chng minh rng x2 + y 2 + 1 1 + 2( x + y . x y (1)

Bi gii. Theo bt ng thc AM-GM th x2 + 1 1 2 x , y 2 + 2 y. x y

Cng li c (1). C du ng thc x = y = 1. Bi 6. Cho x, y, z 0 v x + y + z 3. Chng minh rng y z 3 1 1 1 x + + + + . 2 2 2 1+x 1+y 1+z 2 1+x 1+y 1+z Bi gii. Theo bt ng thc AM-GM th x2 + 1 2x x 1 2 1+x 2

x y z 3 + + , 2 2 2 1+x 1+y 1+z 2 1 du ng thc xy ra x = y = z = . 2 1 1 1 + + 3. 1+x 1+y 1+z 3.


3

1 (1 + x)(1 + y)(1 + z)

1 3 . (1 + x) + (1 + y) + (1 + z) 2 3 C du ng thc x = y = z = 1. Bi 7. Cho a, b, c > 0. Chng minh rng ab bc ca a+b+c + + . a+b b+c c+a 2 Bi gii. V (a + b)2 4ab nn a+b ab . Suy ra a+b 4 (1)

bc ca a+b b+c c+a a+b+c ab + + + + = . a+b b+c c+a 4 4 4 2

72

Tri h Hng Vng ln th 6-2010

C du ng thc a = b = c > 0. Bi 8. Cho a, b, c > 0. Chng minh rng (1 + a3 ).(1 + b3 ).(1 + c3 ) (1 + ab2 ).(1 + bc2 ).(1 + ca2 ). Bi gii. Ta c (1) a3 + b3 + c3 + a3 b3 + b3 c3 + c3 a3 ab2 + bc2 + ca2 + a3 b2 c + b3 c2 a + c3 a2 b . Theo bt ng thc AM-GM th 3 a3 + 2b3 = a3 + b3 + b3 3. a3 .b3 .b3 = 3ab2 (a3 + 2b3 ) + (b3 + 2c3 ) + (c3 + 2a3 ) 3ab2 + 3bc2 + 3ca2 a3 + b3 + c3 ab2 + bc2 + ca2 . Tng t, ta c 3 a3 b3 + 2b3 c3 = a3 b3 + b3 c3 + b3 c3 3 a3 b3 .b3 c3 .c3 a3 = 3ab3 c2 (a3 b3 + 2b3 c3 ) + (b3 c3 + 2c3 a3 ) + (c3 a3 + 2a3 b3 ) 3ab3 c2 + 3bc3 a2 + 3ca3 b2 a3 b3 + b3 c3 + c3 a3 a3 b2 c + b3 c2 a + c3 a2 b. Cng (3) vi (4) c (2) (1). C du ng thc a = b = c. Bi 9. Cho x > 0, y > 0. Chng minh rng 1 1 4 + , , x y x+y du ng thc xy ra khi x = y. Bi gii. Theo bt ng thc AM-GM, ta c 1 2 1 4 . x + y 2 xy 2 xy x+y xy x+y (1) (4) (3) (2) (1)

3.3. Chuyn chn lc v bt ng thc

73

Mt khc, li c 2 1 1 + . x y xy T (1) v (2) suy ra pcm. C du ng thc khi v ch khi 1 1 = x y x=y x = y. (2)

Bi 10. Cho cc s dng x, y, z tho mn iu kin xyz = 1. Chng minh rng: 1 + x3 + y 3 + xy 1 + y3 + z3 + yz 1 + z 3 + x3 3 3. zx (1)

Khi no du ng thc xy ra? Bi gii. Theo bt ng thc AM-GM th 1 + x3 + y 3 3 3 x3 y 3 = 3xy 1 + x3 + y 3 3 xy 1 + x3 + y 3 3 = . xy xy xy Tng t, ta c 1 + y3 + z3 3 yz yz 1 + z 3 + x3 3 . zx zx 3 xy (2)

(3)

(4)

Cng tng v ca cc bt ng thc (2),(3),(4) ta c 1 + x3 + y 3 1 + y3 + z3 1 + z 3 + x3 1 1 1 + + 3 + + xy yz zx xy yz zx (v theo bi: xyz = 1). 3.3 3 1 x2 .y 2 .z 2 =3 3

74

Tri h Hng Vng ln th 6-2010

C du ng thc khi v ch khi x3 = y 3 = z 3 = 1 1 1 1 xy = yz = zx rng

x = y = z = 1. 1 1 1 + + = 4. Chng minh x y z

Bi 11. Cho x, y, z l cc s dng tho mn

1 1 1 + + 1. 2x + y + z x + 2y + z x + y + 2z Bi gii. V (a b)2 0 a2 + b2 2ab 0 a2 + b2 + 2ab 4ab 1 1 + 4 1 1 4 1 a+b + a b. (a + b)2 4ab ab a+b a b a+b a+b 4 C du ng thc khi v ch khi a = b. Suy ra: 1 1 + 1 1 x+y x+z = . 2x + y + z (x + y) + (x + z) 4 Tng t, ta c 1 1 + 1 1 x+y y+z = x + 2y + z (x + y) + (y + z) 4 1 1 + 1 1 x+z y+z = x + y + 2z (x + z) + (y + z) 4 Cng tng v ca bt ng thc (1), (2) ,(3), ta c: 1 1 1 + + 1 1 1 x+y y+z z+x + + 2x + y + z x + 2y + z x + y + 2z 2 1 1 1 1 1 1 1 1 1 + + + + + 4 x y y z x y z z x + + 4 4 4 2 = = 2 =1 2 2 2 (2) (1)

(3)

3.3. Chuyn chn lc v bt ng thc

75

. C du ng thc khi v ch khi : x+y =y+z =z =x x=y=z 3 x=y=z= . 1 1 1 + + =4 4 x y z Chng minh rng vi mi x R ta c 12 5
x

Bi 12.

15 4

20 3

3 x + 4x + 5x .

Khi no du ng thc xy ra? Bi gii. Theo bt ng thc AM-GM ta c 12 5 15 4 20 3


x

+
x

15 4 20 3 12 5
x

2
x

12 5 15 4 20 3
x

.
x

15 4

= 2 32x = 2.3x = 2 52x = 2.5x = 2 42x = 2.4x

(1)

+
x

2
x

20 . 3 . 12 5
x

(2) (3)

Cng tng v ca cc bt ng thc (1), (2), (3), ta c: 2. 12 5 12 5 +


x

15 4 + 15 4

+
x

20 3 + 20 3

2 (3x + 4x + 5x )
x

3 x + 4x + 5x .

C du ng thc khi v ch khi : x 12 = 5 15 x = 4 20 x = 3

15 4 20 3 12 5

x = 0.

76

Tri h Hng Vng ln th 6-2010

Bi 13. Chng minh rng vi mi x, y > 0, ta c y (1 + x) 1 + x ng thc xy ra khi no? Bi gii. Theo bt ng thc AM-GM, ta c (1 + x) = 1 + 1+ x x x + + 3 3 3 4
4

9 1+ y

256.

1.

x3 . 33

(1) (2)

y3 y y y y y3 4 = 1+ 4 4 1. + + . =4 x 3x 3x 3x 27x3 (3x)3 9 1+ y
2

3 3 3 1+ + + y y y
2 3

44

33 1. y

= 16 4

272 . y3

(3)

Nhn tng v ca cc bt ng thc (1) ,(2) , (3) ta c 9 y . 1+ (1 + x) 1 + x y


2

4.4.16

x3 y 3 272 . . = 256. 27 27x3 y 3

Bi 14. Cho x, y, z l ba s tho mn iu kin x + y + z = 0. Chng minh rng Bi gii. Theo bt ng thc AM-GM, ta c 3 + 4x = 1 + 1 + 1 + 4x x 4 1.1.1.4x = 2.4 8 . 4 (1)

C du ng thc khi v ch khi 1= x = x = x 3 y 3y 3 y 1= = = 3x 3x 3x 3 3 3 1= = = y y y

x=3 y = 3x y=9

x=3 y=9

3.3. Chuyn chn lc v bt ng thc

77

3+

4x

1+1+1+

4y

3 + 4z =

1 + 1 + 1 + 4z

y 4 y 4 1.1.1.4 = 2.4 8 . z 4 z 4 1.1.1.4 = 2.4 8 .

(2) (3)

Cng tng v ca cc bt ng thc (1),(2),(3), ta c 3 + 4x + 3 + 4y + x y z 3 + 4z 2 4 8 + 4 8 + 4 8

(v x + y + z = 0). Du ng thc xy ra: x 1 = 1 = 1 = 4y 1=1=1=4 1 = 1 = 1 = 4z x = y = z = 0. y z x 4 8 = 48 = 48

2. 3

x+y+z 0 3 8 4 = 2.3 4 8 = 2.3 = 6

3 Bi 15. Cho a, b, c l cc s dng tho mn a + b + c = . Chng minh rng 4 3 3 a + 3b + b + 3c + 3 c + 3a 3. Bi gii. Theo bt ng thc AM-GM ta c
3

(a + 3b) .1.1 (b + 3c) .1.1

a + 3b + 1 + 1 3

(1) (2) (3)

b + 3c + 1 + 1 3 c + 3a + 1 + 1 (c + 3a) .1.1 3

Cng tng v ca cc bt ng thc (1), (2), (3) ta c: 3 3 3 3 4. + 6 4 (a + b + c) + 6 = 4 = 3. c + 3a 3 3

a + 3b +

b + 3c +

78

Tri h Hng Vng ln th 6-2010

C du ng thc khi v ch khi: a + 3b = 1 = 1 b + 3c = 1 = 1 c + 3a = 1 = 1 a+b+c= 3 4

1 a=b=c= . 4

II. Bt ng thc Cauchy-Bunhiacovski. Bt ng thc Cauchy-Bunhiacovski c tnh tng qut rt cao. Thng ch dng mc sau y:Vi a1 , a2 , . . . , an , x1 , x2 , . . . , xn l cc s thc, ta lun c (a1 .x1 + a2 .x2 + + an xn )2 a1 2 + a2 2 + + an 2 . x1 2 + x2 2 + + xn 2 . a2 an a1 = = = , vi quy c rng vi x1 x2 xn x1 = 0 th a1 = 0, x2 = 0 th a2 = 0, . . . C du ng thc khi v ch khi c c s m mong mun bt ng thc phi chng minh, cn la chn cch vit mt s hng di dng tch an xn (phn no l an , phn no l xn ). 1. Php bin i thun Cauchy-Bunhiacovski Kh nhiu bt ng thc trong cc k thi quc t, v ch quc gia ca nhiu nc trn th gii.Tuy nhin nu xut pht t (a + b + c)2 chng ta thu c cc bt ng thc c bn. Nhng cc bi ton v dng ny trong nhng nm gn y thng kh hn v xut pht t cc biu thc i xng khc nh: 1 1 1 + + a b c
2

1 1 1 + + ab bc ca

a b c + + b c a

;...

Trong mc ny chng ta chng minh mt s bt ng thc trong cc k thi quc gia cc nc , quc t v xy dng phng php chng minh v xy dng cc bt ng thc dng ny.

3.3. Chuyn chn lc v bt ng thc

79

Bi 1. Vi a, b, c > 0, a + b + c = 1. Chng minh rng 1+a 1+b 1+c + + 2 1a 1b 1c c nh sau: 2a 2b 2c + + 2 1a 1b 1c + 2b 1 1 a 1b + 2c b c a + + a b c 1 1 b 1c a b c + + b c a (1) . (1)

Bi gii . Chng ta trnh by mt cch gii hon ton khc vi p n

(1) 3 +

2a hay

1 1 c 1a

a hay

1 1 c b+c

+b

1 1 a c+a

+c

1 1 b a+b

3 2 (2)

P = Ta c

bc ca 3 ab + + . c (b + c) a (c + a) b (a + b) 2 ab + c bc + a ca b
2

ab b+c+ c (b + c)

bc c+a+ a (c + a)

ca a+b b (a + b)

p dng bt ng thc Cauchy-Bunhiacovski, suy ra: ab + c bc + a ca b


2

P.2 (a + b + c) .

Mt khc ta c (p dng: (a + b + c)2 3 (ab + bc + ca)) ab + c bc + a ca b


2

3 (a + b + c) .

80

Tri h Hng Vng ln th 6-2010

Thu c P 2, ta chng minh c (2) (1) l pcm. Bi 2. Vi a, b, c > 0. Chng minh rng: P = b c 27 a + + bc (c + a) ca (a + b) cb (b + c) 2(a + b + c)2

Bi gii. Ta c a + bc = b + ca c ab
2

a c+a+ bc (c + a) P.2 (a + b + c)

b a+b+ ca (a + b)

c b+c ab (b + c)

(1)

Mt khc ta c a + bc b + ca c ab
2

1 1 1 + + a b c

27 (a + b + c)

(2)

T (1), (2) suy ra: P

27 (pcm). 2(a + b + c)2

Bi 3. Vi a, b, c > 0, abc = 1, hy tm gi tr nh nht ca biu thc : P = Bi gii. Ta c 1 1 1 + + a b c = 1 1 1 + 2 + 2 a2 (b + c) b (c + a) c (a + b)


2

1 1 1 b+c+ c+a+ a+b b c+a a b+c c a+b Mt khc ta c 1 1 1 + + a b c


2

2P (a + b + c)

(1)

1 1 1 + + ab bc ca

3 (a + b + c) = 3 (a + b + c) abc

(2)

T (1), (2) suy ra: P

3 = Pmin (khi a = b = c = 1). 2

3.3. Chuyn chn lc v bt ng thc

81

T cc bi gii mu trn chng ta xy dng phng php gii cho cc bt ng thc dng ny nh sau: Bc 1. Gch nhng tha s dng tng trong bt ng thc tm biu thc xut pht. Bc 2. T biu thc xut pht m t Cauchy-Bunhiacovski biu thc chnh c mt trong bt ng thc. Bc 3. S dng mt s bt ng thc trung gian quen thuc chng minh bt ng thc. Sau y chng ta xt mt s v d minh ho: Bi 1. Vi a, b, c > 0, abc = 1. Chng minh rng: P = a3 1 1 1 1 + 3 + 3 (ab + bc + ca) . (b + c) b (c + a) c (a + b) 2

Bi gii. Ta c 1 1 1 + + a b c
2

1 a a (b + c)

a (b + c) +

1 b b (c + a)

b (c + a) +

1 c c (a + b) (1)

c (a + b)

P.2(ab + bc + ca) Mt khc: 1 1 1 + + a b c T (1),(2) suy ra: P 1 (ab + bc + ca) . 2


2

= (ab + bc + ca)2

(2)

Bi 2. Bi gii. Ta c a2 b2 c2 + + b c a
2

a2 b2 c2 c+a+ a+b+ b+c b c+a c a+b a b+c

(1)

P.2(a+b+c)

82

Tri h Hng Vng ln th 6-2010

Mt khc ta c a2 b2 c2 + + b c a T (1) v (2) suy ra: P


2

(a + b + c)2

(2)

1 (a + b + c) . 2

Bi 3. Vi a, b, c > 0. Chng minh rng: P = a6 b6 c6 1 + 3 + 3 (a + b + c) . b3 (a + c) c (a + b) a (b + c) 2

Bi gii. Ta c a3 b3 c3 + + b c a =
2

b3 c3 a3 bc + ca + ca + cb + ab + ac b bc + ca c ca + cb a ab + ac P.2 (ab + bc + ca)

(1)

Mt khc ta c a3 b3 c3 + + b c a T (1) v (2) suy ra: P


2

(ab + bc + ca)2

(2)

1 (ab + bc + ca) . (pcm) 2

2. Php bin i nghch Cauchy-Bunhiacovski Nhiu bt ng thc xoay vng hay c xy dng t cc php bin i nghch Cauchy-Bunhiacovski m chng ta s trnh by trong mc ny. Bi 1. Vi p, q, r v x, y, z l cc s thc khng m.Chng minh rng: P = p 2 q r 2 1 2 x + y2 + z2 . x + y2 + z (xy + yz + zx) q+r r+p p+q 2

Bi gii. nhanh chng xy dng cch gii cho cc bt ng thc dng ny chng ta trnh by cc bc c th nh sau

3.3. Chuyn chn lc v bt ng thc

83

Bc 1. Thm vo cc s hng rt (p + q + r) lm tha s chung, ta c P = p 2 x + x2 q+r q r 2 y2 + y2 + z + z 2 (x2 y 2 z 2 ) r+p p+q x2 y2 z2 (x2 y 2 z 2 ) + + q+r r+p p+q +

P = (p + q + r)

Bc 2. Biu din di dng Cauchy-Bunhiacovski: P = 1 2 q+r


2 2

r+p
2

p+q

x q+r

y r+p

z p+q

(x2 + y 2 + z 2 ). Suy ra: 1 1 2 P (x + y + z)2 x2 + y 2 + z 2 = (xy + yz + zx) x + y2 + z2 . 2 2 Du ng thc xy ra khi v ch khi : p q r = = . y+zx x+zy x+yz Bi 2. Vi a, b, c > 0, tm gi tr nh nht ca biu thc: P = Bi gii. Ta c 4b 5c 3a +3 + +4 + + 5 12 b+c c+a a+b 4 5 3 12 + + P = (a + b + c) b+c c+a a+b 1 2 2 2 P = b+c + c+a + a+b 2 2 2 2 3 4 5 . 12 + + b+c c+a a+b 2 1 3 + 2 + 5 12 2 P = 3a 4b 5c + + . b+c c+a a+b

84

Tri h Hng Vng ln th 6-2010

Vy: Pmin = 1 3+2+ 5 2


2

b+c a+b c+a 12 khi : = = . 2 3 5

3. Bt ng thc th t xy dng t bt ng thc Cauchy-Bunhiacovski.

Nh bt ng thc Cauchy-Bunhiacovski chng ta chuyn mt s bt ng thc th t n gin thnh nhng bt ng thc th t bc 2. Bi 1. Vi Chng minh rng: a2 b b2 c c2 a + 2 + 2 a+b+c c2 a b Bi gii. Ta c (a + b + c)2 = Ta chng minh: a2 b b2 c c2 a ab2 bc2 ca2 + 2 + 2 2 + 2 + 2. c2 4 3 a 4 3b 4 3c a b a b + b c + c a a3 b4 + b3 c4 + c3 a4 a3 b3 (a b) + b3 c3 (b c) + c3 a3 (c a) 0 a3 (b3 c3 + c3 )(a b) + b3 c3 (b c) + c3 a3 (c a) 0 a3 (b3 c3 )(a b) + c3 [a3 (a b) + b3 (b c) + a3 (c a)] 0 a3 (b3 c3 )(a b) + c3 [a3 (c b) + b3 (b c)] 0 a3 (b3 c3 )(a b) + c3 (b c)(b3 a3 ) 0. (Hin nhin ng). Suy ra: (a + b + c)2 a+b+c a2 b b2 c c2 a + 2 + 2 c2 a b a2 b b2 c c2 a + 2 + 2. c2 a b
2

a c b a c b b. b+ c. c+ a. a c a a b b c . c2 b a2 c b2 a + 2 + 2 a2 b c .

a2 b b2 c c2 a + 2 + 2 c2 a b

3.3. Chuyn chn lc v bt ng thc

85

(pcm) Bi 2. Vi a b c > 0. Chng minh rng: 1 1 c2 1 a2 b2 + 3+ 3 + + . 3 c a a ca ab bc Bi gii. Ta c 1 c 1 b 1 a 1 c 1 1 1 1 b 1 a + + = . . . + . . . + . . . ca a a b b c c a a ab bc b b c c b2 c2 a2 a2 b2 c2 + + 3 + 3+ 3 . a3 b3 c b3 c a Ta chng minh bt ng thc: a2 b2 c2 b2 c2 a2 + 3+ 3 3 + 3+ 3 a3 5 b c b c a b c3 + c5 a3 + a5 b3 a5 c3 + b5 a3 + c5 b3 a3 b3 (a2 b2 ) + b3 c3 (b2 c2 ) + c3 a3 (c2 a2 ) 0 a3 (b3 c3 + c3 )(a2 b2 ) + b3 c3 (b2 c2 ) + c3 a3 (c2 a2 ) 0 a3 (b3 c3 )(a2 b2 ) + c3 (b2 c2 )(a3 b3 ) 0 (Hin nhin ng). Suy ra: 1 1 1 + + ca ab bc
2 2 2

b2 c2 a2 + 3+ 3 c3 a b

1 1 1 b2 c2 a2 + + 3 + 3 + 3. c a b ca ab bc (pcm) Hon ton tng t ta chng minh c bt ng thc Vi a b c > 0. Chng minh rng a3 b b3 c c3 a + 3 + 3 (a + b + c) c3 a b Ti liu tham kho 1. Nguyn Vn Mu (2005), Bt ng thc, nh l v p dng, NXB Gio dc.

86

Tri h Hng Vng ln th 6-2010

2. Nguyn Vn Mu (2002), a thc i s v phn thc hu t, NXB Gio dc.

3.4

Mt s nhn xt v ging dy chuyn ng dng nguyn l Dirichlet


L Th Thanh Bnh, Trng THCS Bnh Minh, Hi Dng

Tm tt ni dung 3. Trong qu trnh ging dy, bn cnh vic cung cp h thng kin thc v cc k nng c bn cho hc sinh, ngi thy cn tm ti khai thc h thng kin thc nng cao nhm bi dng pht trin t duy suy lun Ton hc cho hc sinh nng khiu vi mong mun cc em s tr thnh nhng ch nhn tng lai c kh nng t duy nhy bn, linh hot, sng to, c tin cy cao nhm p ng c yu cu ngy cng cao ca nn kinh t trong thi i cng nghip hin i. Trong bi ny, chng ti trnh by mt s nhn xt nh l nhng kinh nghim thc tin trong vic "Hng dn hc sinh s dng nguyn l Dirichlet gii mt s bi tp hnh hc" bc THCS. 3.4.1 Phn m u

Nhn xt rng, vic gii cc bi ton thng da vo cc nh ngha v tnh cht c trnh by chi tit trong phn l thuyt. Ni dung cc bi ton l xoay quanh vic vn dng v khai thc cc kha cnh khc nhau ca cc khi nim v c trng c bn ca vn ang xt. Cc bc gii ca mi bi ton tuy vn thng qua 4 bc c bn (c hiu, xy dng lc gii, thc hin gii theo lc chn v xem li) nhng thng ngn gn hn. Cc suy lun trong qu trnh gii mi bi ton theo lc trn thng

3.4. Mt s nhn xt v ging dy chuyn ng dng nguyn l Dirichlet

87

rt t nhin v i t d n kh. Nhng bi ton c lc gii r rng, d nhn bit l nhng bi ton dng c bn, chun mc. Bn cnh nhng bi ton c bn v chun mc cn c mt s bi ton dng phc tp hn m sau khi c xong ni dung, hc sinh cha nhn ra c lc gii v cha xc nh c n thuc dng ton c bn (quen thuc) no trong chng trnh. Thm ch c nhng bi ton khi xem li gii hc sinh c th vn khng hiu ti sao li c nhng suy lun nh vy m trong sch gio khoa cha cp n. Trong mt s trng hp, li gii s dng mt vi khng nh tuy rt hin nhin nhng hc sinh li cha h c bit n. Nhng bi ton c cch gii nh vy thng c coi l dng ton khng mu mc. l nhng dng ton kh thng xut hin trong cc k thi chn hc sinh gii cc cp v k thi tuyn sinh vo cc lp chuyn Ton trn ton quc. Trong bi ny, chng ti ghi li nhng iu gp v cch gii quyt chuyn "ng dng nguyn l Dirichlet" gii mt s bi tp hnh hc bc THCS (t lp 7 n lp 9).

3.4.2

Phn ni dung

Nguyn l Dirichlet Nguyn l Dirichlet l mt trong nhng nguyn l n gin, c dng kh ph bin trong s hc, i s v hnh hc v c pht biu bng nhiu cch khc nhau. Sau y l cch pht biu theo ngn ng "th" v "lng": Nu nht m con th vo n ci lng , vi m > n ( m, n l cc s t nhin) th tn ti mt ci lng cha t nht 2 con th. Nguyn l c th m rng nh sau : Nu nht m con th vo n ci lng, vi m > n k(m, n, k l cc s t nhin)

th tn ti mt ci lng cha t nht k + 1 con th.

88

Tri h Hng Vng ln th 6-2010

Phng php chung gii mt bi ton bng cch s dng nguyn l Dirichlet ta cn thc hin cc bc sau: 1. Tm hiu bi, xc nh hai i tng ca bi ton. S lng mi i tng trong gi thit ca bi ton dng ny l cc s nguyn dng. 2. Xy dng thut gii: a. Tin hnh phn chia cc i tng trong gi thit ca bi ton thnh hai tp hp cc i tng {A}, {B}. y l bc quan trng nht ca tin trnh gii ton. Vic phn chia nh vy thng da trn tnh cht ca tng loi yu t. c s phn t ln hn c chn lm "th", tp hp kia chn lm "lng". Nu vi "th" v "lng", th bi ton c gii xong. V d V d 1. Cho t gic ABCD. Dng ba mu xanh, , vng t mu cc nh ca t gic. Chng t rng c hai nh c t cng mu. Phn tch. - Xc nh cc i tng ca bi ton: nh, mu t. - Xc nh s lng ca tng loi i tng: 4 nh, ba mu. - Cc i tng trong gi thit ca bi ton c phn chia thnh hai tp hp: Tp {A} gm 4 nt v tp {B} gm ba mu xanh, , vng. "lng". Ta coi tp {A} l th, tp {B} l lng (v 4 > 3). - So snh s phn t ca hai tp hp gn mi tp hp vi "th" hoc b. Xc nh v so snh s phn t ca mi tp hp {A}, {B}, tp hp no

trong bi ton ang xt ta ch ra c hai tp hp cc i tng tng ng

- S dng nguyn l Dirichlet a ra kt lun. Theo nguyn l Dirichlet

tn ti mt lng cha khng t hn hai th. iu c ngha l c hai im

3.4. Mt s nhn xt v ging dy chuyn ng dng nguyn l Dirichlet

89

c t cng mu. Gii. S nh c t mu l 4. S mu dng t l 3. V 4 > 3 nn theo nguyn l Dirichlet c t nht hai nt cng mu. Cu hi khai thc: T bi tp n gin, kt qu d nhn thy ta tip tc t ra nhiu tnh hung khc nhau a n ra bi ton tng qut nhm hiu r hn v mi quan h gia hai i tng. - Nu ch dng hai mu t th s im c t mu t nht l bao nhiu chc chn c hai im c t cng mu? - Tng qut: nu s im c t mu l a, s mu dng t l b (a, b l cc s t nhin) th a v b quan h nh th no vi nhau lun c t nht hai im c t cng mu? (a ln hn b t nht 1 n v) - Nu ch dng hai mu t th s im c t mu t nht l bao nhiu chc chn c ba im c t cng mu? Tm mi quan h gia a, b trong trng hp ny? (a b 2 + 1) V d 2. Trn mt t giy k vung c 7 ng k ngang v 9 ng k dc. Giao im ca mt ng k ngang vi mt ng k dc c gi l nt. Ngi ta t cc nt trn t giy bng hai mu xanh v . Chng minh rng c t nht hai nt cng mu.

- Xc nh cc i tng ca bi ton: nt, mu t. - Xc nh s lng ca tng loi i tng: S lng cc nt trn t giy l 7 9 = 63. S mu dng t l 2. - Cc i tng trong gi thit ca bi ton c phn chia thnh hai tp hp:

90

Tri h Hng Vng ln th 6-2010

- So snh s phn t ca hai tp hp gn mi tp hp vi "th" hoc "lng". Ta coi tp {A} l th, tp {B} l lng (v 63 > 2). - S dng nguyn l Dirichlet a ra kt lun. Theo nguyn l Dirichlet tn ti mt lng cha khng t hn hai th. iu c ngha l c khng t hn hai nt cng mu. Gii. S lng cc nt trn t giy l 7 9 = 63 (nt).

Tp {A} gm 63 nt v tp {B} gm hai mu xanh v .

S mu dng t l 2 (mu).

V 63 > 2 nn theo nguyn l Dirichlet c t nht hai nt cng mu. Cu hi khai thc: 1) Kt qu bi ton thay i nh th no nu ta t cc nt trn t giy bng: a. 31 mu khc nhau ? b. Dng trong khong t 2 n 31 mu? Tr li: Kt qu bi ton khng thay i do s "th "lun ln hn s "chung". 2) Nu dng 31 mu khc nhau t ta c th khng nh: c t nht ba nt c t cng mu hay khng? V sao? Tr li: Theo nguyn l Dirichlet m rng ta khng nh chc chn c t nht ba nt c t cng mu v 63 > 2 31. 3) Hy t mt bi tng t V d 2? Trn y l dng bi tp n gin nht, gip hnh thnh r cc bc suy lun. Ta tip tc t ra cc tnh hung kh hn nh bit trc s phn t ca tp hp "th" phi xc nh tp hp "lng" v s "lng" ph hp. V d sau y trnh by mt cch to ra tp hp "lng".

3.4. Mt s nhn xt v ging dy chuyn ng dng nguyn l Dirichlet

91

V d 3. Trong tam gic u c cnh bng 4 (n v di, c hiu n cui bi vit) ly 17 im. Chng minh rng trong 17 im c t nht hai im m khong cch gia chng khng vt qu 1. Phn tch. T iu kin "khong cch gia hai im khng vt qu 1" v cnh ca tam gic u bng 4 gi cho ta tm n mt i tng hnh hc khc tp hp 17 im cho. c c "t nht hai im m khong cch gia chng khng vt qu 1" th ta coi tp hp 17 im l tp hp "th" suy ra tp hp cc i tng mi l tp hp "lng". Suy ra s phn t ca tp hp cc i tng mi ny phi nh hn 17. Bng cc suy lun trn hy tm cch to ra cc "lng" nht "th"? Gii. Chia tam gic u c cnh bng 4 thnh 16 tam gic u c cnh bng 1 (nh hnh v). V 17 > 16, theo nguyn l Dirichle, tn ti t nht mt tam gic u cnh bng 1 c cha t nht 2 im trong s 17 im cho. Khong cch gia hai im lun khng vt qu 1. Ta chng minh rng khong cch gia hai im bt k nm trong tam gic u khng ln hn cnh tam gic. Ta k hiu hai im K, L nm trong tam gic ABC u, khi ta c KAL < 60o. Mt trong hai gc cn li ca BC, ta c AE > AK.Trong AKL khng nh hn 60o, chng hn ALK 60o AK > KL. Gi E l giao im ca AK vi cnh ), nn AB > AE. Kt hp cc kt qu trn ta suy ra iu cn chng minh. rn cho hc sinh c kh nng linh hot v t duy sng to, ta tip tc gii thiu cc bi tp tng t, hc sinh phi to tp hp cc "lng" bng cc cch khc nhau nh trong cc v d sau y. V d 4. Trong mt phng cho 2009 im sao cho c 3 im bt k c t nht 2 im cch nhau mt khong khng vt qu 1. Chng minh rng tn ti ABE, AEB 60o (n l gc ngoi ca AEC

92

Tri h Hng Vng ln th 6-2010

mt hnh trn bn knh bng 1 cha t nht 1005 im. Gii. Ly mt im A bt k trong 2009 im cho, v d ng trn C1 tm A bn knhbng 1. + Nu tt c cc im nm trong hnh trn C1 th bi ton hin nhin ng. + Nu tt c cc im B m khong cch gia A v B ln hn 1 th ta v ng trn C2 tm B bn knh bng 1. Khi , xt mt im C ty trong s 2007 im cn li. Xt 3 im A, B, C, v AB > 1 nn theo gi thit th c AC 1 hoc BC 1. Ni cch khc, im C phi thuc C1 hoc C2 . Suy ra 2007 im khc B v A phi nm

trong C1 hoc C2 . Theo nguyn l Dirichlet, c mt hnh trn cha t nht 1004 im. Tnh thm tm ca hnh trn ny th hnh trn ny chnh l hnh trn bn knh bng 1 cha t nht 1005 im trong 2009 im cho. V d 5. Trong hnh trn c din tch bng 1 ta ly 17 im bt k, khng c ba im no thng hng. Chng minh rng c t nht 3 im lp thnh mt 1 tam gic c din tch nh hn . 8 Phn tch. - Trc ht cn nhc li nguyn l Dirichlet m rng. - Da vo bi hy xc nh xem i tng no trong bi ton c coi l tp hp "th"? - T cc iu kin " hnh trn c din tch bng 1 " v "tam gic c din 1 tch nh hn " gi cho ta ngh n i tng hnh hc no? 8 - Vy i tng no c coi l "lng" trong bi ton ny? - Mi "lng" cha bao nhiu con th? 1 = 8. 8 - Hy chia hnh trn c din tch bng 1 thnh cc hnh c din tch bng 1 nhau v bng ? 8 - Xc nh s "lng"? (17 1) : (3 1) = 8 hoc 1 :

3.4. Mt s nhn xt v ging dy chuyn ng dng nguyn l Dirichlet

93

Gii. Chia hnh trn thnh 8 phn bng nhau. Mi phn c din tch l 1 . 8 Do 17 : 8 = 2( mod 1) nn theo nguyn l Dirichlet c 1 phn cha t nht

3 im. Ba im ny l nh ca mt tam gic c din tch nh hn din tch mi hnh qut. Vy c t nht 3 im trong 17 im cho lp thnh mt tam gic c din 1 tch nh hn . 8 Cu hi tng qut ho: Kt qu ca bi ton thay i nh th no nu ta ly trong hnh trn n im (n N, n 3)? Phn tch. Trong trng hp ly n im trong hnh trn (n N, n 3), ta xt hai trng hp sau y: Trng hp 1: Nu n = 2k + 1(k N, k 1) ta chia hnh trn thnh k 1 phn bng nhau, mi phn l 1 hnh qut c din tch bng . k Trng hp 2: Nu n = 2k(k N, k 2) ta chia hnh trn thnh k 1 1 phn bng nhau, mi phn l 1 hnh qut c din tch bng . k1 Lp lun tng t ta cng suy ra kt qu nh trn. Trong mt s bi tp hnh hc ngoi s dng nguyn l Dirichlet ta cn phi kt hp vi cc phng php khc nh phng php gii bi ton cc tr, xp x,. . . V d 6. Trong hnh vung c cnh bng 1 cho 33 im bt k. Chng minh rng trong cc im cho c th tm c 3 im lp thnh tam gic c din 1 tch khng ln hn . 32 Gii. Chia hnh vung cnh 1 thnh 16 hnh vung con nh hnh v. V 33 > 216 1 nn theo nguyn l Dirichlet c mt hnh vung con (cnh ) cha t nht 4 3 trong 33 im cho. Ta chng minh 3 im ny lp nn mt tam gic c

94

Tri h Hng Vng ln th 6-2010

din tch khng ln hn

1 . 32 1 Gi s 3 im A, B, C nm trong hnh vung DEF G cnh . Ta xt 2 4 trng hp sau y: Trng hp 1: C mt cnh ca tam gic nm trn cnh ca hnh vung.

Gi s cnh AB ca tam gic nm trn cnh DG ca hnh vung. K ng 1 1 1 cao CH. Ta c SABC = CH AB CH DG ED DG = . 2 2 32 Trng hp 2: Khng c cnh no ca tam gic nm trn cnh ca hnh vung. Qua nh B, ta k ng thng song song vi cnh hnh vung v ct cnh AC ti M. Gi AH, CK ln lt l ng cao cc tam gic ABM, CBM. Xt SABC = SAM B + SCBM 1 1 = AH BM + CK BM 2 2 1 = BM (AH + CK) 2 1 BM ED DG ED = . 32 1 Vy trong mi trng hp ta lun c: SABC . 32 Tng t nh V d 6 hc sinh d dng gii c bi tp hay v kh sau y: Trong hnh vung cnh 4cm ngi ta t 33 im trong khng c ba im no thng hng. Chng minh rng t 33 im ni trn lun c th tm c 3 im sao cho din tch tam gic c nh l 3 im khng vt qu 1 2 dm . 2 ( thi chn hc sinh gii lp 9 tnh Hi Dng nm hc 2008-2009) V d 7. Trong mt hnh vung cnh bng 7, ly 51 im. Chng minh rng c 3 im trong 51 im cho nm trong mt hnh trn c bn knh bng 1. Phn tch. - Trc ht cn nhc li nguyn l Dirichlet m rng.

3.4. Mt s nhn xt v ging dy chuyn ng dng nguyn l Dirichlet

95

- Da vo cu hi ca bi ton, xc nh xem i tng no trong bi ton c coi l tp hp "th"? tp hp "lng"? Mi "lng" cha bao nhiu con th? - Tm cch chia hnh vung cnh bng 7 thnh 25 "lng" ? Gii. Chia hnh vung cnh bng 7 thnh 25 hnh vung bng nhau, cnh ca 7 mi hnh vung nh bng . 5 V 51 im cho thuc 25 hnh vung nh, m 51 > 225 nn theo nguyn l Dirichlet, c mt hnh vung c cha t nht 3 im (3 = 2 + 1) trong s 51 7 im cho. Hnh vung cnh bng c bn knh ng trn ngoi tip l: 5 7 5
2

- Xc nh s "lng"? (51 1) : (3 1) = 25.

7 5

98 < 1. 100

Vy bi ton c chng minh. Hnh trn ny chnh l hnh trn bn knh bng 1, cha hnh vung ta ch ra trn. 98 100 thnh 1, k thut ly xp x rt quan trng v cn thit khi tm li gii ca gii V d 7 ta cn s dng php xp x nhm l trn s v t nhiu bi tp. i khi ta cn ly xp x da vo hnh dng ca cc hnh trong tng trng hp c th. Sau y l mt v d in hnh. V d 8. Cho 13 im phn bit nm trong hay trn cnh mt tam gic u c cnh bng 6cm. Chng minh rng lun tn ti hai im trong s 13 im cho m khong cch gia chng khng vt qu 3cm. ( thi vo lp 10 chuyn Ton trng HSP H Ni nm hc 2008-2009)

Phn tch. - T cu hi ca bi ton, em hy xc nh xem i tng no c coi l "th"?

96

Tri h Hng Vng ln th 6-2010

- C 13 th, mun nht t nht hai th vo cng mt lng th s lng nhiu - Tm cch chia tam gic u thnh 12 phn m khong cch ln nht gia hai im trong mi phn khng vt qu 3cm. Gii. Gi s tam gic cho l ABC. Gi M, N, P l trung im ca cc cnh BC, CA, AB v G l trng tm ca tam gic ABC. Ly A0 , B0 , C0 , X, Y, Z, T, S, R ln lt l trung im ca cc on thng GA, GB, GC, BM, CM, CN, AN, AP, BP. Tam gic ABC c chia thnh 12 phn nh hnh v. Theo nguyn l Dirichlet, trong s 13 im cho tn ti hai im cng thuc mt phn. Do cnh ca tam gic ABC bng 6cm nn GA0 = AA0 = GB0 = BB0 = CC0 = GC0 = 3cm. Do , hai im ni trn tho mn yu cu bi. V d 9. Trong hnh vung c cnh bng 4 ly 33 im phn bit. Chng minh rng c ba im nm trong phn chung ca ba hnh trn c cng bn knh l 2. ( thi vo lp 10 chuyn Ton HSP TP. H Ch Minh nm hc 20082009) Gii. Chia hnh vung cho thnh 16 hnh vung n v (cc cnh song nn theo nguyn l Dirichlet, tn ti t nht 3 im nm trong hoc trn cnh ca mt hnh vung n v. Gi s l ba im A, B, C trong hoc nm trn cnh ca hnh vung n v MNP Q. Ta c MP = 2 v vi mi im E thuc hnh vung MNP Q th 2 = MP AE. T hnh trn (A, 2) ph ton b hnh vung MNP Q. Tng t, cc hnh trn (B, 2), (C, 2) cng ph ton b hnh vung MNP Q. Vy ba hnh trn (A, 2), (B, 2), (C, 2) u cha hnh vung MNP Q nn ba im A, B, C nm trong phn chung ca ba hnh trn ni trn. song vi cc cnh ca hnh vung cho v c di bng 1). Do 33 > 16 2 nht l bao nhiu? (13 1) : (2 1) = 12.

3.4. Mt s nhn xt v ging dy chuyn ng dng nguyn l Dirichlet

97

V d 10. Cho hnh bnh hnh ABCD, k 17 ng thng sao cho mi ng 1 thng chia ABCD thnh hai hnh thang c t s din tch bng . Chng minh 3 rng trong 17 ng thng c 5 ng thng ng quy. Phn tch. Chn tp hp 17 ng thng l tp hp "th", mun chng minh trong 17 ng thng c 5 ng thng ng quy th phi to ra c tp hp "lng" l cc im c bit trong hnh bnh hnh sao cho s xc nh v tr cc im c bit ? im l (17 1) : (5 1) = 4. Cn c vo cc iu kin cn li ca bi ton

Gii. Gi M, Q, N, P ln lt l cc trung im ca AB, BC, CD, DA. V ABCD l hnh bnh hnh nn MN//AD//BC; P Q//AB//CD. Gi d l mt trong 17 ng thng cho. Nu d ct AB ti E; CD ti F ; P Q ti L th LP, LQ ln lt l ng trung bnh ca cc hnh thang AEF D, EBCF. Ta c : S(AEF D)/S(EBCF ) = 1/3 hoc S(EBCF )/S(EBF C) = 1/3 suy ra LP/LQ = 1/3 hoc l LQ/LP = 1/3. Trn P Q ly hai im L1 , L2 tha mn iu kin L1 P/L1 Q = L2 Q/L2 P = 1/3 khi L trng vi L1 hoc L trng vi L2 . Ngha l nu d ct AB v CD th d phi qua L1 hoc L2 . Tng t, trn MN ly hai im K1 , K2 tha mn iu kin K1 M/K1 N = K2 N/K2 M = 1/3 khi nu d ct AD v BC th d phi qua K1 hoc K2 . Tm li, mi ng thng trong s 17 ng thng cho phi i qua mt trong 4 im L1 ; L2 ; K1 ; K2 . V 17 > 4 4 nn theo nguyn l Dirichlet, trong 17 ng thng s c t

nht 5 ng thng (5 = 4 + 1) cng i qua mt trong bn im L1 ; L2 ; K1 ; K2 (5 ng thng ng quy, pcm).

98

Tri h Hng Vng ln th 6-2010

3.4.3

Bi tp vn dng

Bi tp 1. Trong hnh vung cnh bng 1 cho 5 im bt k. Chng minh rng trong cc im cho c th tm c 2 im sao cho khong cch gia 2 chng khng ln hn . 2 Bi tp 2. Cho hnh vung ABCD c AB = 14cm. Trong hnh vung c nh du 76 im phn bit. Chng minh rng tn ti mt ng trn c bn knh 2cm cha trong n t nht 4 im trong s cc im trn. ( thi vo lp 10 chuyn Ton H Vinh nm hc 2005-2006) Bi tp 3. Cho mt hnh vung c cnh bng 10. Bn trong hnh vung ta nh du 201 im. Chng minh rng lun tm c mt tam gic m cc 1 nh l im c nh du c din tch khng ln hn (nu 3 im nh 2 du thng hng, th ta coi tam gic vi nh l cc im c din tch bng 0 ). Bi tp 4. Bn trong mt hnh ch nht kch thc 3 4 ta nh du 6

im. Chng minh rng lun tm c hai im nh du cch nhau mt khong khng ln hn 5. ABC u c cnh AB = 1. Bn trong tam gic ta nh

Bi tp 5. Cho

du 5 im phn bit. Chng minh rng tn ti hai trong 5 im nh du cch nhau mt khong b hn 0, 5. Bi tp 6. Trong mt phng cho tp hp M gm 25 im c tnh cht l vi 3 im bt k thuc M tn ti hai im cch nhau mt khong b hn 1. Chng minh rng lun tm c mt ng trn c bn knh 1 cha trong n khng t hn 13 im thuc M. Bi tp 7. Cho 2009 im trn mt phng sao cho trong bt k 3 im no cng c

3.4. Mt s nhn xt v ging dy chuyn ng dng nguyn l Dirichlet

99

2 im m khong cch gia chng nh hn 1. Chng minh rng c t nht 1005 im nm trong mt ng trn bn knh 1. Bi tp 8. Bn trong ng trn (O, R) ta nh du 7 im, khng c im no trng vi tm ca ng trn. Chng minh rng lun tm c hai im nh du cch nhau mt khong nh hn. Bi tp 9. Cho mt tp hp gm 6 im nm trong trong mt phng c tnh cht l 3 im bt k thuc tp hp l nh ca mt tam gic vi cc cnh c di khc nhau. Chng minh rng cnh nh nht ca mt tam gic l cnh ln nht ca mt tam gic khc. Bi tp 10. Cho mt tp hp gm 9 ng thng m mi ng ct hnh 2 vung thnh hai t gic c t s din tch bng . Chng minh rng c t nht 3 3 trong 9 ng thng ng quy. Bi tp 11. Bn trong mt a gic li 2ncnh ta ly im P . Qua mi nh ca a gic v P ta k mt ng thng. Chng minh rng tn ti mt cnh a gic khng c im chung vi cc ng thng va k. Bi tp 12. Cho a gic u (H) c 14 nh. Chng minh rng trong 6 nh bt k ca (H) lun c 4 nh l cc nh ca mt hnh thang. ( thi vo lp 10 chuyn Ton, Tin HKHTN-HQG H Ni nm hc 2005-2006) Bi tp 13. Trong hnh ch nht kch thc 1 2 ta ly 6n2 + 1 im ( n l 1 s nguyn dng). Chng minh rng tn ti 1 hnh trn vi bn knh cha n khng t hn 4 trong s cc im cho. 3.4.4 Hng dn cch gii

Bi tp 1. Chia hnh vung cnh bng 1 thnh 4 hnh vung con cnh

1 2 nh hnh v. C 5 im nm trong 4 hnh vung nn phi c mt hnh vung

100

Tri h Hng Vng ln th 6-2010

cha t nht 2 trong 5 im cho. Hai im ny nm trong ng trn c ng knh l ng cho ca hnh vung con cha n nn khong cch gia 2 . chng khng vt qu ng knh ng trn c di 2 Bi tp 2. Chia hnh vung ABCD thnh 25 hnh vung nh c cnh bng 14 cm. V 76 : 25 = 3( mod 1) nn theo nguyn l Dirichlet tn ti mt 5 hnh vung nh IJKH cha t nht 4 im trong s 76 im cho. Gi O 14 14 l tm hnh vung IJKH. Ta c IJ = cm nn IK = 2cm. Suy ra 5 5 7 2cm. OI = 5 Do ng trn ngoi tip hnh vung IJHK c tm O bn knh OI cha 7 2 < 2 nn ng trn tm O tt c cc im trong hnh vung IJKH v 5 bn knh 2cm tho mn iu kin bi cho. Bi tp 3. Th l tp hp 201 im, lng c xc nh nh sau: Ta chia hnh vung ban u thnh 100 hnh vung nh bng cc ng thng song song vi hai cnh lin tip ca hnh vung .Mi hnh vung nh c cnh bng 1. V cc im c nh du nm trong hnh vung bn u, nn cc im phi thuc vo mt trong cc hnh vung nh. Ta coi 100 hnh vung nh l lng. C 201 th c nht vo 100 lng, suy ra c mt lng c nht khng t hn 3 th. Gi s A, B, C l 3 im thuc hnh vung MNP Q c 1 cnh MN = 1. Ta chng minh c rng SABC . 2 Bi tp 4. Ta chia hnh ch nht theo hnh v di y v coi tp hp 5 min a gic l "lng". Mi min a gic hoc l mt hnh thang vung hoc l mt ng gic. Khong cch xa nht gia hai im trn bin a gic bng 5. Bi tp 5. Trc ht ta chng minh rng khong cch gia hai im bt k nm trong tam gic u khng ln hn cnh tam gic. Ta k hiu hai im K, L nm trong ABC u, khi ta c KAL < 60o . Mt trong hai gc cn li

3.4. Mt s nhn xt v ging dy chuyn ng dng nguyn l Dirichlet

101

ca

l giao im ca AK vi cnh BC, ta c AE > AK. Trong 60o (n l gc ngoi ca suy ra iu cn chng minh.

AKL khng nh hn 60o , chng hn ALK 60o AK > KL. Gi E ABE, AEB > AEC), nn AB > AE. Kt hp cc kt qu trn ta

Nhn xt cng vi s 0, 5 gi cho ta tm mt tp hp cc i tng hnh hc khc tp hp 5 im nh du. Ta k hiu M, N, P ln lt l trung im cc cnh ca ABC. Cc on thng MN, MP, NP chia tam gic ban BMP, CNP, MNP } c cnh bng u thnh 4 tam gic u { AMN,

0, 5. Ta coi tp {A} gm 5 im l th, tp {B} gm 4 tam gic u lit

k trn l lng. Theo nguyn tc Dirichlet tn ti mt lng cha t nht hai th. iu c ngha l tn ti t nht hai im c nh du nm bn trong hoc trn cnh ca mt trong 4 tam gic u lit k. Ta k hiu K, L l hai im nh du v xt cc trng hp sau: a) K, L nm trong AMN.Theo nhn xt nu trn KL < MN = 0, 5. b) K, L nm trn on thng MN, v cc im khng th trng vi cc im M, N do KL < MN. Bi tp 6. Gi s A l im thuc M, ta dng ng trn (A, 1). Nu mi im cn li ca M nm trong ng trn , th ta c ngay iu cn chng minh. Gi s B l im thuc M nm ngoi ng trn . Ta xt mt im C bt k thuc M khc A, B. Theo gi thit ta c hoc AC < 1 hoc BC < 1. Ta coi tp hp 23 im thuc M l th, tp hp hai ng trn (A, 1), (B, 1) l lng. Nh vy 23 th c nht vo hai lng, theo nguyn l Dirichle tn ti mt lng cha 12 th. Ngha l tn ti mt ng trn (A, 1) hoc (B, 1) cha trong n 13 im thuc M. Bi tp 7. Ly mt im bt k trong 2009 im cho lm tm v ng trn bn knh 1, cn li 2008 im trn mt phng. Theo gi thit trong 3 im bt k c 2 im m khong cch gia chng

102

Tri h Hng Vng ln th 6-2010

nh hn 1 nn c 1 im nm ngoi ng trn th c mt im nm trong ng trn. Chng hn, im A (O) suy ra OA > 1, do B (O). / O v tnh c tm na l 1005 im. Bi tp 8. Chia ng trn thnh 6 hnh qut c gc tm bng nhau. Coi tp 7 im l th, tp hp 6 hnh qut l lng. Bi tp 9. Ta k hiu {A1 , A2 , . . . , A6 } l tp hp im cho. Ta xt mt Ta c tt c 1004 cp im nn c t nht 1004 im thuc ng trn tm

tam gic bt k c nh ti cc im . V di cc cnh ca cng mt tam gic khc nhau, nn ta s sn cnh c di nh nht bng mu . Hai cnh cn li ta sn xanh. Vi cch lm nh vy tp hp cc on thng An Am ni mu xanh. Ta cn chng minh rng tn ti mt tam gic c c 3 cnh mu . Tht vy ta xt cc on thng c chung u mt l A1 . Tp hp cc on thng c coi l th. Tp hp cc mu dng sn cc on l lng. C 5 th c nht vo hai lng, khi tn ti mt lng cha t nht 3 th. iu c ngha l c t nht 3 on chung u mt A1 c sn cng mu. Ga s c t nht 3 on thng A1 A2 , A1 A3 , A1 A4 c sn , khi A2 A3 A4 phi c mt cnh mu , chng hn A2 A3 . Vy tn ti mt tam gic c 3 cnh . Nu c t nht 3 on mu xanh cng u mt l A1 A2 , A1 A3 , A1 A4 th A2 A3 A4 phi c 3 cnh mu . Nu A2 A3 A4 c cnh v gi s A2 A3 l cnh di nht ca n, th n chnh l cnh nh nht ca mt tam gic khc. hai im bt k {An , Am } thuc tp hp im cho hoc c mu hoc c

Bi tp 10. K hiu ABCD l hnh vung cho, dn , n = 1, 2, 3, . . . , 9 l cc ng thng ct hnh vung. MN v P Q l cc ng trung bnh ca hnh vung. T iu kin bi ton ta suy ra mi ng thng dn ct MN hoc P Q ti mt trong cc im I, J, K, L khc nhau(xem hnh di y) Coi tp hp cc ng thng dn l th , tp hp im {I, J, K, L} l lng.

3.4. Mt s nhn xt v ging dy chuyn ng dng nguyn l Dirichlet

103

Bi tp 11. Nu P thuc vo mt ng cho ca a gic, chng hn l AB, th P A, P B l cc ng thng trng nhau khng ct c cnh a gic. Ta xt P khng nm trn bt k ng cho no. Ta nh s cc nh ca a gic A1 A2 . . . A2n v xt ng cho A1 An+1 ct a gic thnh hai phn. Ta coi P l im trong ca a gic A1 A2 . . . An+1 . Cc ng thng P An+1 , P An+2, . . . , P A2n , P A1 khng th ct cc cnh An+1 An+2 , An+2 An+3 , An+3 An+4 , . . . A2n A1 . S cc ng thng cn li l P A2 , P A3 , . . . , P An c th ct cc cnh . Ta coi lng l tp hp cc ng thng P A2 , P A3 , . . . , P An1 v c c thy n1 lng. Th l tp hp cc cnh An+1 An+2 , An+2 An+3 , An+3An+4 , . . . A2n A1 . C c thy n + 1 th. iu c ngha l tn ti hai cnh cng b ct bi mt ng thng. iu ny khng th xy ra, v a gic l li. Bi tp 12. Cc nh ca a gic u (H) chia ng trn ngoi tip n thnh 14 cung bng nhau, mi cung c s o l = . Cc dy ni hai nh 7 ca (H) chn cc cung nh c s o l , 2, 3, . . . , 7. Do vy di cc dy ch nhn 7 gi tr khc nhau. 65 = 15. V 15 dy 2 ny c di nhn khng qu 7 gi tr khc nhau nn theo nguyn l Dirichlet Ly 6 nh ca (H) th s dy ni hai trong 6 nh l: phi c 3 dy cng di. Trong 3 dy lun c hai dy khng chung u mt. Tht vy, nu 2 dy bt k trong 3 dy chung u mt th 3 dy to thnh mt tam gic u, suy ra s nh ca (H) chia ht cho 3. iu ny v l v (H) c 14 nh (14 khng chia ht cho 3). D thy, 2 dy bng nhau ca mt ng trn khng chung u mt th 4 u mt ca chng l 4 nh ca mt hnh thang cn. T suy ra trong 6 nh bt k ca (H) lun c 4 nh l cc nh ca mt hnh thang. Bi tp 13. Chia cc cnh ca hnh ch nht thnh n on v 2n on bng 1 nhau, mi on c di l . Ni cc im chia bng cc ng thng song n song vi cc cnh ca hnh ch nht ta c 2 2n = 2n2 hnh vung nh vi

104

Tri h Hng Vng ln th 6-2010

1 . n V (6n2 + 1) : 2n2 = 3( mod 1) nn theo nguyn l Dirichlet, tn ti mt 1 hnh vung nh cha t nht 4 im. V hnh vung c cnh ni tip ng n 2 trn bn knh v ng trn ny c cha trong ng trn ng tm 2n 1 1 bn knh nn ta suy ra tn ti mt hnh trn bn knh cha khng t hn n n 4 trong s cc im cho. cnh l 3.4.5 Kt lun

Bng cc phng php phn tch, d on, hnh thnh thut gii, phn loi bi tp sau nhiu nm lin tc ging dy chuyn chng ti thu c mt s kt qu sau y: Cc em hc sinh rt ho hng, t tin khi s dng nguyn l Dirichlet gii cc bi tp hnh hc khng mu mc, nim vui c thay th bi thi i ngi mi khi gp bi ton tng t trc kia khi cc em cha c lm quen vi chuyn ny. Nh vy chuyn gp phn tch cc ho hot ng ca hc sinh ng thi nng cao cht lng dy v hc ca thy v tr. Chuyn gp phn tng thm kh nng sng to cho hc sinh, qua pht trin t duy Ton hc, gip cc em yu Ton hc hn v ngy cng say m vi b mn. Ti liu tham kho 1. Nguyn Vn Mu, Trn Nam Dng, ng Huy Run, V nh Ha, ng Hng Thng, 2008, Chuyn chn lc T hp v ton ri rc, NXB Gio Dc.

3.5. S dng tnh n iu ca hm s tm gii hn

105

3.5

S dng tnh n iu ca hm s tm gii hn


Trng THPT Chuyn Hong Vn Th, Ha Bnh

3.5.1

Cc tnh cht

1. K hiu: l mt trong cc tp: [a; b], (a; b), (a; b], [a; b), (; a), (; a], (b; +), [b; +), R. 2. nh l 1: Cho f : l hm lin tc khi : 1. Phng trnh f (x) = x c nghim phng trnh fn (x) = x c nghim. 2. Gi ; ln lt l cc mt tri, mt phi ca bit lim[f (x) x] v + lim[f (x) x] cng dng hoc cng m.
x x

Khi phng trnh f (x) = x c nghim duy nht phng trnh fn (x) = x c nghim duy nht. Trong fn (x) = f (f (...f (x)...)).
n ln

Chng minh. 1. (a) Nu phng trnh f (x) = x c nghim l x0 th x0 cng l nghim ca phng trnh fn (x) = x. (b) Nu phng trnh f (x) = x v nghim th f (x) x > 0 hoc f (x) dn n phng trnh fn (x) = x cng v nghim. x < 0 vi mi x , do fn (x) > x hoc fn (x) < x vi mi x ,

2. (a) Gi s phng trnh f (x) = x c nghim duy nht l x0 th r rng y cng l nghim ca phng trnh fn (x) = x. t F (x) = f (x) x, do F (x) l hm lin tc nn trn cc khong (x0 ; ) v (; x0 ) F (x) gi nguyn mt du.

106

Tri h Hng Vng ln th 6-2010

(x0 ; ) v (; x0 ) f (x) > x, x \{x0 }.

+ Nu lim[f (x) x] v lim[f (x) x] cng dng th F (x) > 0 trong +


x x

Xt x1 \{x0 } f (x1 ) > x1 f (f (x1 )) > f (x1 ) > x1 fn (x) > x1 . x1 khng l nghim ca phng trnh fn (x) = x. fn (x) = x c nghim duy nht: x = x0 . + Nu lim[f (x) x] v lim[f (x) x] cng m chng minh tng t. +
x x

(b) Ta thy mi nghim ca phng trnh f (x) = x u l nghim ca phng trnh fn (x) = x do nu phng trnh fn (x) = x c nghim duy nht th phng trnh f (x) = x c nghim duy nht.

xn+1 = f (xn ), x N . Khi :

3. nh l 2: Cho f : l hm ng bin, dy (xn ) tha mn: 1. Nu x1 < x2 th (xn ) l dy tng. 2. Nu x1 > x2 th (xn ) l dy gim.

Chng minh. Tht vy:

1. Ta chng minh xn < xn+1 bng phng php quy np.

- Vi n = 1, ta c x1 < x2 , mnh ng. - Gi s mnh ng vi n = k, k N , tc l uk < uk+1 , ta chng bin) uk+1 < uk+2 (pcm). 2. Chng minh tng t. minh mnh ng vi n = k + 1, c f (uk ) < f (uk+1) (do f l hm ng

C th m rng nh l nh sau: n k, xk . Khi : Cho k N v f : l hm ng bin. Dy (xn ) tho mn: xn+1 = f (xn ),

3.5. S dng tnh n iu ca hm s tm gii hn

107

1. Nu xk < xk+1 th xn < xn+1 , n k. 2. Nu xk > xk+1 th xn > xn+1 , n k. 4. nh l 3: Cho f : l hm ng bin, dy (xn ) tha mn:

xn+1 = f (xn ), x N . Khi :

1. Cc dy (x2n ) v (x2n+1 ) n iu trong mt dy tng v mt dy gim. 2. Nu (xn ) b chn th = lim x2n v = lim x2n+1 . 3. Nu f l hm lin tc th , l nghim ca phng trnh: f (f (x)) = x. (1) V vy nu phng trnh (1) c nghim duy nht th = . V: lim xn = = . Chng minh. 1. V f (x) l hm gim nn hm f (f (x)) ng bin, p dng nh l 2 ta c iu phi chng minh. 2. Suy ra trc tip t phn a. 3. Ta c f (f (x2n )) = f (x2n+1 ) = x2n+2 , v lim f (f (x2n )) = lim x2n+2 = lim x2n = f (x) l hm lin tc = f (f ()).

Chng minh tng t ta cng c: f (f ()) = . Vy , l nghim ca phng trnh f (f (x)) = x.

108

Tri h Hng Vng ln th 6-2010

3.5.2

Cc v d

V d 3.9. Cho dy s (un ) nh sau: u1 = 2, 7 u3 3un+1(un+1 1) = un + 1, n N . n+1 Chng minh rng dy s (un ) c gii hn hu hn. Gii. Ta c: u3 3un+1(un+1 1) = un + 1, n+1 (un+1 3 = un , 1) 3 u + 1, un+1 = n n N n N n N

Do u1 = 2, 7 > 0 nn bng quy np ta chng minh c un > 1, n 1 x + 1, c f (x) = > 0 vi mi x = 0 v f (x) l hm 3 3 x2 lin tc, do f (x) l hm ng bin trn trn R. Li c un+1 = f (un ), n N , v u1 = 2, 7 > u2 = 3 2, 7 + 1 nn ta suy ra Xt hm f (x) = c (un ) l dy gim m li b chn di ti 1 nn dy (un ) c gii hn hu hn. V d 3.10. Cho dy s (un ) tha mn: u1 = 2, 9 un , n N . un+1 = 3 + 2 1 un Chng minh rng dy s (un ) c gii hn hu hn. Gii. Xt hm f (x) = N, n 2. 3

x 1 < 0, x > , x > 1, c: f (x) = x2 1 2 (x2 1)3 1 f (x) l hm nghch bin trn (1; +). 3 , n N . Tht vy: Ta chng minh 3 < un < 3 + 2 - Bi ton ng vi n = 1. 3+

3.5. S dng tnh n iu ca hm s tm gii hn

109

- Gi s bi ton ng vi n = k, k N ta chng minh bi ton ng vi n = k + 1. Ta c uk > ra 3 v uk+1 = f (uk ), m f (x) l hm gim trn (1; +) nn suy f (uk ) < f ( 3) uk+1 < 3 + 3 > 0 uk+1 = 3+ 3+ uk u2 1 k > 3 3 . 2

Li c uk > Vy

a, b l nghim ca phng trnh f (f (x)) = x. Xt hm F (x) = f (f (x)) x, vi 3 < x < 3 +

3 , n N . 2 T p dng nh l 3, ta suy ra c: a = lim x2n+1 , b = lim x2n , trong 3 < un <

3 , c F (x) = 2 3 f (x).f (f (x)) 1. Do 3 + > f (x) > 3 v 0 > f (x) vi mi 3 < 2 3 3 x < 3+ nn F (x) < 0 vi mi 3 < x < 3 + , li c F ( 3) > 2 2 3 0, F 3 + < 0. 2 phng trnh F (x) = 0 c nghim duy nht l a. lim u2n+1 = lim u2n = lim un = a. Vy dy (un ) c gii hn hu hn khi n +.

V d 3.11. Cho s thc a v dy s (un ) xc nh nh sau: u1 = a un+1 = un + sin un , n N , n 2. Tm lim un . Gii. Xt hm f (x) = x + sin x, x R, ta c f (x) = 1 + cos x 0, x R, khi vi x1 , x2 (x1 < x2 ) ta c: trong [x1 ; x2 ] c hu hn im m ti f (x) = 0, v vy f (x) ng bin trn [x1 ; x2 ] f (x1 ) < f (x2 ). Vy f (x) ng bin trn R.

110

Tri h Hng Vng ln th 6-2010

Nu a = k, k Z, d dng chng minh c bng qui np un = a, n N . Nu k2 < a < k2 + (1) C un+1 = f (un ) v f (x) l hm ng bin trn R, m u1 = a < u2 = a + sin a nn theo nh l 2 ta c (un ) l dy tng li c (un ) b chn trong khong (k2; + k2). lim un t b = lim un , ta c b l nghim ca phng trnh: b = b + sin b sinb = 0. Nu k2 + < a < (k + 1)2 (2) Kt hp vi (1) b = + k2. sin a > 0.

C un+1 = f (un ) v f (x) l hm ng bin trn R, m u1 = a > u2 = a + sin a nn theo nh l 2 ta c (un ) l dy gim li c (un ) b chn trong khong ( + 2k; (k + 1)2). lim un , t c = lim un , ta c c l nghim ca phng trnh: c = c + sin c sin c = 0. Kt hp vi (2) ta c c = + k2. Vy lim un = + k2. V d 3.12. Cho dy s (xn ) nh sau: x1 = a, a > 2 xn+1 = a a + xn , n N . Chng minh rng dy s (xn ) c gii hn hu hn. Gii. Bng quy np ta chng minh c 0 xn a, n N . Xt hm f (x) = a a + x, x [0; a] c xn+1 = f (xn ) v f (x) = 4 a a+x a+x 1 < 0, x [0; a]

sin a < 0.

p dng nh l 2 ta c dy (xn ) c tch thnh hai dy con (x2n ) v

f (x) l hm nghch bin.

3.5. S dng tnh n iu ca hm s tm gii hn

111

(x2n+1 ), trong c mt dy tng v mt dy gim. Mt khc ta li c dy (xn ) b chn suy ra tn ti lim x2n = v lim x2n+1 = , trong , l nghim ca phng trnh: f (f (x)) = x Xt hm F (x) = F (x) = a a+ a a a+ a a a+ a a+x =x a] c 1.

a + x x, vi x [0; 1

a+x a+

a+x a

a+x a+x

Vi x [0; a

a] ta c a a a+ a. a > 1 a+ . a 4 1 2 3 . a a 2 4 a a+

a + x. a + x =

Thay vai tr ca x bi a a+

1 a . a= 2 1 2 3 > . 2 > 0, 12 > 0, 3. 2 2 4 a a + x chng minh tng t ta c a a+x a+ a a + x > 0, 3.

F (x) < 0, 9 < 0 F (x) l hm nghch bin, li c F (0) > 0, F ( a) < 0. Phng trnh F (x) x = 0 c nghim duy nht = . lim x2n = lim x2n+1 = lim xn . Vy c lim xn = T vi T tho mn f (f (T )) = T .

6. Bi tp: Bi ton 3.1. Cho dy s (xn ) xc nh nh sau: x1 = 1 3 1 x n+1 = x2 1, n N . 2 n

112

Tri h Hng Vng ln th 6-2010

Chng minh rng dy s (xn ) c gii hn hu hn v tm gii hn . Hng dn. - Bng phng php quy np, ta chng minh c 1 < xn < 0, n 2. 1 - Xt hm s f (x) = x2 1 trn on [1; 0]. Ta c xn+1 = f (xn ), n 2 N . Hm s f (x) gim trn [1; 0], do cc dy con (x2n ), (x2n+1 ) n iu (bt u t x2 ). - Vy tn ti cc gii hn lim x2n = a, lim x2n+1 = b, v a, b [1; 0], a, b l

nghim ca phng trnh f (x) = x. Ta thy, trong on [1; 0], phng trnh f (x) = x c nghim duy nht x = 1 3. lim xn = 1 3. Bi ton 3.2. Cho dy s (un ) xc nh nh sau: u1 = 1 1 , n N . un+1 = 1 + un Tm lim(un ). Hng dn. - Bng phng php quy np, ta chng minh c 0 < un < 1, n 2. - Xt hm s f (x) = v 1 trn on [0; 1]. Ta c un+1 = f (un ), n N x+1 1 < 0 x [0; 1]. (x + 1)2

f (x) =

Hm s f (x) gim trn [0; 1], do cc dy con (u2n ), (u2n+1 ) n iu.

3.5. S dng tnh n iu ca hm s tm gii hn

113

nghim ca phng trnh f (f (x)) = x. Ta thy trong on [0; 1], phng trnh 1 + 5 . f (f (x)) = x c nghim duy nht x = 2 1 + 5 . lim un = 2 Bi ton 3.3. Cho dy s (un ) xc nh nh sau: u1 = a, a R 1 un+1 = cos un , n N . 2 Chng minh rng dy (un ) hi t. Hng dn.

- Vy tn ti cc gii hn lim u2n = a, lim u2n+1 = b, v a, b [0; 1], a, b l

- D thy un (0; 1), n 3. 1 - Xt hm s f (x) = cos x trn khong (0; 1), ta c un+1 = f (un ), n 3. 2 1 Ta c f (x) = sin x < 0 trn khong (0; 1), do cc dy con (u2n ), (u2n+1 ) 2 n iu, bt u t u3 . M (un ) b chn, do tn ti cc gii hn lim u2n = a, lim u2n+1 = b, vi a, b [0; 1], v a, b l nghim ca phng trnh f (x) = x. Ta thy trong on [0; 1], phng trnh f (x) = x c nghim duy nht.

- Vy dy (un ) hi t.

Ti liu tham kho 1. Nguyn Vn Mu (2002)). Cc bi ton chn lc v lng gic, NXB Gio dc.

114

Tri h Hng Vng ln th 6-2010

3.6

S phc v ng dng trong hnh hc


Nguyn ng Pht1 - H V Anh2

3.6.1

M u

Ngy nay, chng trnh Ton hc bc Trung hc ph thng ca hu ht cc nc u c phn kin thc s phc. nc ta, sau nhiu ln ci cch, ni dung s phc cui cng cng c a vo chng trnh Gii tch 12, tuy nhin cn rt n gin. V nhiu l do khc nhau, rt nhiu hc sinh, thm ch l hc sinh kh, gii sau khi hc xong phn s phc cng ch hiu mt cch rt n s: s dng s phc, c th gii c mi phng trnh bc hai, tnh mt vi tng c bit, ... Vic s dng s phc trong nghin cu, kho st hnh hc phng t ra c nhiu thun li, nht l trong vic xem xt cc vn lin quan n cc php bin hnh ca mt phng cng vi hnh hc ca chng. Do khun kh c hn,trong bi vit ny chng ti ch m t mt s kt qu, khi nim c bn ca Hnh hc Euclid phng di dng ngn ng s phc nh gc, khong cch, s ng quy, thng hng, ng thng, ng trn cng vi mt s php di hnh, ng dng dng c bn nht. Cc kt qu c th, y hn, u c trnh by trong [1],[2].[7],[9],[10]. 3.6.2 M t mt s kt qu ca hnh hc phng bng ngn ng s phc

Khong cch gia hai im Cho trc hai im M(m), N(n). Khi , di on MN bng MN = |n m| = d(m; n)
1 2

i hc S phm H Ni THPT Chuyn Vnh Phc

3.6. S phc v ng dng trong hnh hc

115

ng thng Trong mt phng cho trc on thng AB. Khi , im M chia on thng AB theo t s k R \ {1} khi v ch khi MA = k MB, a m = k (b m) trong a, b v m l ta v ca cc im A, B v M theo th t . T , nu k hiu [AB] l ch on thng AB, k hiu (AB) l ch ng thng AB, k hiu [AB) l ch tia AB, ta c cc kt qu sau Cho trc hai im A(a), B(b) phn bit v im M(m). Khi M [AB] t 0 : z m = t (bm) t [0; 1] : m = (1 t)a+ t b (1) M (AB) t R : m a = t(b a) t R : m = (1 t)a + tb (2) nh l 3.1. Cho trc hai im A(a), B(b) phn bit v im M(m). Khi , cc mnh sau tng ng M [AB) t > 0 : m = (1 t)a + tb arg (m a) = arg (b a) ma = t R+ ba

i qua hai im W1 (w1 ), W2 (w2 ) l

T , rng t = t t R, ta thu c phng trnh ca ng thng

(z w1 ) (w2 w1 ) (z w1 ) (w2 w1 ) = 0

(3)

116

Tri h Hng Vng ln th 6-2010

Gc gia hai ng thng Trong mt phng phc, cho hai im M1 (z1 ), M2 (z2 ) v k = arg zk , k = 1, 2. Khi , do (Ox, OM1)+(OM1 ; OM2 ) (Ox; OM2 ) nn (OM1 ; OM2 ) (Ox; OM2) (Ox; OM1) (mod 2) (mod 2)
M2 (z2 )

M1 (z1 )

hay gc nh hng to bi tia OM1 vi tia OM2 bng arg

nh l 3.2. Hai tam gic ABC, A B C ng dng cng hng khi v ch khi c a ca = ba b a

z2 T , nu cho z1 bn im phn bit Mk (zk ), k = 1, 2, 3, 4 th gc (nh hng) to bi ng z4 z2 thng M1 M3 vi ng thng M2 M4 bng arg z3 z1

V hai tam gic ABC, A B C ng dng ngc hng khi v ch khi c a ca = ba b a Tch v hng ca hai s phc Trong mt phng phc cho hai im M1 (z1 ), M2 (z2 ). Khi OM1 OM2 = OM1 OM2 cos M1 OM2 Nu zk c modul bng rk , v c argument bng k th OM1 OM2 = r1 r2 cos(2 1 ) = r1 r2 (cos 1 cos 2 + sin 1 sin 2 ) Do < z1 ; z2 >= 1 (z1 z2 + z1 z2 ) 2

3.6. S phc v ng dng trong hnh hc

117

T suy ra < z1 ; z2 > =< z1 ; z2 > v do < z1 ; z2 > R Tch v hng ca hai s phc cng c cc tnh cht nh tch v hng ca hai vc-t. Ngoi ra < z1 ; zz2 >= z < z1 ; z2 > v < zz1 ; z2 >= z < z1 ; z2 >. Nhn xt. 1. Trong mt phng phc cho hai im M1 (z1 ), M2 (z2 ). Khi < z1 ; z2 > bng phng tch ca O vi ng trn ng knh M1 M2 2. Nu A(a), B(b), C(c) v D(d) l bn im phn bit ca mt phng phc, th AB CD < b a; d c >= 0 Re Tch ngoi ca hai s phc. Din tch tam gic Trong mt phng phc cho hai im M1 (z1 ), M2 (z2 ). Khi OM1 OM2 = |OM1| |OM2 | sin M1 OM2 Nu zk c modul bng rk , v c argument bng k th OM1 OM2 = r1 r2 sin(2 1 ) = r1 r2 (sin 2 cos 1 cos 2 sin 1 ) Do i (z1 z2 z1 z2 ) 2 T , do z1 z2 = z1 z2 nn suy ra Im z1 z2 = 0 z1 z2 = ba dc =0

Tch ngoi ca hai s phc cng c cc tnh cht nh tch ngoi ca hai vc-

Nhn xt.

t trong mt phng, ngoi ra (zz1 )z2 = z (z1 z2 ) v z1 (zz2 ) = z (z1 z2 ) 1. Ba im A(a), B(b), C(c) thng hng khi v ch khi (b a) (c a) = 0 2. Nu A(a), B(b) l hai im phn bit, khng thng hng vi O th ab = a gic nh hng A1 A2 . . . An 2 [OAB], trong , k hiu [A1 A2 . . . An ] l ch din tch i s ca

118

Tri h Hng Vng ln th 6-2010

3. T nhn xt 2, vi ba im A(a), B(b) v C(c) phn bit, khng thng hng, th [ABC] = ng trn ng trn tm M0 (z0 ) bn knh R l tp hp nhng im M(z) sao cho M0 M = R hay |z z0 | = R tc l zz z0 z z0 z + z0 z0 R2 = 0 T , mi ng trn u c phng trnh dng zz + z + z + = 0 trong C v R. 1 (a b + b c + c a) 2

ng trn nay c tm vi ta v , bn knh R =

M t cc php bin hnh phng bng ngn ng s phc


Php di hnh.

Php tnh tin. Php tnh tin theo vc-t = (v) l php bin hnh v bin im M(z) thnh im M (z ) sao cho MM = . Do , biu thc ca v php tnh tin l z = f (z) = z + v Php quay. Php quay tm M0 (z0 ) gc quay l php bin hnh bin M(z) thnh im M (z )m M0 M = M0 M v (M0 M; M0 M ) (mod 2). Php i xng-trc. Php i xng qua ng thng l php bin hnh l trung trc ca MM . T

T , biu thc ca php quay l z z0 = ei (z z0 ) bin mi im M(z) thnh im M (z ) sao cho Php i xng qua trc thc: z = f (z) = z

3.6. S phc v ng dng trong hnh hc

119

Php i xng qua trc o: z = f (z) = z Do 2(Ox; ) = (Ox; OM) + (Ox; OM ( y = (z0 )) nn php i xng qua ng thng i qua gc O v im z0 = ei 2 c biu thc

z = f (z) = ei z T , nu = T ( ) vi = (z0 ) th php i xng qua c biu thc v v z = ei z + 2z0

M (z) M (z) v
M (z )

M (z )
2

2 v M (z )

Php v t.

Php v t tm C(z0 ), t s r R lf php bin hnh bin im M(z) thnh im M (z ) m CM = r CM . Do , c biu thc z = r (z z0 ) + z0 iu kin ng quy, thng hng, vung gc v cng nm trn mt ng trn (ng vin) nh l 3.3. Cho ba ng thng vi phng trnh 1 : (z z1 ) u1 =

0 2 ; (z z2 ) u2 = 0 3 : (z z3 ) u3 = 0. Khi , ba ng thng

120

Tri h Hng Vng ln th 6-2010

1 , 2 , 3 ng quy khi v ch khi (u1 u2 )2 + (u2 u3 )2 + (u3 u1 )2 = 0 nh l 3.4. Ba im M1 (z1 ), M2 (z2 ), M3 (z3 ) thng hng khi v ch khi z3 z1 R z2 z1 =0 (u1 u2 )(z3 u3) + (u2 u3 )(z1 u1 ) + (u3 u1 )(z2 u2 ) = 0

hay Im

nh l 3.5. Bn im Mk (zk ), k = 1, 2, 3, 4 cng nm trn mt ng thng hay ng trn khi v ch khi z3 z2 z3 z4 : R z1 z2 z1 z4

z3 z1 z2 z1

nh l 3.6. Nu A(a), B(b), C(c) v D(d) l bn im phn bit ca mt phng phc, th AB CD < b a; d c >= 0 Re 3.6.3 Mt s v d p dng ba dc =0

H qu 3.1. Bn im Mk (zk ), k = 1, 2, 3, 4 cng nm trn mt ng thng z3 z4 z3 z2 R v R khi v ch khi z1 z2 z1 z4 Bn im Mk (zk ), k = 1, 2, 3, 4 cng nm trn mt ng trn khi v ch z3 z2 z3 z4 z3 z2 z3 z4 khi : R nhng R v R z1 z2 z1 z4 z1 z2 z1 z4

Cho A(a), B(b), C(c) l ba nh ca mt tam gic. Khi tam gic ABC l tam gic u khi v ch khi hai tam gic ABC, BCA l ng dng cng hng. iu ny tng ng vi ca ab = (c a)(c b) = (a b)(b a) ba cb c2 ca bc + ab = a2 b2 + 2ab a2 + b2 + c2 = ab + bc + ca

3.6. S phc v ng dng trong hnh hc

121

Mt khc, tam gic ABC u, nh hng dng (tng ng m) iff php quay tm A, gc quay + (tng ng ) bin B thnh C, do 3 3 Tam gic ABC u, nh hng dng iff a + b + c 2 = 0, v tam gic ABC u, nh hng m iff a 2 + b + c = 0, trong 3 = 1. Bi ton 1. (Napolon) Ly cc cnh BC, CA, AB ca tam gic ABC lm y, dng ra ngoi cc tam gic u vi tm tng ng A0 , B0 , C0 . Chng minh rng A0 , B0 , C0 l nh ca mt tam gic u. Li gii. Gi s tam gic ABC nh hng dng. Gi x l ta v ca im X no trong mt phng. Ta c

a + c1 + b 2 = 0, b + a1 + c 2 = 0, c + b1 + a 2 = 0

Do A0 , B0 , C0 theo th t l trng tm cc tam gic BCA1 , CAB1 , ABC1 nn

3a0 = b + c + a1 , 3b0 = c + a + b1 , 3c0 = a + b + c1

3(c0 + a0 + b0 2 = a + b + c1 + (b + c + a1 ) + 2(c + a + b1 ) = (b + a1 + c 2 ) + (c + b1 + a 2 ) + (a + c1 + b 2 ) 2 = 0

Suy ra iu phi chng minh.

122

Tri h Hng Vng ln th 6-2010

Li gii 2. Gi s tam gic ABC nh hng m, v x l ta v ca im X no trong mt phng. Khi , ta c c=e


i 2 3

B1

(ba0 )+a0 , a = e

i 2 3

(cb0 )+b0 , b = e

i 2 3

A C (ac01)+c0 C0

B0

Suy ra b = ei 3 (ei 3 (c b0 ) + b0 c0 ) + c0
4 2 4 2 2 2

= ei 3 (ei 3 (b a0 ) + a0 b0 ) + ei 3 (b0 c0 ) + c0 = b a0 + ei 3 (a0 b0 ) + ei 3 (b0 c0 ) + c0


i 3

B A0

T c0 a0 = e (b0 a0 ) iu c ngha l tam gic A0 B0 C0 u


A1

Bi ton 2. (BMO 1990 - Shortlist) Cho tam gic ABC. Ly cc cnh lm y, dng ra ngoi ba ngic u. Tm tt c cc gi tr ca n sao cho tm ca ba a gic u l nh ca mt tam gic u. B0 Li gii. Gi s tam gic ABC nh hng m. Gi A0 , B0 , C0 l tm ca cc a gic u dng trn cnh BC, CA, AB (hnh v). Khi BA0 C = CB0 A = AB0 C = 2 n 2 t = ei n v gi a, b, c, a0 , b0 , c0 ln lt l ta v ca cc im A, B, C, A0, B0 , C0 theo th t . T gi thit, ta c a = b0 +(cb0 ), b = c0 +(ac0 ), c = a0 +(ba0 ) T b0 =

C0
2 n

A
2 n

2 n

A0

b a c b a c ; c0 = ; a0 = 1 1 1

Tam gic A0 B0 C0 u khi v ch khi a2 + b2 + c2 = a0 b0 + b0 c0 + c0 a0 0 0 0 Thay a0 , b0 , c0 tm c trn vo, khai trin, rt gn, ta c (1 + + 2 ) (b a)2 + (a c)2 + (c b)2 = 0

3.6. S phc v ng dng trong hnh hc

123

iu ny tng ng vi 1 + + 2 = 0 do n = 3. Bi ton 3. Trn cc cnh ca lc gic li c tm i xng A1 A2 A3 A4 A5 A6 , dng v pha ngoi cc tam gic u Ak Ak+1 Bk (vi k = 1, 2, . . . , 6 v quy c A7 A1 ). Chng minh rng trung im cc on thng Bk Bk+1 l nh ca mt lc gic u (vi k = 1, 2, . . . , 6 v B7 B1 ) Li gii. Gi s lc gic A1 A2 A3 A4 A5 A6 nh hng m. Chn tm i xng O ca lc gic lm gc, gi x l ta v ca im X trong mt phng phc, t nn bk + 2 ak + 4 ak+1 bk = ak+1 + ak . Suy ra bk = bk+3
B1 P6 B6 A1 A2
3

ei 3 = . Khi ak+3 = ak . Do tam gic Ak Ak+1 Bk u, c hng dng,

P1

B2

P2 A3

A6 P5 A5

O A4

B3 B5 P4 B4 P3

Do Pk l trung im ca Bk Bk+1 nn pk = pk + pk+3 =

bk +bk+1 2

k = 1, 2, . . . , 6 T

bk + bk+1 bk+3 + bk+4 (bk + bk+3 ) + (bk+1 + bk+4 ) + = =0 2 2 2

do lc gic P1 P2 P3 P4 P5 P6 nhn O lm tm i xng.

124

Tri h Hng Vng ln th 6-2010

K hiu f l php quay tm O gc quay . Ta c 3 1 b1 + b2 ) = (a2 + a1 + a3 + a2 ) 2 2

f (p1 ) = (

1 = (a2 + a1 + a3 ) 2 1 = a3 + a2 + 2 a1 2 1 = (a3 + a2 + a4 ) = p2 2

Do , f (p1 ) = p2 . Tng t, cng c f (p2 ) = p3 , f (p3 ) = p4 (PCM) Bi ton 4. (IMO 1977) Cho hnh vung ABCD. Dng v pha trong hnh vung cc tam gic u ABK, BCL, CDM v DAN. Chng minh rng trung im cc on thng KL, LM, MN, NK, BK, BL, CL, DM, DN v NA l nh ca mt thp nh gic u. Li gii. Gi s hnh vung ABCD nh hng dng. Chn tm O ca hnh vung lm gc, gi x l ta v ca im X trong mt phng phc. Khi b = ia, c = a, d = ia. t ei 3 = ta c k = (i + )a, = ( + i)a, m = (i )a, n = ( i)a rng a gic P1 Q1 S1 P2 Q2 S2 P3 Q3 S3 P4 Q4 S4 nhn O lm tm i xng, do vi f l php quay tm O, gc quay + th ch cn 6 chng minh f (pk ) = qk , f (qk ) = sk v f (sk ) = pk+1 (k = 1, 2) l
A

D K

P1 Q1 L S1 P2

S4

Q4

P4

S3 N Q3 Q2 S2 P3

3.6. S phc v ng dng trong hnh hc

125

T cch dng, ta c

a 1 a p1 = (k + ) = [(i 1) + (i + 1)] , p2 = [(i + 1) + (i 1)] , 2 2 2 a a q1 = [i(1 + ) + ] , s1 = [1 + i + ] 2 2

Khi , vi = ei 6 th

f (p1 ) = p1 =

a [(i 1) + (i + 1)] = q1 2 a f (q1 ) = q1 = [i + i + ] = s1 2 a f (s1 ) = s1 = [ + i + ] = p2 2

Mt cch tng t, cng c f (p2 ) = q2 , f (q2 ) = s2 , f (s2) = p3 (PCM) Nhn xt. Bi ton ny hon ton c th gii bng phng php ta nh trong [5], hay phng php tng hp nh trong [6], tuy nhin li gii qu di. Li gii c trnh by trn c xut pht t tng s dng php quay vc-t, tuy nhin bng cng c s phc, lm gim i ng k cc ng tc bin i phc tp trn cc vc-t Bi ton 5. (SEA-MO 1998) Cho tam gic ABC. Ly im P khc pha vi C i vi ng thng AB, im Q khc pha vi B i vi ng thng CA v im R cng pha vi A i vi ng thng BC sao cho cc tam gic BCR, ACQ v BAP ng dng. Chng minh rng t gic AP RQ l mt hnh bnh hnh.

126

Tri h Hng Vng ln th 6-2010

Li gii 1. Gi s tam gic ABC nh hng dng v gi x l ta v ca im BR X. t BP = AQ = BC = t, ABP = BA AC CAQ = CBR = , = ei . Khi , t gi thit suy ra p = (t + 1)b ta, v r = (t + 1)b tc q = (t + 1)a tc
P B

Q A

Khi p+q = (t+1)(a+b)t(a+c) = (t+1)btc+(t+1)ata = a+r (PCM) Li gii 2. T gi thit, suy ra cc tam gic BCR, ACQ v BAP ng dng cng hng. Vy rb qa pb = = =zC rc qc pa T p= Suy ra p+q = b zc + (1 z)a =a+r 1z b za a zc b zc ; q= ; r= 1z 1z 1z

Bi ton 6. Trong mt phng cho bn tam gic ABC, AB1 C1 , A2 BC2 v A3 B3 C ng dng, cng hng. Gi A0 , B0 v C0 theo th t l trung im ca A2 A3 , B1 B3 v C1 C2 . Chng minh rng A0 B0 C0 ABC Li gii. Gi x l ta v ca im X. Gi s php ng dng f1 (z) = 1 z + 1 bin tam gic ABC thnh tam gic AB1 C1 , php ng dng f2 (z) = 2 z + 2 bin tam gic ABC thnh tam gic A2 BC2 , php ng dng f3 (z) = 3 z + 3 bin tam gic ABC thnh tam gic A3 B3 C.

3.6. S phc v ng dng trong hnh hc

127

Khi 1 1 b0 a0 = (b1 + b3 a2 a3 ) = [(b1 a) + (a b) + (b a2 ) + (b3 a3 )] 2 2 1 = (1 + 2 + 3 1) (b a) 2 1 Tng t, cng c c0 a0 = (1 + 2 + 3 1) (c a) 2 c0 a0 c a Vy = (PCM) b0 a0 b a Nhn xt Nu t AB = t v = (AC; AB), th bng cch lm tng t nh AC c iu phi chng minh. Bng nhng cch lm nh trn, khng nhng ta chng minh c cc tam gic ng dng, m cn ch ra c chng ng dng cng hng, v cng tm c t s ng dng theo cc t s d cho. Bi ton 7. (Italy MO 1996) Cho ng trn (O) v im A ngoi (O). Vi mi im P trn ng trn, dng hnh vung AP QR, vi cc nh theo ngc chiu kim ng h. Tm qu tch im Q khi P chy khp trn (O). Li gii. Khng mt tng qut, coi ng trn (O) c tm ti gc, bn knh bng 1, gi x l ta v ca im X trn mt phng. Khi , ta c q = ei 2 (a p) + p q = ia + (1 i)p Do , qu tch ca im Q l ng trn c tm ti im A (ia) (tc l A = QO 2 (A)), bn knh R = |(1 i)p| = 2 Bi ton 8. (Bulgaria MO 1997) Cho hai hnh vung n v K1 , K2 vi tm M, N trong mt phng sao cho MN = 4. Bit rng hnh vung K1 c hai cnh song song vi MN, hnh vung K2 c mt ng cho nm trn ng thng MN, tm qu tch trung im XY , trong X l mt im trong ca K1 , Y l mt im trong ca K2 Li gii. Khng mt tng qut, coi M(2), N(2) v gi w l ta v ca im

li gii 1 bi ton 5, ta cng chng minh c b0 a0 = tei (c0 a0 ) v cng

128

Tri h Hng Vng ln th 6-2010

5 i i i W trong mt phng phc. Khi a1 = 2 2 , b1 = 3 2 , c1 = 3 + 2 , d1 = 2 2 5 2 + i 2

v a2 = 2
C1

1 , b2 2

=2

i , c2 2

=2+

1 , d2 2

=2+

i 2

D2

D1 X M A1

Z Y A2 B1 B2 N C2

V Y nm trong hnh vung A2 B2 C2 D2 khi v ch khi y = y1 + y2 i, yk R vi |y1 + y2 2| < T 2


1 2 1 2

1 R vi |x2 | < 2 , |x1 + 2| <

Ta c X nm trong hnh vung A1 B1 C1 D1 khi v ch khi x = x1 + x2 i, xk


1 2

Vy, vi Z l trung im XY th z =
1 < x1 < 2 + 2 , 2 1+ 2 , |u 2 1 2

v |y1 y2 2| <

1 2

x1 +y1 2

+i

x2 +y2 2

= u + iv. suy ra |u| <


2+1 2

Tng t, cng c |v| <

< y1 y2 < 2 +

bt Vy, qu tch im Z l min gic gii hn bi cc ng thng |x| = 2+1 2+1 2+1 2+1 , |y| = , |x + y| = , |x y| = 2 2 2 2 2 2 Nhn xt. V mt hnh hc, qu tch im Z l min trong a gic u

+ v|, |u v| <

1 2 2+1 2 2

c nh l trung im cc on ni cc nh ca hai hnh vung l nh ca 1 A1 B1 C1 D1 , A2 B2 C2 D2 tng ng qua cc php tnh tin theo cc vc-t 2 MN v 1 NM (hnh v) 2 Bi ton 9. (Poland MO 1999) BC DE F A. Chng minh rng Cho lc gic li ABCDEF c A + C + E = 360 v AB CD EF =

AB F D EC = BF DE CA Li gii. Gi w l ta v ca im W trong mt phng phc. t b a = x, c b = y, d c = z, e d = t, f e = u, a f = v.

3.6. S phc v ng dng trong hnh hc

129

Do AB CD EF = BC DE F A nn |xzu| = |ytv| Do A + C + E = 360 nn arg


z y x v

(1) (2)

Do x + y + z + t + u + v = 0 nn

T (1),(2) suy ra xzu = vyt hay xzu + vyt = 0 xt(x + y + z + t + u + v) + (xzu + vyt) = 0 x2 t + xty + xtz + xt2 + xtu + xtv + xzu + vyt = 0 (xt2 + xtz + xtu + xzu) + (x2 t + xty + xtv + vyt) = 0 x(t + z)(t + u) + t(x + y)(x + v) = 0

u t

l mt s thc dng.

x v

z y

u t

= 0 iu ny c ngha l

Do |x(t + z)(t + u)| = |t(x + y)(x + v)| (PCM)

Bi ton 10. Cho t gic li ABCD. Dng cc hnh vung AMBN v CP DQ cng hng. Chng minh rng |MQ2 NP 2 | = 4SABCD Li gii. Coi t gic ABCD nh hng m (hnh v). Gi X, Y, Z, T theo th t l trung im cc cnh AB, BC, CD, DA ca t gic ABCD. Gi w l ta v ca im W trong mt phng phc. rng MP 2 NQ2 = (MP + NQ)(MP NQ) Ta c (p m) (q n) = 2 < z x; n m + p q > = 2 < z x; i(a b + d c) > D = 4i < z x; t y > Vy |(p m)2 (q n)2 | = 4SABCD (Do SABCD = 2SXY ZT )
2 2

M B X A N T Q Y

Z C

Bi ton 11. Xt t gic ABCD khng c hai cnh no song song. Gi Ga , Gb , Gc , Gd theo th t l trng tm cc tam gic BCD, CDA, DAB, ABC. Chng minh rng nu AGa = BGb v CGc = DGd th ABCD l mt hnh

130

Tri h Hng Vng ln th 6-2010

thang cn. Li gii. Gi x l ta v ca im X trong mt phng phc v t s = a + b + c + d. Ta c ga = b+c+d sa sb sc sd = , gb = , gc = , gd = 3 3 3 3 3

Do AGa = BGb nn |a ga | = |b gb | |4a s| = |4b s| < 4a s; 4a s >=< 4b s; 4b s > T 2(|a|2 |b|2 ) =< (a b); s > (1) (2)

Tng t, t CGc = DGd cng c 2(|c|2 |d|2) =< (c d); s > Tr (1) cho (2) v i v, ta c 2(|a|2 |b|2 |c|2 + |d|2 ) =< (a b c + d); (a + b + c + d) > 2(|a|2 |b|2 |c|2 + |d|2 ) = |a + d|2 |b + c|2 aa ad ad + dd = bb bc bc + cc |a d|2 = |b c|2

Tc l AD = BC Cng (1) vi (2) v i v, ta c 2(|a|2 |b|2 + |c|2 |d|2) =< a b d + c; a + b + c + d > v tng t nh trn, thu c AC = BD T (3) v (4) suy ra iu phi chng minh.

(3)

(4)

Bi ton 12. Gi G l trng tm t gic ABCD. Chng minh rng GA Li gii. Gi w l ta v ca im W trong mt phng phc. Do G(g) l trng tm t gic ABCD nn g =
a+b+c+d 4

GD AD = MN, trong M, N theo th t l trung im AD, BC.

3.6. S phc v ng dng trong hnh hc

131

Ta c GA GD < a g; d g >= 0 a+b+c+d a+b+c+d ;d >= 0 < a 4 4 < 3a b c d; 3d b c a >= 0 < (a b c + d) + 2(a d); (a b c + d) 2a d) >= 0 a+d b+c 2 2 < a + d b c; a + d b c >= 4 < a d; a d >
2

= |a d|2 MN = AD

Bi ton 13. (St. Petersburg 2000) Cho tam gic ABC nhn ni tip trong ng trn . ng thng , v gi M l trung im AC. Chng minh rng tam gic BKM cn. Li gii. Khng mt tng qut, coi l ng trn n v, a = x + yi, b = i, c = z + ti. Khi =
x+z 2

l tip

tuyn ca ti B, K l hnh chiu ca trc tm H ca tam gic ABC trn

+i

y+t 2

Do H l trc tm ca tam gic, nn h = x + z + (y + t + 1)i. Khi k = x + z + i. Ta c |b | = V |k | =


x+z 2 2 x+z 2 2 y+t2 2 2

=
1 2

1 2

(x + z)2 + (y + t 2)2

(1) (2)

2yt 2 2

(x + z)2 + (y + t 2)2

T (1),(2) suy ra iu phi chng minh. Bi ton 14. Cho tam gic ABC ni tip ng trn . Gi A1 l trung im cnh BC v A2 l hnh chiu ca A1 trn tip tuyn ca ti A. Cc im B1 , B2 , C1 , C2 c xc nh mt cch tng t. Chng minh rng cc ng thng A1 A2 , B1 B2 , C1 C2 ng quy. Hy xc nh v tr hnh hc im ng quy. Li gii. Khng mt tng qut, coi l ng trn n v. Gi w l ta v ca im W trong mt phng phc.

132

Tri h Hng Vng ln th 6-2010

A2 A

HN

O C

A1

Ta c a1 =

b+c 2

v ng thng A1 A2 l ng thng i qua A1 (a1 ), song

song vi OA, do A1 A2 c phng trnh az az = a b+c a 2 b+c 2

Do aa = 1 nn phng trnh c vit li di dng z a2 z = hay z a2 z = a+b+c a+b+c a2 2 2


a+b+c 2

b+c a2 2

b+c 2

Gi N l tm ng trn Euler ca tam gic, th n =

do A1 A2 i

qua N. Tng t cng c B1 B2 , C1 C2 i qua N (PCM) 3.6.4 Bi tp

1. Cho ABCD l mt hnh vung c nh. Xt tt c cc hnh vung P QRS sao cho P, R nm trn hai cnh khc nhau v Q nm trn mt ng cho ca hnh vung ABCD. Tm tt c cc v tr c th c ca im S.

3.6. S phc v ng dng trong hnh hc

133

(Colombia MO 1997) 2. Xt im P nm bn trong hoc trn bin ca hnh vung ABCD. Tm gi tr ln nht, gi tr nh nht c th c ca f (P ) = ABP + BCP + CDP + DAP (Greece MO 1997) 3. Cho t gic li ABCD. Gi E, F, G, H theo th t l tm cc hnh vung vi cc cnh AB, BC, CD, DA dng ra pha ngoi t gic. Chng minh rng (a) Trung im cc ng cho ca hai t gic ABCD, EF GH l nh ca mt hnh vung. (b) EF v GH vung gc vi nhau v bng nhau 4. Trn cc cnh AB, BC, CA ca tam gic ABC, dng ra pha ngoi ba tam gic ng dng ABC1 , A1 BC, AB1 C. Chng minh rng trng tm hai tam gic ABC, A1 B1 C1 trng nhau. Hi kt lun ca bi ton cn ng khng nu cc tam gic ABC1 , A1 BC, AB1 C dng vo pha trong ca tam gic ABC? 5. Trn ng trn cho trc hai im A, B c nh v mt im M di ng trn . Trn tia MA ly im P sao cho MP = MB. Tm qu tch im P . 6. Cho t gic li ABCD. Ly cc cnh AB, CD lm y, dng ra ngoi cc tam gic vung cn ABX, CDY . Chng minh rng 2XY AC + BD

134

Tri h Hng Vng ln th 6-2010

7. Cho tam gic ABC ni tip trong ng trn n v. Bit rng tn ti (0; ) sao cho a + b sin + c cos = 0 ( y a, b, c l ta v ca cc 2 1+ 2 nh A, B, C), chng minh rng 1 < SABC 2 (Romanian 2003) 8. Gi M, N, P, Q, R, S theo th t l trung im cc cnh AB, BC, CD, DE, EF, F A ca lc gic ABCDEF . Chng minh rng RN 2 = MQ2 +P S 2 MQ PS (Romanian 1994) 9. Cho t gic li ABCD. Gi E, F, G, H theo th t l trung im cc cnh AB, BC, CD, DA. Chng minh rng AB CD BC 2 + DA2 = 2(EG2 + F H 2 ) 10. Xt im M trn ng trong ngoi tip ca tam gic ABC, khng trng vi cc nh ca tam gic. Chng minh rng tm ng trn Euler ca cc tam gic MBC.MCA, MAB l nh ca mt tam gic ng dng vi tam gic ABC. 11. Chng minh rng tam gic ABC vung khi v ch khi ng trn Euler tip xc vi ng trn ngoi tip. 12. Cho tam gic ABC. Gi a , b v c theo th t l cc ng trn nhn cc ng trung tuyn k t A, B v C lm ng knh. Chng minh rng, nu hai trong ba ng trn ny tip xc vi ng trn ni tip ca tam gic ABC, th ng trn th ba cng tip xc vi ng trn ni tip ca tam gic. (Romanian 2008)

3.6. S phc v ng dng trong hnh hc

135

13. Cho t gic ABCD ni tip. Gi Ea , Eb , Ec , Ed theo th t l tm ng trn Euler ca cc tam gic BCD, CDA, DAB, ABC. Chng minh rng cc ng thng AEa , BEb , CEc , DEd ng quy. Hy xc nh v tr hnh hc im ng quy.

Ph lc
Bng cc cng thc c bn thng dng trong vic ng dng s phc vo gii ton hnh hc
Sau y l mt bng (lit k) cc cng thc c bn cn dng n trong vic gii ton Hnh hc phng bng s phc m chng ta s gp sau ny. l nhng cng thc tnh di, tnh ln ca gc, tnh din tch, tnh t s n (ca ba im), tnh t s kp (ca bn im); thit lp iu kin song song, vung gc ca cc ng thng, iu kin (du hiu) ng dng ca hai tam gic, iu kin lin thuc, phng trnh ng trn... 1. OA = aa, AB = (a b)(a b) 2. C (AB) AB AC = (b a)(c a) (abc) = ca ca = (abc) = cb cb (t s n ca ba im A(a), B(b), C(c) l s thc)
2 2

3. Bn im A, B, C, D thng hng hay ng vin cn v l t s kp (a, b, c, d) l thc (a, b, c, d) = C th ca da ca da : = : = (a, b, c, d) cb db cb db ca da ca da : ; v R cb db cb db ca da : R cb db

A, B, C, D thng hng khi v ch khi

A, B, C, D ng vin cn v l
136

3.6. S phc v ng dng trong hnh hc

137

nhng da ca v R. cb db CD

4. Bn im A(a), B(b), C(c), D(d) khng thng hng. Khi AB v AB CD ba R dc ba dc =0

5. Tam gic ABC u, c hng thun (nh hng dng) khi v ch khi a + b + c 2 = 0 trong = cos 2 + i sin 2 = ei 3 3 3
2

ba iR tc l Re dc

(Hay c bit, c th xy ra A B C) Tam gic ABC u, c hng nghch (nh hng m) khi v ch khi a 2 + b + c = 0 trong = cos 2 + i sin 2 = ei 3 3 3
2

6. Hai tam gic A1 B1 C1 v A2 B2 C2 ng dng cng hng khi v ch khi b2 a2 b1 a1 = = ( C) c1 a1 c2 a2 b2 a2 b1 a1 = = ( C) c1 a1 c2 a2 7. Din tch ca tam gic ABC nh hng, vi cc nh A(a), B(b), C(c), c tnh theo cng thc i 4 a a 1 b b 1 c c 1 a a 1 b b 1 c c 1 =0

Hai tam gic A1 B1 C1 v A2 B2 C2 ng dng ngc hng khi v ch khi

Do A(a), B(b), C(c) thng hng khi v ch khi

138

Tri h Hng Vng ln th 6-2010

8. Phng trnh ng thng z + z + = 0 trong C , R 9. Khong cch t im M(z0 ) n ng thng : z + z + = 0 bng | z0 + z0 + | 2

10. Phng trnh ng trn z z + z + z + trong C, R 11. Phng trnh ng trn i qua ba im A(a), B(b), C(c) phn bit, khng thng hng c dng za zc za zc : : = ba bc ba bc (b a)(d c) + (b a)(d c) 12. cos(AB; CD) = 2|b a| |d c|

T AB CD (b a)(d c) + (b a)(d c) = 0

(b a)(d c) + (b a)(d c) 13. sin(AB; CD) = 2i|b a| |d c|

14. AB v CD l hai dy cung ca ng trn z z = R2 . Khi AB CD ab = cd v AB CD ab + cd = 0 (a a = b b = c c = d d = R2 )

15. AB, CD l hai dy cung ca ng trn n v zz = 1, (AB)(CD) = S. Khi s= a + b (c + d) (a a = b b = c c = d d = 1) ab cd

16. Nu C l chn ng cao, h t nh C ca tam gic ABC ni tip trong ng trn n v z z = 1, th c = ab 1 a+b+c 2 c

vi a a = b b = c c = d d = 1

3.6. S phc v ng dng trong hnh hc

139

17. AB l dy cung ca ng trn n v z z = 1. Khi Z [AB] hay Z (AB) z + abz = a + b 18. C l giao im cc tip tuyn ti A v B ca ng trn n v z z = 1. Khi 1 1 2 = + c a b 19. AB l dy cung ca ng trn n v z z = 1. Khi vi mi im M thng AB c xc nh bi h= 1 (m abm + a + b) 2

khng nm trn ng trn, hnh chiu (vung gc) ca M trn ng

20. Tam gic ABC ni tip trong ng trn n v z z = 1, th th din tch S ca n c xc nh bi S= i (a b)(b c)(c a) 4 abc

Ti liu tham kho


[1] Titu Andreescu, Dorin Andrica, Complexnumbers from A to Z . . ., Birkhuser 2006 a [2] I.M. Yaglom, Kompleksnye qisla i ih primenenie v geometrii, Moskva 1963 [3] S.I. Xvarcburd, Izbrannye voproksy matematiki Fakultativnyi kurs 10, Moskva 1980 [4] Nguyn Cnh Ton, Hnh hc cao cp, Nh xut bn Gio dc 1979 [5] V Dng Thy, 40 nm olympic Ton hc quc t, Nh xut bn Gio dc 2001 [6] L Hi Chu, Thi v ch ton Quc t, Nh xut bn tr 2001 [7] on Qunh, S phc vi Hnh hc phng, Nh xut bn Gio dc 1998 [8] Titu Andresscu, Mathematical Olypiads from Around the World [9] H V Anh,S phc vi cc php bin hnh ca mt phng, Lun vn Thc s khoa hc, Trng i hc Khoa hc T nhin 1999 [10] Nguyn Hu in, Phng php s phc v Hnh hc phng, Nh xut bn i hc Quc gia H Ni, 2000 [11] Cc t liu khc ly t website www.mathlinks.ro

140

Phng php l ng gic v p d ng

141

2.6

Phng php l ng gic v p d ng


T Ton, Tr ng THPT Chuyn Lo Cai

Tm t t n i dung 1. L ng gic l ph n ki n th c quan tr ng trong chng trnh ton THPT, ngoi nh ng bi ton lin quan tr c ti p n l ng gic, nh bi n i l ng gic, h th c l ng gic trong tam gic, t gic, phng trnh l ng gic, tch phn c a cc hm s l ng gic,..., th m t l ng khng nh cc bi ton l i c chuy n v lm vi c v i i t ng l l ng gic thng qua cc php t l ng gic. Trong bi vi t nay ta s p phng php l ng gic vo gi i quy t m t s d ng ton c b n l cc bi ton v phng trnh, h phng trnh, cc bi ton ch ng minh b t ng th c v m t s bi ton lin quan n dy s .

2.6.1

Cc k t qu c b n

M t s ng th c l ng gic
1) sin2 x + cos2 x = 1, x R 1 , x = + k 2) 1 + tan2 x = cos2 x 2 1 3) 1 + cot2 x = , x = k sin2 x 1 x 4) cot x + = cot , x = k sin x 2 5) V i ; ; = + k, k Z, ta c: 2 tan + tan + tan = tan . tan . tan + + = n(n Z) 6) V i ; ; = + k, k Z, ta c: 2 + n(n Z) 2

tan . tan + tan . tan + tan . tan = 1 + + =

M t s php t l ng gic

7) N u x2 + y 2 = 1 th, t 8) N u x2 + y 2 = a2 th, t

x = sin t y = cos t

x = a sin t y = a cos t x = cos 2 2 2 9) N u x + y + z = 1 th, t y = sin . cos z = sin . sin

Phng php l ng gic v p d ng

142

x = a cos 2 2 2 2 10) N u x + y + z = a th, t y = a sin . cos z = a sin . sin 11) Trong m i tr ng h p ta u c th l ng gic ha theo hm tan x ho c cot x

2.6.2

p d ng trong gi i phng trnh, h phng trnh


2+ 2 2+x

V d 1. Gi i phng trnh: x =

Gi i. i u ki n: 0 < x 2. t x = 2. cos t, i u ki n: < t < (). Khi phng trnh tr thnh: 2 2 2 cos t = 2+ 2 2+ 2(1 + cos t)

t 2(1 cos ) 2 t 2 cos t = 2 + 2 sin 4 t t 2 cos t = 2(sin + cos ) 8 8 t sin( t) = sin( + )() 2 4 8 Gi i phng trnh (**) v k t h p v i i u ki n (*), ta nh n c 2 gi tr c a 2 2 t th a mn l: t1 = ; t2 = 9 7 2 2 V y phng trnh cho c 2 nghi m: x = 2 cos v x = 2 cos . 9 7 x = y(4 y) V d 2. Cho (x;y;z) l nghi m c a h phng trnh: y = z(4 z) z = x(4 x) Tm t t c cc gi tr m t ng S = x + y + z c th nh n c. 2 cos t = Gi i. Gi s (x; y; z) l nghi m c a h phng trnh cho. C ng v v i v tng ng cc phng trnh c a h , ta nh n c: 3S = x2 + y 2 + z 2 0 = S 0. V S 0 nn trong 3 s x, y, z ph i c t nh t m t s khng m, khng m t tnh t ng qut, ta gi s x 0, t phng trnh (1) c a h ta suy ra 0 x 4. B ng php hon v vng quanh ta c: 0 x, y, z 4. t x = 4 sin2 , v i 0 . Khi t PT(3) suy ra: z = 4. sin2 2, thay vo PT(2), suy ra: y = 4. sin2 4, thay tr l i phng trnh u, ta nh n c x = 4. sin2 8. Nh v y l nghi m c a phng trnh: sin2 8 = sin2 cos 16 = cos 2 k k = ; = (k Z) 7 9

Phng php l ng gic v p d ng Tr ng h p 1: =

143

k v 0 , nn k {0; 1; 2; 3; 4; 5; 6; 7}, tuy nhin 7 v i k {4; 5; 6; 7} hay k {0; 1; 2; 3} u cho cng m t gi tr c a x, y, z. *) V i k = 0 = = 0 = S = 0 2 + *) V i k {1; 2; 3} th S c cng m t gi tr b ng: S = 4 sin2 + 4 sin2 7 7 3 4 sin2 =7 7 k Tr ng h p 2: = v 0 , nn k {0; 1; 2; 3; 4}. 9 *) V i k = 0 = = 0 = S = 0. 2 *) V i k {1; 2; 4} th S c cng m t gi tr b ng: S = 4 sin2 + 4 sin2 + 9 9 4 4 sin2 = 6. 7 *) V i k = 3 th S = 4 sin2 + 4 3

sin2

4 2 + 4 sin2 = 9. 3 3 K t lu n: S {0; 6; 7; 9}. V d 3. Tm cc s th c x, y, z th a mn x6 +y 6 +z 6 6(x4 +y 4 +z 4 )+10(x2 +y 2 +z 2 )2(x3 y+y 3 z+z 3 x)+6(xy+yz+zx) = 0

Gi i. Bi n i v tri thnh: P = (x3 3xy)2 +(y 3 3y z)2 +(z 3 3z x)2 y = x3 3x Do P = 0 x = z 3 3z() z = y 3 3y *) N u x > 2 th y = x(x2 3) > 2 = z = y(y 2 3) > 2. T c ng theo v ba phng trnh c a h (*) ta c: 0 = x3 + y 3 + z 3 4x 4y 4z = x(x2 4) + y(y 2 4) + z(z 2 4) > 0 (V l). *) L p lu n hon ton tng t v i < 2, ta cng c mu thu n. V y |x| 2 x y = 2(4 cos3 t 3 cos t) = 2. cos 3t t x = 2 cos t, 0 t , suy ra: x = 2(4 cos3 3t 3 cos 3t) = 2. cos 9t z = 2(4 cos3 9t 3 cos 9t) = 2. cos 27t k l D n n, cos t = cos 27t t = (k = 0, 1, 2, ..., 13); t = (l = 0, 1, 2, ..., 14) 13 14 Ng c l i, d dng ki m tra c r ng, n u cos t = cos 27t th b (x; y; z) = (2 cos t; 2 cos 3t; 2 cos 9t), th a mn h (*). Thnh th cc b (x; y; z) k th a mn bi l: (2 cos t; 2 cos 3t; 2 cos 9t), v i t = (k = 0, 1, 2, ..., 13) ho c 13 l t = (l = 0, 1, 2, ..., 14). 14

Phng php l ng gic v p d ng

144

2.6.3

p d ng trong ch ng minh b t ng th c

V d 4. Xt cc s th c dng a, b, c, th a mn i u ki n abc + a + c = b. Ch ng minh r ng: a2 Gi i. t P = a2 2 3 10 2 2 + 2 +1 b +1 c +1 3

2 2 3 2 + 2 +1 b +1 c +1 Ta c abc + a + c = b a + c = b(1 ac) N u ac = 1 th a + c = 0 (v l), do a, b > 0. V y ac = 1 Vi t l i u ki n i a = tan A a+c cho d i d ng: b = , i u ny g i cho ta n php t: b = tan B 1 ac c = tan C tan A + tan C Do a, b, c > 0, nn t n t i A, B, C (0; ), th a mn tan B = 2 1 tan A. tan C T y ta nh n c m i lin h gi a A, B, C l: A + B + C = . Khi ta bi n i P = 2. cos2 A 2 cos2 (A + C) + 3. cos2 C = cos 2A cos(2A + 2C) + 3 cos2 C = 2 sin(2A + C). sin C + 3 cos2 C 2 sin C + 3 cos2 C = 2 sin C + 3 3. sin2 C 10 1 10 = 3(sin C )2 3 3 3 sin C = 1 D u "=" x y ra 3 sin(2A + C) = 1 2 2 Khi , (a; b; c) = ( ; 2; ) 2 4 b a abc V d 5. Tm gi tr nh nh t c a bi u th c Q = + + . a + bc b + ca c + ab V i a, b, c > 0 v th a mn i u ki n a + b + c = 1 Gi i. ab 1 1 c Vi t Q d i d ng: Q = + ca + bc ab 1+ 1+ 1+ b a c ab ca ab bc bc ca T gi thi t: a + b + c = 1 + + = 1() c b c a a b bc A = tan a 2 ac B t = tan ; (A, B, C (0; )) b 2 ab C = tan c 2 T (*), ta c: A + B + C = . V y A, B, C l 3 gc c a m t tam gic.

Phng php l ng gic v p d ng 1 Khi , Q = 1 + tan2 1 tan + C 2 = cos2

145 A B sin C + cos2 + 2 2 2

B 1 + tan2 2 1 = 1 + (cos A + cos B + sin C). 2


A 2

C 1 + tan2 2

A+B M t khc, ta c: cos A + cos B + sin C + sin 2. cos + 2 cos 3 2 2 A+B+C 3 = 4. cos = 23. 4. cos 4 6 cos A B = 1 2 C+ 3 3 3 1 3 =1 ) = 1+ . D u "=" x y ra Suy ra: Q 1+ (2 3 sin 2 2 4 2 A+B C 3 = 2 2 A=B= a=b=2 33 6 C = 2 c=74 3 3 a=b=2 33 3 3 V y max Q = 1 + , t c 4 c=74 3 V d 6. Cc s dng x, y, z th a mn i u ki n 1 16xyz (). Tm gi tr nh nh t c a bi u th c x2 + y 2 + z 2 = 4 x + y + z + 4xyz S= 1 + 4xy + 4yz + 4zx Gi i. T gi thi t suy ra 0 < 2x < 1, 0 < 2y < 1, 0 < 2z < 1. t 2x = a = cos A; 2y = b = cos B; 2z = c, A, B 0; . Khi , i u ki n (*) tr thnh: 2 cos2 A + cos2 B + c2 + 2c cos A cos B = 1 (c + cos A cos B)2 (1 cos2 A)(1 cos2 B) = 0 (c + cos A cos B)2 sin2 A sin2 B = 0 [c + cos(A + B)][c + cos(A B) = 0] c = cos(A + B), v (cos(A B) > 0) c = cos C, trong A, B, C l ba gc c a m t tam gic nh n. M t khc A B B C C A A B C tan tan + tan tan + tan tan = 1, suy ra tan2 tan2 tan2 2 2 2 2 2 2 2 2 2 1 . 27 A B C A B C cos2 cos2 cos2 27. sin2 sin2 sin2 2 2 2 2 2 2 (1 + a)(1 + b)(1 + c) 27(1 a)(1 b)(1 c) 28(a + b + c + abc) 26(1 + ab + bc + ca) 28(x + y + z + 4xyz) 13(1 + 4xy + 4yz + 4zx) 13 13 1 Suy ra S . V y min S = . t c khi x = y = z = 28 28 4

Phng php l ng gic v p d ng

146

2.6.4

Dy s v gi i h n

Tr c h t ta xt v d quen thu c sau y. V d 7. Ch ng minh r ng: 2+ 2+ 2 + + 2+ 2 2+ 2 + + 2n1 1 2 = 2 2. cos 2n+1

n d u cn

n d u cn

Gi i. 2+ 2+ 2 + +
n-d u cn

2 + +
n-d u cn

2+

2 + +
n-d u cn

2n1 1 2 = 2 2. cos 2n+1

2+

2 + +
n-d u cn

2 = 2 cos

2n+1

+ 2 sin

2n+1

Ta c cc k t qu : i) 2+ 2 + +
n-d u cn

2 = 2 cos

2n+1

, n N

ii)

2 + +
n-d u cn

2 = 2 sin n+1 , n N 2

Hai k t qu trn c ch ng minh b ng phng php qui n p Ch ng minh (i). *) V i n = 1 th (i) tr thnh: 2 = 2. cos (lun ng). 4 *) Gi s (i) ng v i n = k(k > 1, k N ), ta c 2+ 2 + +
k d u cn

2 = 2 cos k+1 2

*) Xt 2+ 2k+2 2+ 2 + +
k+1 d u cn

2=

2+

2+

2 + +
k d u cn

2=

2 + 2 cos

2k+1

=2| cos Vy

|, v i 0 <

| cos k+2 | = cos k+2 2 2 2 2 + + 2 = 2 cos k+2 , ta c(i) ng v i n = k + 1. Theo 2 2k+2 <

k+1 d u cn

nguyn l qui n p th (i) ng. Ch ng minh tng t ta c (ii). T y ta nh n c pcm.

Phng php l ng gic v p d ng x1 = 1 2 V d 8. Cho dy s xn th a mn i u ki n 1 x xn + n+1 = 2 Ch ng minh r ng dy s c gi i h n, tm gi i h n ?

147

x2 + n

1 4n

cot 1 4 = 1 cot , suy ra x = 1 x + x2 + 1 = Gi i. Ta c: x1 = = 1 2 1 2 2 2 22 2 4 1 cot 3 22 2 1 D on, s h ng t ng qut xn = n cot n+1 , n N (). Ta d dng ki m tra 2 2 1 l i kh ng nh () b ng phng php qui n p ton h c. V y xn = n cot n+1 . 2 2 1 2 cos 2n+1 2 Do lim xn = lim n cot n+1 = lim n+1 . = 2 2 2 sin n+1 2 x1 = 2 V d 9. Cho dy s xn , c xc nh b i: 2xn xn+1 = , n = 2, 3, ... xn + 1
n

Tm lim
i=1

xi .

1 1 1 2x1 = ; x2 = x + 1 = . 1 1 cos 2 cos 3 2 2 2 n 1 B ng qui n p ta ch ng minh c: xk = xi = 2n . sin n+1 . T , 2 i=1 cos k+1 2 n Suy ra: lim xi = . 2 i=1 Gi i. Ta c: x1 = 2= Nh n xt 1. Qua cc v d trn ta th y, vi c a ra cng th c s h ng t ng qut c a dy s , hon ton ph thu c vo vi c bi n i v nh d u (coi) s h ng ban u x1 l gi tr l ng gic c a m t gc c bi t, sau d a vo cc php bi n i l ng gic, ta d on c qui lu t xc nh c a s h ng t ng qut, cu i cng l ch ng minh cng th c d on b ng phng php qui n p, cc bi ton v gi i h n c a dy cng t c gi i quy t. m t s bi ton vi c ch ra cc lin h l khng h n gi n, ta xt ti p cc v d sau. x1 = 3 3 V d 10. Cho dy s (xn ) c xc nh: xn + 2 3 xn+1 = ; n = 2, 3, ... 1 + ( 3 2)xn Ch ng minh r ng: x12n+2 = 1 31 Gi i. Ta c: tan = tan( ) = = 2 3. 12 3 4 31

Phng php l ng gic v p d ng

148

xn + tan 12 Vi t l i xn+1 , d i d ng: xn+1 = . L i c x1 = tan 6 , suy ra: 1 tan xn 12 x2 = tan( + ) v x3 = tan( + + ) = tan( + 2 ) 6 12 6 12 12 6 12 D on: xn = tan[ + (n 1) )]. K t qu ny c ch ng minh b ng 6 12 qui n p ton h c. V y, ta c s h ng t ng qut: xn = tan[ + (n 1) )]. 6 12 Khi : x12n+2 = tan( +(12n+1) ) = tan( +n+ ) = tan = 1 (pcm) 6 12 6 12 4 V d 11. Cho hai dy s dng (xn ); (yn ), n = 1, 2, ..., c xc nh: 1 x =y = 1 1 2 xn , n = 1, 2, 3, ... xn+1 = 2 4yn+1 1 yn y n+1 = 2 4xn+1 + 1 Tm gi i h n c a dy (xn ) v (yn )?
2 Gi i. Tr c h t ta ch ng minh b ng qui n p r ng: x2 + yn = 1(1), n N . n Th t v y: 2 V i n = 1 th x2 + y1 = 1, h th c (1) ng. Gi s (1) ng v i n = k, t c l 1 2 2 xk + yk = 1. Ta i ch ng minh (1), cn ng v i n = k + 1. 2 2 Ta c: 1 = x2 + yk = [xk+1 (4yk+1 1)]2 + [yk+1 (4x2 1)]2 k k+1 4 2 2 x2 (16yk+1 8yk+1 + 1) + yk+1 (16x4 8x2 + 1) 1 = 0 k+1 k+1 k+1 2 4 2 2 (x2 + yk+1 1) + (16x2 yk+1 16x2 yk+1 + 16x4 yk+1 ) = 0 k+1 k+1 k+1 k+1 2 2 (x2 + yk+1 1)(16x2 yk+1 + 1) = 0 k+1 k+1 2 2 x2 + yk+1 = 1. V y (1) ng v i n = k + 1. V y x2 + yn = 1(1), n N . n k+1 yn t xn = sin n ; yn = cos n , t h th c truy h i yn+1 = , ta c: 2 4xn+1 + 1 cos n cos n+1 = 2 4 sin n+1 + 1 2 = cos n+1 (1 4 sin n+1 ) = cos n 2 cos n+1 (4 cos n+1 3) = cos n cos 3n+1 = cos n = 3n+1 = n , do n > 0, n N 1 1 1 T y, suy ra: n = n1 , m t khc x1 = = sin 1 = 3 2 2 = 1 = = n = . V y xn = sin ; yn = cos 4 4.3n1 4.3n1 4.3n1 Suy ra: lim xn = lim sin = sin(lim )=0 4.3n1 4.3n1 lim yn = lim cos = cos(lim )=1 n1 4.3 4.3n1

Phng php l ng gic v p d ng

149

Bi t p
Bi t p 1. Gi i phng trnh: a) 4x3 + 2 1 x2 3x 1 = 0 7 3x = b) x + 29 4 25x Bi t a) b) c) p 2. Gi i cc h phng trnh: 2x + x2 y = y 2y + y 2 z = z 2z + z 2 x = x x3 3x = y(3x2 1) y 3 3y = z(3y 2 1) z 3 3z = x(3z 2 1) 1 1 1 3(x + ) = 4(y + ) = 5(z + ) x y z xy + yz + zx = 1

x2 + y 2 = 9 Bi t p 3. Tm nghi m x, y, z, t c a h : z 2 + t2 = 16 sao cho bi u th c xt + yz 12 A = x + z t gi tr nh nh t. Bi t p 4. Cho x, y, z > 0 v th a mn i u ki n x + y + z = 1. Ch ng minh r ng: xy yz zx 3 + + . z + xy x + yz y + zx 2 Bi t p 5. Cho x, y, z R, th a mn i u ki n x2 + y 2 + z 2 = 1. Ch ng minh 1+ 3 r ng: 2xy + yz + zx 2 Bi t p 6. Cho x, y, z R, th a mn i u ki n x2 y 2 + 2yx2 + 1 = 0. Tm gi tr nh nh t v gi tr l n nh t c a bi u th c: 2 1 + + y(y + 2 x x Bi t p 7. Cho dy s (an ) c xc nh b i: f (x, y) = Tm s h ng t ng qut c a an a1 = 2 Bi t p 8. Cho dy: : an+1 = 2 an , n N Tm s h ng t ng qut c a an 1 + 2) x a1 = 2 2

an+1 =

2 2

1 a2 , n N n

ng d ng php kh v nh l Viete vo hnh h c ph ng a1 = 2 Bi t p 9. Cho dy s (an ) c xc nh b i: an+1 = Tm lim


n i=1

150 2an an + 1

ai

Ti li u tham kh o 1. Nguy n Vn M u, Chuyn ch n l c l ng gic v p d ng, NXB Gio d c 2008. 2. Nguy n Vn M u, Chuyn ch n l c dy s v p d ng, NXB Gio d c 2008. 3. K y u h i ngh khoa h c cc chuyn ton h c trong h THPT chuyn 2005. 4. K y u ton h c tr i h Hng Vng. 5. Cc wedsite ton h c http://www.mathscope.org, http://www.mathlinks.ro 6. T p tr ton h c v tu i tr .

2.7

ng d ng php kh hnh h c ph ng

v nh l Vit vo

Nguy n Quang H p, Tr ng THPT Chuyn Nguy n T t Thnh

2.7.1

Php kh

Cho a th c f (x) c b c m. Gi s f (x) c cc nghi m x1 , x2 , . . . , xn . P (x) , P (x), Q(x) l nh ng a th c. Ta xt hm h u t y = Qx Chng ta s tm: P (xi ) 1) a th c b c m nh n yi = , i = 1, 2, . . . , m lm nghi m. Q(xi ) 2) Quan h gi a cc yi qua cc h s c a f (x). Vi c tm phng trnh a th c v i cc h s tnh qua cc h s c a f (x) nh n y1 , y2 , . . . , ym lm nghi m tng ng v i vi c kh x t h sau. f (x) = 0 P (x) y= Q(x) Suy ra phng trnh a th c g(y) = 0 c n tm.

ng d ng php kh v nh l Viete vo hnh h c ph ng

151

2.7.2

nh l Vit

Gi s a th c b c m : f (x) = am xm + am1 xm1 + + a1 x + a0 , (am = 0) c m nghi m x1 , x2 , . . . , xm . Khi ta c x1 + x2 + + xm = am1 am am2 x x + x x + + x 1 2 1 3 m1 xm = am ............................................... x1 x2 xm = (1)m a0 am

2.7.3

ng d ng

Ta th ng s d ng nh ng k t qu sau y: (a) N u a th c f (x) = x3 ax2 + bx c c ba nghi m khng m x1 , x2 , x3 th a 3 b 3c 3

Ch ng minh. V x1 , x2 , x3 l cc nghi m c a (1) nn theo nh l Vit ta c x1 + x2 + x3 = a x1 x2 + x1 x3 + + x2 x3 = b x1 x2 x3 = c M xi 0(i = 1, 2, 3) nn x1 + x2 + x3 3 Vy x1 x2 + x2 x3 + x3 x1 3 1 2 3

x1 x2 x3

a b 3 c. 3 3 (b) Cho tam gic ABC c cc c nh a, b, c cc bn knh ng trn ngo i ti p, n i ti p l n l t l R, r v cc bn knh cc ng trn bng ti p l ra , rb , rc . t 2p = a + b + c. Khi b1) a, b, c l ba nghi m c a phng trnh: x3 2px2 + (p2 + r2 + 4Rr)x4Rrp = 0 b2) ra , rb , rc l ba nghi m c a phng trnh: x3 (4R + r)x2 + p2 xp2 r = 0 (2) (1)

ng d ng php kh v nh l Viete vo hnh h c ph ng Ch ng minh. r A 2 pa 2 a = b1) Ta c: sin a = A r2 2R 1 + tan2 1+ 2 (p a)2 2 tan a3 2pa2 + (p2 + r2 + 4Rr)a 4pRr = 0.

152

i u ny ch ng t a l nghi m c a phng trnh x3 (4R+r)x2 +p2 xp2 r = 0 (1). Tng t b, c cng l nghi m c a (1). a + b + c = 2p ab + bc + ca = p2 + r2 + 4Rr T y suy ra Cch 2: Ta ch ng minh abc = 4pRr a, b, c l nghi m c a (1) - theo nh l Vit o. b2) Ta s ch ng minh ra + rb + rc = 4R + r ra rb + rb rc + rc ra = p2 ra rb rc = p2 r Khi ra , rb , rc l ba nghi m c a phng trnh (2). A B C Th t v y: ra + rb + rc = p(tan + tan + tan ) = 2 2 2 A B B C C A p = [(tan + tan ) + (tan + tan ) + (tan + tan )] 2 2 2 2 2 2 2 A+B B+C C +A ) ) sin( sin( p sin( 2 ) 2 2 = + + A B B C C A 2 cos cos cos cos cos cos 2 2 2 2 2 2 A B C 2 2 2 p cos 2 + cos 2 + cos 2 = A B C 2 cos cos cos 2 2 2 3 + cos A + cos B + cos C =p A B C 4 cos cos cos 2 2 2 A B C 4 + 4 sin sin sin 2 2 2 =p sin A + sin B + sin C A B C = R(4 + 4 sin sin sin ) 2 2 2 = 4R + r.

ng d ng php kh v nh l Viete vo hnh h c ph ng V r = (p a) tan A b+ca A A = tan = R[sin B + sin C sin A] tan 2 2 2 2 B+C BC A A A = R[2 sin cos 2 sin cos ] tan 2 2 2 2 2 A A BC B+C = 2R cos tan [cos cos ] 2 2 2 2 A B C = 4R sin sin sin 2 2 2

153

A B B C C A tan + tan tan + tan tan ) = p2 2 2 2 2 2 2 +) T S = pr = (pa)ra = (pb)rb = (pc)rc = p(p a)(p b)(p c) Ta c S 3 = (p a)(p b)(p c)ra rb rc = pr.p(p a)(p b)(p c) Do ra rb rc = p2 r +) ra rb + rb rc + rc ra = p2 (tan V d 1. Trong tam gic ABC. ra + r rb + r rc + r ra + r rb + r rc + r tU= + + ,V = . ra r rb r rc r ra r rb r rc r p2 + r 2 Ch ng minh r ng V U = 2 v U + V = . Rr L i gi i. S d ng k t qu : ra , rb , rc l ba nghi m c a phng trnh x3 (4R + r)x2 + p2 x p2 r = 0 Ta tm phng trnh b c 3 nh n ya = (2)

x+r y+1 nghi m. Xt y = hay x = r . xr y1 x3 (4R + r)x2 + p2 x p2 r = 0 y+1 Kh x t h ta c phng trnh x=r y1 2Rry 3 (p2 + r2 2Rr)y 2 + 2(p2 r2 Rr)y (p2 + r2 + 2Rr) = 0

ra + r rb + r rc + r , yb = , yc = l ra r rb r rc r

R rng phng trnh ny nh n ya , yb , yc l nghi m. Khi theo nh l p2 + r2 2Rr p2 + r2 + 2Rr Vit ta c U = ,V = . 2Rr 2Rr p2 + r 2 Do V U = 2 v U + V = Rr V d 2. Ch ng minh r ng trong tam gic ABC ta c
2 2 2 2 2 2 (rb + rb rc + rc )(rc + rc ra + ra )(ra + ra rb + rb )

= (

ra rb rc 3 ra rb rc 2 ) +( ) (ra + rb + rc )2 ra rb rc (ra + rb + rc )3 . r r

ng d ng php kh v nh l Viete vo hnh h c ph ng

154

L i gi i. 2 2 2 2 Ta tm phng trnh b c 3 nh n ya = rb + rb rc + rc , yb = rc + rc ra + ra , yc = 2 ra + ra rb + rb lm nghi m. p2 r 2 2 Ta c ya = (rb + rc ) rb rc = (4R + rra ) . Kh x t h ra x3 (4R + r)x2 + p2 x p2 r = 0 yx = x(4R + r x)2 p2 r Ta c x = 4R + r y + p2 . t T = 4R + r, ta c phng trnh 4R + r

y 3 (2T 2 3p3 )y 2 + (3p4 3p2 T 2 + T 4 )y p4 T 2 + p6 + p2 T 3 r = 0 nh n ya , yb , yc l nghi m. V y ya yb yc = p4 T 2 p6 p2 T 3 r. Thay T = 4R + r = ra + rb + rc , p2 =

ra rb rc ta c r

2 2 2 2 2 2 (rb + rb rc + rc )(rc + rc ra + ra )(ra + ra rb + rb )

= (

ra rb rc 3 ra rb rc 2 ) +( ) (ra + rb + rc )2 ra rb rc (ra + rb + rc )3 . r r

V d 3. Ch ng minh r ng trong tam gic ABC ta c 11R + 2r 3 19R2 + 4Rr + 4p2 3


3

R3 + R2 r + 4Rp2 + 8rp2

L i gi i. G i da , db , dc l kho ng cch t tm ng trn ngo i ti p n tm cc ng trn bng ti p cc gc A, B, C. Khi d2 = R2 + 2Rra , d2 = R2 + a b 2Rrb , d2 = R2 + 2Rrc . Do t h phng trnh c x3 (4R + r)x2 + p2 x p2 r = 0 y = R2 + 2Rx kh x ta c phng trnh y 3 (11R2 + 2Rr)y 2 + (19R4 + 4Rr + 4p2 )y (R6 + R5 r + 4R4 p2 + 8R3 rp2 ) = 0 nh n d2 , db , d2 l cc nghi m. a c p d ng k t qu (a) ta nh n c 11R + 2r 3 19R2 + 4Rr + 4p2 3
3

R3 + R2 r + 4Rp2 + 8rp2

Dy s v m t s tnh ch t

155

V d 4. Cho t gic ABCD n i ti p ng trn tm O bn knh R. K hi u r1 , r2 , r3 , r4 l n l t l bn knh cc ng trn (ABC), (BCD), (CDA), (DAB). Ch ng minh r ng: 1 ab 2Rr1 1 bc 2Rr2 1 cd 2Rr3 1 da 2Rr4 = (a + b)(b + c)(c + d)(d + a) (ac + bd)2

L i gi i. t AC = x, BD = y, a + b + c + d = 2p. T phng trnh nh n ba c nh c a tam gic ABC l nghi m (theo b1) ta a+b c abx = (a + b + x)2Rr1 . Do x = . ab 1 2Rr1 b+c d+a c+d ,y = ,y = Tng t x = cd bc da 1 1 1 2Rr3 2Rr2 2Rr4 T h th c cho t gic n i ti p (ac + bd)2 = x2 y 2 ta c (ac + bd)2 = a+b c+d b+c d+a hay ab cd bc da 1 1 1 1 2Rr1 2Rr3 2Rr2 2Rr4 1 ab 2Rr1 1 bc 2Rr2 1 cd 2Rr3 1 da 2Rr4 = (a + b)(b + c)(c + d)(d + a) . (ac + bd)2

2.7.4

Bi t p p d ng

Bi t p 1. Ch ng minh r ng trong tam gic ABC ta c (a2 + 2Rr)(b2 + 2Rr)(c2 + 2Rr) = 2Rr(ab + bc + ca 2Rr)2 .

Bi t p 2. Cho t gic ABCD n i ti p ng trn tm O bn knh R. K hi u r1 , r2 , r3 , r4 l n l t l bn knh cc ng trn (ABC), (BCD), (CDA), (DAB). Ch ng minh r ng: r1 r3 bc + ad a) = r2 r4 ab + cd b) r1 + r3 = r2 + r4 . Ti li u tham kh o 1. m Vn Nh . K t th c, php kh v ng d ng, NXB Gio d c.

Dy s v m t s tnh ch t

156

2.8

Dy s v m t s tnh ch t
V Th Vn, Tr ng THPT Chuyn B c Giang

2.8.1

M t s phng php th ng dng

Phng php sai phn


Trong m t s bi ton ta th ng ph i x l cc t ng lin quan n cc s h ng c a m t dy s cho tr c. th c hi n yu c u c a u bi, ta th ng ph i nh gi ho c rt g n m t t ng. Phng php sai phn t ra r t hi u qu khi gi i quy t cc v n ny. Sau y l m t s ki u p d ng tnh S(n) = a1 + a2 + + an , ta tm f (n) sao cho an = f (n + 1) f (n). Khi Sn = f (n + 1) f (1). Cch lm ny g i l phng php sai phn h u h n (tch s h ng t ng qut). C th d on hm f (n) b ng s d ng tch phn. Ta bi t tch phn c a a th c b c k l a th c b c k + 1, b i v y, n u f (k) := f (k + 1) f (k) = nk th f (k) ph i c b c k + 1. Cc hm r i r c khng c nguyn hm, ta c th nh gi t ng b ng cc b t ng th c tch phn. V d 1. Cho dy s (xn ) xc nh b i x1 = Tnh [S2010 ] bi t S2010 = 1 ; xn+1 = x2 + xn , n 1. n 2

1 1 1 + + + . x1 + 1 x2 + 1 x2010 + 1

H ng d n. T cng th c truy h i ta c 1 xn+1 Do d S2010 = 2 1 = 1 1 1 1 1 = xn xn + 1 xn + 1 xn xn+1 3 < 2. M t khc d th y (xn ) tng, x2 = , 4

x2011 21 x3 = > 1 suy ra xn > 1, n 3 hay S2010 > 1. V y [S2010 ] = 1. 16 Nh n xt. Ta c th t ng qut bi ny d i d ng: 1) Ch ng minh r ng [Sn ] = 1, n 3, ho c 2) Ch ng minh r ng lim Sn = 1.

V d 2. Cho dy s (xn ) xc nh nh sau x2 x1 = 1; xn+1 = n + xn , n 1, v i a l s dng cho tr c. a x1 x2 xn t un = + + + . Ch ng minh r ng lim un = a. x1+1 x2+1 xn+1

Dy s v m t s tnh ch t H ng d n. T cng th c truy h i, ta c xn =a xn+1 suy ra un = a 1 1 x1 xn+1 =a 1 1 xn+1 . 1 1 xn xn+1

157

D th y (xn ) tng. N u xn b ch n trn th t n t i lim xn = A, v i A h u h n. 1 Khi , m t m t do (xn ) tng v x1 = 1, x2 = +1 > 1 nn A > 1. M t khc, a A2 + A A = 0. chuy n qua gi i h n trong cng th c truy h i ta c A = a Mu thu n. Nh v y, dy s (xn ) l dy tng nhng khng b ch n trn nn 1 lim xn = hay lim = 0. T suy ra i u ph i ch ng minh. xn Nh n xt. Ta c th thay a b ng b t k m t s c th no, ta c k t qu tng ng, ch ng h n a = 2010. Ho c ta c th a ra yu c u ch ng minh un < a, n.

Php th l ng gic
Nhi u dy s v i cng th c ph c t p c th tr thnh cc dy s n gi n nh php th l ng gic. T chng ta c th kh o st c cc tnh ch t c bi t c a dy s , c bi t trong vi c xt tnh tu n hon. Yu c u c a k thu t ny, tr c h t v m t ki n th c chng ta c n n m c cc cng th c l ng gic, tnh ch t c a cc hm s l ng gic. Ngoi ra, k thu t ny i khi cn i h i m t cht nh y c m ton h c. D u hi u ta c th ngh n phng php ny l: trong bi ton c cng th c g i nh n cng th c l ng gic, gi thi t ho c k t lu n gi ng v i tnh ch t hm l ng gic nh tnh b ch n hay tnh tu n hon. V d 3. Cho dy s (xn ) tho mn |x1 | < 1; 2xn+1 = 3 3x2 xn , n 1. n

a) Tm i u ki n cho x1 t t c cc s h ng c a dy s u dng? b) Dy s trn c tu n hon khng? H ng d n. T gi thi t |x1 | < 1 v cng th c truy h i cho d i d ng hm 1 s xn+1 = f (xn ) v i f (x) = 3 3x2 xn , g i cho ta ngh n php th n 2 l ng gic v i cch t x1 = cos , (0; ). 1 2 3 sin cos = cos . Khi x2 = 2 3 2n B ng quy n p, ta tm c xn+1 = cos . 3

Dy s v m t s tnh ch t

158

Nh v y, dy s tr nn n gi n hn r t nhi u, m i s h ng c a dy s l m t hm s l ng gic ph thu c n. 2n a) xn > 0, n cos > 0, n. T tm c suy ra x1 . 3 b) Dy cho tu n hon. V d 4. Hai dy s (an ), (bn ) c xc nh b i a1 = , b1 = , an+1 = an bn ; bn+1 = an + bn , n 1. C bao nhiu c p (, ) tho mn a2010 = b1 ; b2010 = a1 . H ng d n. T gi thi t, thu c k t qu : a2 + b2 = (2 + 2 )n+1 . n+1 n+1 T i u ki n a2010 = b1 ; b2010 = a1 suy ra 2 + 2 = 1. t = cos , = sin , ( [0; 2]). Theo quy n p, ta c an = cos(n); bn = sin(n). Sau tm sao cho cos(2010) = sin sin(2010) = cos p s = .

(4k + 1) tng ng c 2011 b (; ). 2.2011

S p x p l i th t
Th thu t ny th ng c p d ng trong cc bon lin quan n b t ng th c trong dy s . Khi cc s c th t th chng c nh ng tnh ch t c bi t m m t dy b t k khng c. V d 5. T n t i hay khng m t dy s th c xn tho mn i u ki n a) |xn | 0.666, n N ; 1 1 b) |xm xn | + , m = n. n(n + 1) m(m + 1) H ng d n. Gi s t n t i dy s nh v y. V i m i s nguyn dng N , ta s p l i cc s x1 , ..., xN theo th t tng d n xi1 xi2 xiN . Khi |xiN xi1 | = xiN xi(N 1) + + |xi2 xi1 | 1 iN (iN + 1)
N

1 i(N 1) (i(N 1) + 1)

1 1 + i2 (i2 + 1) i1 (i1 + 1)

=2
k=1

1 1 1 = A(N ). ik (ik + 1) iN (iN + 1) i1 (i1 + 1)

Dy s v m t s tnh ch t V i1, i2, . . . , iN l m t hon v c a 1, 2, ..., N nn


N

159

A(N ) = 2
k=1

1 1 1 k(k + 1) iN (iN + 1) i1 (i1 + 1) 1 iN (iN + 1) i1 (i1 + 1) 1 1 4 2 = . 1.2 2.3 3 N +1 1

1 N +1 1 2 1 N +1 =2 1

4 M t khc |xiN xi1 | |xiN | + |xi1 | 2.0, 666 < . 3 4 2 Ch n N l n sao cho > 2.0, 666. Mu thu n. V y khng t n 3 N +1 t i dy s tho mn yu c u.

Phng php quy n p


i v i bi ton ch ng minh tnh ch t c a dy s m tnh ch t l m nh P (n), ta c n ph i ch ng P (n) ng v i n n0 , trong n0 l s t nhin cho tr c, th phng php ch ng minh b ng quy n p l i r t hi u qu . V d 6. Cho dy s (an ) xc nh: a1 = 1; a2 = 1; an+1 = an 2an1 , n 2. Ch ng minh r ng v i n 2 ta lun c 2n+1 7a2 = (2an + an1 )2 . n1 H ng d n. S d ng phng php quy n p. V i n = 2 ta c 23 7 = (2 + 1)2 , suy ra (2.1) ng v i n = 2. Gi s (2.1) ng v i k n, y n 2. Khi : (2an+1 + an )2 = (2an 4an1 + an )2 = (an 4an1 )2 = a2 + 8an an1 + 16a2 n n1 2 = 2(4an + 4an an1 + a2 ) + 14a2 7a2 n1 n1 n 2 2 2 = 2(2an + an1 ) + 14an1 7an = 2(2n+1 7a2 ) + 14a2 7a2 n1 n1 n n 2 = 2 7an . Suy ra (2.1) ng v i k = n + 1. i u ph i ch ng minh (pcm). Ch . K t lu n c a bi ton trn c th pht bi u: Ch ng minh r ng n 2 ta lun c 2n+1 7a2 l m t s chnh phng. n1 V d 7. Cho x1 , x2 , . . . , xn l nh ng s th c dng. Ch ng minh r ng n u x1 x2 xn = 1 th x1 + x2 + . . . + xn n v i n = 1, 2, . . . (2.1)

Dy s v m t s tnh ch t

160

H ng d n. p d ng phng php quy n p. V i n = 2, ta c n ch ng minh x1 x2 = 1 suy ra x1 + x2 2. Th t v y, ta c 1 (x1 1)2 0 x2 + 1 2x1 x1 + 2 x1 + x2 2. 1 x1 ng th c x y ra khi x1 = 1 hay x1 = x2 = 1. Gi s m nh ng v i n 2. Ta s ch ng minh ng cho n + 1, ngha l ch ng minh t x1 x2 xn xn+1 = 1 (2.2) suy ra x1 + x2 + . . . + xn + xn+1 n + 1. T (2.2) cho ta hai tr ng h p: T t c cc s u b ng nhau x1 = x2 = = xn+1 = 1. Khi x1 + x2 + . . . + xn + xn+1 = n + 1. Khng ph i cc s u b ng nhau. Trong cc s c s l n hn 1, th cng c s nh hn 1. Ch ng h n x1 < 1, xn+1 > 1. Khi ta c y1 x2 xn = 1 v i y1 = x1 xn+1 . Do gi thi t quy n p ng v i n, nn ta c y1 + x2 + ... + xn n. Khi x1 + . . . + xn+1 = y1 + x2 + . . . + xn + xn+1 y1 + x1 n + xn+1 y1 + x1 = (n + 1) + xn+1 y1 + x1 1 = (n + 1) + xn+1 x1 xn+1 + x1 1 = (n + 1) + (xn+1 1)(1 x1 ) n + 1 do x1 < 1, xn+1 > 1. pcm (2.3)

S d ng nh l v gi i h n tng ng
nh l 1. Cho dy s (ck ) v i 0 < ck < 1, k = 1, 2, 3, ... Xt cc dy s
n n

xn =
i=1

(1 + ci ) ; yn =
i=1

(1 ci ).

Khi ba ng th c sau l tng ng


n+

lim xn = +
n+ n

(2.4) (2.5) (2.6)

lim yn = 0 ci
i=1

n+

lim

= +

Dy s v m t s tnh ch t Ch ng minh. Ch ng minh (2.4) (2.6).


n

161

Gi s
i=1

ci < M v i 0 < M < +. Khi


n

(1 + ci ) <
i=1 n+

1 1+ n

ci
i=1

<

M 1+ n

< eM ,

v l v lim xn = +, hay (2.6) c ch ng minh. Ch ng minh (2.6) (2.4).


n n

i u ny l hi n nhin v
i=1

(1 + ci ) >
i=1

ci .

Ch ng minh (2.5) (2.6). Nh n xt r ng ng v i b n s b t k a1 , a2 , ..., an v i 0 < ai < 1 b ng quy n p ta d dng ch ng minh c


n n

ai > 1
i=1 n i=1

(1 ai ).

Do lim

n+

(1 ci ) = 0 nn ng v i m i m lun t n t i n > m sao cho


i=1

1 (1 ci ) < . 2 i=1 T ta c
n

ci > 1
i=1 n

1 (1 ci ) > . 2 i=1

Suy ra lim

n+

ci = +.
i=1

Ch ng minh (2.4) (2.5). Ta c


n n n

1>
i=1 n

(1

c2 ) i

=
i=1

(1 + ci )
i=1

(1 ci ).

Nhng v lim

n+

(1 + ci ) = + nn theo nguyn l k p ta c
i=1 n n+

lim

(1 ci ) = 0
i=1

(pcm.)

V d 8. Cho dy s th c dng tng (un ) th a mn lim un = +. Ch ng


n+

minh r ng t n t i k N sao cho u1 u2 uk + + + < k 2010. u2 u3 uk+1

(2.7)

Dy s v m t s tnh ch t H ng d n. (2.7) k u1 u2 uk + + + u2 u3 uk+1


k

162

> 2010
i=1

ui ui+1

> 2010.

Do (un ) l dy tng nn 0 < 1 M t khc, ta c


n

ui ui < 1. t ci = 1 , suy ra 0 < ci < 1. ui+1 ui+1


n

(1 ci ) =
i=1 i=1

ui u1 = ui+1 un+1
n

ti n d n t i 0 khi n +. V y lim
k

n+

ci = +. Do t n t i k N
i=1 k

1
i=1

ui ui+1

=
i=1

ci > 2010.

Nh n xt: Ta c th thay s 2010 b ng m t s th c dng b t k m khng thay i l i gi i. Th c ch t bi ton ny ch ng minh


n n+

lim

1
i=1

ui ui+1

= +.

V d 9. Cho dy s th c dng (un ) th a mn lim un = +. Ch ng minh


n+

r ng v i a > 0 cho tr c, t n t i k N sao cho


k

i=1

ui > a. u1 + u2 + + ui

(2.8)

H ng d n. t ci =

ui . V ui > 0 nn 0 < ci < 1 v u1 + u2 + + ui u1 + u2 + + ui1 , v i i 2. u1 + u2 + + ui u1 u1 + u2 + + un


n+

1 c1 = Suy ra
n

(1 ci ) =
i=1

ti n d n t i 0 khi n +, v ui > 0 v lim un = +.


n n

Do lim

n+

ci = + hay t n t i k N
i=1 i=1

ci > a. pcm

Nh n xt. Ta c th cho a m t gi tr dng c th no , ch ng h n cho a = 2010, l i gi i khng thay i, b i v b n ch t c a n l


n n+

lim

i=1

ui = +. u1 + u2 + + ui

Dy s v m t s tnh ch t

163

2.8.2

Ch ng minh tnh ch t c a dy s

Tnh ch t c a dy s r t a d ng, c th l tnh ch t v s h c, v gi i tch, lin quan n ng th c, b t ng th c, tnh tu n hon... V i nh ng bi ch ng minh tnh ch t c a dy s , ta th ng p d ng m t s phng php trnh by trong ph n tr c. Ngoi ra, ta cng hay s d ng cc phng php khc nh ph n ch ng, tuy n tnh ha phng trnh sai phn,...

Tnh ch t s h c Gi i thi u v tnh ch t s h c


S h c lin quan n s nguyn. Trong dy s , c th c t t c cc s h ng u l s nguyn (dy s nguyn), ho c m t s s h ng nguyn. Tnh ch t s h c c a dy s th ng lin quan n s h ng l s nguyn trong dy s , tnh ch t c th hi n tnh ch t chia h t, nguyn t cng nhau, chnh phng, ng d,... V d 10. Xt dy s (xn ) xc nh nh sau x1 = x2 = 1, xn+2 = 14xn+1 xn 4. Ch ng minh r ng v i m i n, xn l s chnh phng. H ng d n. Xt dy s u1 = u2 = 1, un+2 = 4un+1 un . Ta c un+2 un u2 = (4un+1 un )un u2 = un+1 (4un un+1 ) u2 n+1 n+1 n 2 2 = un+1 un1 un = = u3 u1 u2 = 2, n. Ti p theo, ta ch ng minh xn = u2 b ng phng php quy n p. n 2 Th t v y, ta c x1 = 1 u1 , x2 = 1 u2 . 2 Gi s xk = u2 v i m i k n. Ta ch ng minh xn+1 = u2 . n+1 k Th t v y, ta c xn+1 = 14xn xn1 4 = 14u2 u2 2(un1 un+1 u2 ) n n1 n = 16u2 8un1 un + u2 = (4u2 un1 )2 = u2 . n n1 n n+1 2 Nh v y ta c xn = un v i m i n. pcm (2.9)

Dy s nguyn
Nh ni trn, dy s nguyn c lin quan r t nhi u n tnh ch t s h c c a dy s . Trong nhi u tr ng h p dy s ch l b ngoi, b n ch t bi ton l i l bi ton s h c. Chnh v v y, ngoi nh ng ki n th c v dy s , chng ta c n bi t thm nh ng k t qu c a s h c v tnh ch t chia h t, c s chung l n nh t, b i s chung nh nh t, ng d, s nguyn t , h p s ,... Ch ng h n, nguyn l Diriclet, l nguyn l r t n gi n nhng l i v cng h u hi u trong cc bi ch ng minh, c bi t l ch ng minh s t n t i hay khng t n t i c a m t i t ng tho mn m t i u ki n no .

Dy s v m t s tnh ch t

164

V d 11. Cho dy s nguyn g m n + 1 s : a1 , a2 , ..., an+1 , cc s h ng c a n n m trong o n [1; 2n]. Ch ng minh r ng t n t i hai s h ng c a dy s trn sao cho s ny chia h t cho s kia. H ng d n Do cc s ai u l cc s nguyn dng nn chng u c th d ng ai = 2si ri , v i ri l s l . Cc s ri ch c th nh n n gi tr V dy (an ) c n + 1 s nn theo nguyn l Diriclet t n t i 2 s cho rk = rj hay dy s cho t n t i hai s ak , aj sao cho m chia h t cho s cn l i.

vi t c d i 1, 3, ..., 2n 1. k, j, k = j sao t trong hai s

V d 12. Cho dy s nguyn g m n s : a1 , a2 , ..., an , cc s h ng c a n n m trong o n [1; 2n], tho mn [ai , aj ] > 2n v i m i i = j. Ch ng minh r ng m i 2n . s h ng c a dy s u l n hn 3 H ng d n Khng lm m t tnh t ng qut, gi s a1 l s nh nh t c a dy s cho. 2n 2n . Th t v y, gi s a1 < . Ta c n + 1 s Ta ch c n ch ng minh a1 > 3 3 2a1 , 3a1 , a2 , ..., an u khng l n hn 2n v khng c s no l b i c a s no do [ai , aj ] > 2n v i m i i = j. i u mu thu n v i k t qu bi ton trn. Nh n xt. Khi n m t s c th , ta c bi ton tng ng. Ch ng h n, cho n = 2010, ta c th pht bi u bi ton sau. " Cho 2010 s nguyn dng khng l n hn 4020, tho mn b i s chung nh nh t c a hai s b t k lun l n hn 4020. Ch ng minh r ng m i s trong 2010 s u l n hn 1340". M t dy s truy h i tuy n tnh v i h s nguyn v cc s h ng u u nguyn th dy s l dy s nguyn. Nhng c dy s m cng th c truy h i phi tuy n tnh (c phn th c, c cn th c) m cc s h ng c a n v n nguyn. V d 13. Ch ng minh r ng, m i s h ng c a dy s (an ) xc nh b i a0 = 1; an+1 = 2an + 3a2 2 u nguyn. n H ng d n. Chuy n v bnh phng cng th c truy h i c a dy s ta c a2 4an an+1 + 4a2 = 3a2 2 n+1 n n a2 4an an+1 + a2 + 2 = 0. n+1 n Thay n b ng n + 1 ta c a2 4an an1 + a2 + 2 = 0. n n1 Suy ra an1 , an+1 l nghi m c a phng trnh x2 4an x + a2 + 2 = 0. Theo n vieet ta c an+1 + an1 = 4an hay an+1 = 4an an1 , m a0 = 1, a1 = 3 l cc s nguyn. V y, theo quy n p t t c cc s h ng trong dy u l cc s nguyn.

Dy s v m t s tnh ch t V d 14. Cho dy s (an ) xc nh nh sau a1 = a2 = 97, an+1 = an an1 + (a2 1)(a2 1). n1 n Ch ng minh r ng v i m i n, s 2 + 2 + 2an l s chnh phng.

165

(2.10)

H ng d n. Tr c h t ta ch ra an Z. Ti p theo, ta ch ng minh 2(1 + an ) l s chnh phng. V cu i cng l a ra i u ph i ch ng minh. V i x N khng chnh phng b t k, ta lun c x = k 2 l, v i l l c c a x v l khng chnh phng. Ta g i l l ph n khng chnh phng c a x. Ta c a3 = 2.972 1. T cng th c truy h i suy ra an+1 an an1 = (a2 1)(a2 1) n1 n

a2 2an+1 an an1 + a2 a2 = a2 a2 a2 a2 + 1 n+1 n n1 n n1 n n1 a2 + a2 + a2 = 2an+1 an an1 + 1. n+1 n n1 T (2.11) thay n l n l t b ng m + 2 v m + 1 ta c a2 + a2 + a2 = 2am+2 am+3 am+1 + 1. m+3 m+2 m+1 a2 + a2 + a2 = 2am+2 am am+1 + 1 m+2 m+1 m Tr v v i v c a (2.12) v (2.13) ta c a2 a2 = am+2 am+1 (am+3 am ) m+3 m am+3 + am = 2am+1 am+2 (t gi thi t d suy ra dy s (an ) tng nn am+3 am > 0). V a1 , a2 , a3 N an N . T (2.12) ta c am+3 (am+3 2am+1 am+2 ) + a2 + a2 1 = 0 m+2 m+1 am+3 am + 1 = a2 + a2 . m+2 m+1 T (2.14) v (2.15) suy ra (am+3 + 1)(am + 1) = (am+2 + am+1 )2 . (2.16) (2.15) (2.14) (2.12) (2.13) (2.11)

Nh v y tch c a (am+3 + 1) v (am + 1) lun l m t s chnh phng. V v y ph n khng chnh phng c a (am+3 + 1) v (am + 1) l b ng nhau. Ch l ph n khng chnh phng c a a1 + 1, a2 + 1, a3 + 1 l 2, do b ng quy n p ta thu c ph n khng chnh phng c a an + 1 l 2 v i n N . V y 2 an + 1 = 2kn . T (2.16) ta c
2 2 km+3 km = km+1 + km+2 1.

(2.17)

M t khc, tr v v i v c a (2.14) v (2.15) ta thu c (am+3 1)(am 1) = (am+2 am+1 )2 .

Dy s v m t s tnh ch t

166

2 L p lu n nh trn ta c an 1 = 6ln (do a1 1, a2 1, a3 1 c ph n khng chnh phng l 6). Khi , ta c 2 2 2 6lm + 2 = 2km (km + 1)(km 1) = 3lm .

(2.18)

B ng quy n p k t h p v i (2.14) ta d dng ch ng minh c v i n, an u c d ng 4k + 1. Suy ra km l s l , nn (km + 1) v (km 1) u c c l 2. T (2.18) suy ra m t trong 2 s (km + 1) v (km 1) c d ng 2b2 s cn l i l 6c2 . L i t (2.17) b ng quy n p ta th y km lun c d ng 6q + 1 nn km + 1 ph i c d ng 2b2 . V v y 2 + 2 + 2am = 2 + 2.2km = 2 + 2km = 2.2b2 = 4b2 . pcm R rng, v i v d ny, u tin ta ph i ch ng minh dy s (an ) l dy s nguyn, sau ch ng minh 2 + 2an l s chnh phng r i m i ch ng minh yu c u bi ton. y, ta ph i s d ng nhi u tnh ch t s h c nh tnh ch t chia h t, chia c d, tnh ch t v s chnh phng,... Nh v y, ta c th xy d ng dy s nguyn t l i gi i c a phng trnh nghi m nguyn. Ch ng h n, t phng trnh Pell x2 Dy 2 = k, gi s n c nghi m khng t m th ng (x0 ; y0 ), a, b l nghi m c s c a phng trnh lin k t v i n x2 Dy 2 = 1. Khi hai dy s (xn ), (yn ) xc nh b i xn+1 = axn + bDyn , yn+1 = bxn + ayn th (xn ), (yn ) l nghi m c a phng trnh x2 Dy 2 = k. T ta c th tm c xn+1 = axn + b D(x2 k); n
2 yn+1 = ayn + b Dyn + k

v nh v y xu t hi n hai dy s nguyn c cho b i cng th c khng nguyn. M t khc, ta cng c th t o ra dy s nguyn xc nh b i cng th c truy h i m c phn s t phng trnh b c hai: x2 4an x + a2 + 2 = 0 nh sau. n a2 + 2 2 Theo nh l Vieet th an+1 an1 = an + 2, nn an+1 = n . Khi ta c an1 a2 + 2 . Ch ng bi ton: "Cho dy s (an ) xc nh b i a0 = 1, a1 = 3, an+1 = n an1 minh r ng an nguyn v i m i n". gi i bi ton ny ta c th dng phng php tuy n tnh ho phng trnh sai phn. V d 15. (Bulgari 1978) Cho dy s (an ) th a mn an+1 = Ch ng minh r ng, n u a0 , a1 v v i m i n. H ng d n. T cng th c truy h i ta c an+1 an1 a2 = c. n a2 + a2 + c 1 T suy ra an+1 = 0 an an1 . a0 a1 a2 + c n . an1

a2 + a2 + c 0 1 l cc s nguyn th an nguyn a0 a1

Dy s v m t s tnh ch t

167

Bi t p
Bi t p 10. Cho dy s (un ) xc nh nh sau u1 = ; u2 = , un+1 = aun un1 . Ch ng minh r ng un+1 un1 u2 = a 2 2 , n 2. n Bi t p 11. Cho dy s (xn ) xc nh nh sau x1 = 2 ; u2 = 2 , xn+1 = (a2 + 2)xn xn1 2(a 2 2 ). Ch ng minh r ng xn l s chnh phng v i m i n. Bi t p 12. Cho dy s (an ) xc nh nh sau a1 = 1; a2 = 2, a3 = 24, an = 6a2 an3 8an1 a2 n1 n2 . an2 an3

Ch ng minh r ng, m i n, an nguyn v an chia h t cho n. H ng d n. T cng th c truy h i ta c an an1 an2 =6 8 . an1 an2 an3 an+1 t bn = , ta c b1 = 2; b2 = 12; bn = 6bn1 8bn2 . Gi i phng trnh an sai phn tuy n tnh ny, ta c bn = 4n 2n . . . . Khi ta c an+1 = (4n 2n )an = 2n (2n 1)an . R rng a1 . a2 . a3 . .1, .2, .3. . . Gi s an . n 3, ta s ch ng minh an+1 . + 1). Th t v y, ta c .n, .(n . N u (n + 1) l s nguyn t th (2n 1). + 1) ( nh l Fermat) nn .(n . an+1 . + 1). .(n N u (n + 1) l h p s th (n + 1) phn tch thnh tch cc s nguyn t p nh hn n. . . . M t khc an+1 = 2n (2n 1)an , an . an+1 . k n an+1 . v i p .n .k, .p, . nguyn t nh hn b ng n. Hay an+1 . + 1). pcm .(n Bi t p 13. Cho k N . Cho dy s (an ) xc nh nh sau a1 = k + 1; an+1 = a2 kan + k, n N . n Ch ng minh r ng m = n, (am , an ) = 1. H ng d n. Ta ch c n ch ng minh v i m > n, m > n ta c: (am , an ) = 1. Th t v y, v i

am k = a2 kam1 = am1 (am1 k) m1 = am1 am2 ...an (an k) = am1 am2 ...a1 . R rng (am , a1 ) = 1. Gi s (am , ak ) = 1, 1 k n 1 < m 2. Ta s ch ng minh (am , an ) = 1. Gi s (am , an ) = d. . . . . . Ta c am . an . k . an k . an1 an2 ...a1 . theo gi thi t .d, .d .d .d .d, (am , ak ) = 1k n 1 (am , an1 an2 ...a1 ) = 1 d = 1.

Dy s v m t s tnh ch t

168

Tnh ch t gi i tch
Tnh ch t gi i tch c a dy s c th hi n tnh b ch n, tnh h i t hay phn k, tnh tng gi m, quy lu t c a dy s nh c p s c ng, c p s nhn,...Ngoi ra tnh ch t gi i tch c a dy cn th hi n cc bi u th c ch a cc s h ng c a dy s . V d 16. Cho dy s (un ) tho mn i u ki n u1 = u2 = 1, un+1 = 3un un1 , n 2. t Sn =
n i=1

arccot ui . Ch ng minh Sn h i t . Tm gi i h n c a Sn .

H ng d n. G i (fn ) l dy Phibonacci tho mn f1 = f2 = 1, fn+1 = fn + fn1 , n 2. Ta s ch ng minh b ng quy n p un = f2n3 . (2.19)

Ta c u3 = 2 = 1 + 1 = f1 + f2 = f3 , (2.19) ng v i n = 3. Gi s (2.19) ng v i k n. Ta ph i ch ng minh, (2.19) ng v i n + 1. Th t v y, ta c un+1 = 3un un1 = 3f2n3 f2n5 = 2f2n3 + f2n4 = f2n3 + f2n2 = f2n1 . 2 M dy Phibonacci (fn ) c tnh ch t fn = fn+1 fn1 + (1)n1 (d dng ch ng minh b ng quy n p). f2k2 f2k4 + 1 2 . T f2k3 = f2k2 f2k4 + (1)2k4 , k 3 hay f2k3 = f2k2 f2k4 Suy ra arccot uk = arccot f2k3 = arccot f2k2 f2k4 + 1 f2k2 f2k4 = arccot f2k4 arccot f2k2 , k 3. 3 3 arccot f2n2 . V y lim Sn = . 4 4

Sn =

n i=1

arccot ui =

V d 17. Cho dy s (un ) tho mn i u ki n 5 u0 = 2; u1 = , un+1 = un (u2 2) u1 , n 1. n1 2 CMR v i n 1, [un ] = 2 H ng d n. 2k (1)k , k 0. Ta ch ng minh an+1 = 2an + (1)n . 3 V a0 = 0; a1 = 1 nn ak u nguyn. Ta t ak =
2n (1)n 3

Dy s v m t s tnh ch t Ta ch ng minh b ng quy n p cng th c sau un = 2an + 2an , n 0.

169

(2.20)

V i k = 0, 1 d th y. Gi s (2.20) ng v i k = n 1 v k = n. Ta ph i ch ng minh (2.20) ng v i k = n + 1. Th t v y, ta c un+1 = un (u2 2) n1 5 2 5 2 5 2


n

= (2an + 2an )[(2an1 + 2an1 )2 2] = (2an + 2an )(22an1 + 22an1 ) = (2an + 2an )(2an (1) = 22an +(1) + 2(1) = 2an+1 + 2an+1 .
n n1 n1

5 2 ) 5 2

+ 2(1)

n1 a n

+ 2(1)

n1

+ 22an (1)

Nh v y, ta c n 1, [un ] = [2an + 2an ] = 2an ,do 0 < 2an < 1, n 0. V d 18. Ch ng minh r ng n u (an ) l dy s dng tng ch t v khng b a2 a1 a3 a2 an an1 ch n th dy + + + khng b ch n. a2 a3 an H ng d n. Tr c h t ta ch ng minh B . T n t i dy s nguyn dng 1 = k1 < k2 < < kn < th a mn
n

i=1

aki aki1 . aki

(2.21)

khng b ch n. Th t v y, chng ta c th xy d ng dy (kn ) th a mn aki aki1 2 ak i 3

nh sau. Gi s xy d ng c 1 = k1 < k2 < < kn1 . V ak khng b ch n nn lun t n t i mt s kn th a mn akn 3akn1 suy ra akn akn1 2 . akn 3 2 T m i s h ng trong chu i (2.21) lun l n hn ho c b ng . T ta 3 c chu i (2.21) khng b ch n. By gi , ta c ak aki+1 1 aki +1 aki aki +2 aki +1 + + + i+1 aki +1 aki +2 aki+1 ak aki+1 1 ak aki ak +1 aki i + + i+1 i+1 . aki+1 aki+1 aki+1 Nh v y chu i cho l n hn chu i (2.21). pcm

Dy s v m t s tnh ch t

170

Tnh tu n hon c a dy s
nh ngha 1. Dy s (un ) c g i l dy s tu n hon (c ng tnh) n u t n t i s N sao cho un+s = un , n N. S s nh v y c g i l chu k c a dy. Chu k nguyn dng nh nh t c a m t dy s tu n hon c g i l chu k c s dy s . Trong th c hnh, ch ng minh m t dy s l tu n hon khng nh t thi t ph i xc nh chu k c s c a n. M t dy s tu n hon v i chu k 1 l dy h ng. nh l 2. ( V tnh tu n hon c a dy s d ) Cho m, k N , m 2 v dy s nguyn (an ) tho mn an+k = c1 an+k1 + c2 an+k2 + + ck an + ck+1 , n N . Trong a1 , a2 , ..., ak , c1 , c2 , ..., ck , ck+1 l nh ng s nguyn. G i rn l s d trong php chia an cho m. Khi , n u (ck , m) = 1 th dy (rn ) trn tu n hon. V d 19. Cho dy s (un ) xc nh nh sau u1 = 2, un+1 = 2 + un , n 1. 1 2un

Ch ng minh r ng a) un = 0 v i m i n nguyn dng. b) Dy khng tu n hon. H ng d n. Nh n xt: t cng th c truy h i c a dy, ta lin t ng n cng th c c ng c a hm s l ng gic tang, v yu c u bi ton c lin quan n tnh tu n hon. V th , c th ngh n vi c dng n php th l ng gic. G i l gc sao cho tan = 2. 2 tan = tan 2. Ta c u1 = tan , u2 = 1 tan 2 Theo quy n p, ta ch ng minh c un = tan n. 2un a) T cng th c tnh un ta suy ra u2n = . 1 u2 n T suy ra n u t n t i n un = 0 th s t n t i n l un = 0. Gi s u2k+1 = 0, khi u2k = 2 v ta c 2uk 2 = u2k = u2 + uk 1 = 0 uk v t , trong khi theo cng k 1 u2 k th c truy h i cho trong gi thi t th uk lun h u t . Nh v y, un = 0, n 1. b) Gi s dy tu n hon. Khi t n t i hai s nguyn dng n v k, (n > k) sao cho un = uk hay tan n = tan k (n k) = m unk = 0. Mu thu n v i k t qu ph n a).

Dy s v m t s tnh ch t V d 20. Cho dy s (xn ) xc nh nh sau x1 = 19, x2 = 5, x3 = 1890, xn+3 = 30xn+2 + 4xn+1 + 2011xn + 1664, n 1. Ch ng minh r ng dy s (xn ) c v s s h ng chia h t cho 2010.

171

H ng d n. G i rn l s d trong php chia xn cho 2010. Ta c (2010, 2011) = 1 nn dy s (rn ) tu n hon, gi s v i chu k s. Ta c k N 2011xks = xks+3 30xks+2 4xks+1 1664 x3 30x2 4x1 1664 (mod 2010) 1890 30.5 4.19 1664 (mod 2010) 0 (mod 2010). . M (2011, 2010) = 1 nn xks . .2010. . t n = ks, do c v s k N nn cng c v s n = ks N xn . .2010. V d 21. Cho dy s (an ) xc nh nh sau a2n = an , n 1, a = 1, n 1, 4n+1 a4n+3 = 0, n 0. Ch ng minh r ng dy s khng tu n hon. H ng d n. Gi s dy (an ) tu n hon, ngha l t n t i s N sao cho an+s = an , n N . N u s 0 (mod 4) th s + 1 1 (mod 4) suy ra as+1 = 1, m as+1 = a1 nn a1 = 1. M t khc, do s 0 (mod 4) nn s = 2k t, v i k, t N , k > 1, t l . Khi , s + 2k1 = 2k1 (2t + 1). T , 1 = a1 = a2k1 = as+2k1 = a2t+1 = 0 (do t l ). Mu thu n. N u s 1 (mod 4) th s + 2 3 (mod 4) suy ra as+2 = 0, m as+2 = a2 = a1 nn a1 = 0. Cng c s + 4 1 (mod 4) as+4 = 1 m a1 = a4 = as+4 a1 = 1. Mu thu n. N u s 2 (mod 4) th s + 3 1 (mod 4) suy ra 0 = a3 = as+3 = 1. Mu thu n. N u s 3 (mod 4) th s + 4 3 (mod 4) suy ra as+4 = 0, m as+4 = a4 = a2 = a1 nn a1 = 0. L i c s + 2 1 (mod 4) a1 = a2 = as+2 = 1. Mu thu n. Nh v y, gi s sai hay dy cho khng l dy tu n hon. V d 22. Cho dy s th c (an ) th a mn |an | = an+1 + an1 , n = 1, 2, 3, ... Ch ng minh r ng dy s tu n hon v i chu k 9.

Dy s v m t s tnh ch t

172

H ng d n. T cng th c truy h i ta suy ra t ng c a hai s k nhau c t nh t m t s khng m. Ta c |an | = an+1 + an1 n = 1, 2, 3, ..., suy ra trong s an+1 , an1 t nh t m t s khng m. V th t n t i hai s ng k nhau khng m, gi s l am v am+1 . Ta c am1 = am am+1 ; am+2 = am+1 am .

Suy ra am1 v am+2 i nhau hay m t trong 2 s ph i khng m. Do ta c 3 s ng k nhau u khng m. c bi t s gi a 2 s s b ng t ng c a 2 s cn l i. Ta k hi u chng l a, a + b, b (a, b > 0). Theo cng th c truy h i, n u bi t hai s h ng lin ti p th ta lun xc nh c s h ng ng li n tr c v ng li n sau 2 s . V v y, ta c: N u a b th m t ph n c a dy s l ..., b, 2b a, b, a, a + b, b, a, a b, b, 2b a, ... N u a b th m t ph n c a dy s l ..., 2a b, a, b a, b, a, a + b, b, a, a b, 2a b, a, ... i u ch ng t dy cho tu n hon v i chu k 9. Ch . Ta c th gi i bi ton ny b ng cch khc nh sau. T cng th c truy h i ta suy cng th c truy h i li an+1 = |an | an1 v cng th c truy h i ti n an1 = |an | an+1 . V i s t nhin n b t k, ta s ch ng minh an+9 = an . Th t v y, ta t an+4 = x, an+5 = y. T hai cng th c truy h i li v ti n trn ta bi u di n c an+9 v an theo x, y nh sau. an+9 = ||||y| x| y| |y| + x| ||y| x| + y; an = ||||x| y| x| |x| + y| ||x| y| + x. (2.22) (2.23)

t bi u th c bn ph i c a (2.22) l g(x, y), khi bi u th c bn ph i c a (2.23) l g(y, x). Ta ch ng minh g(x, y) = g(y, x) b ng cch xt 3 tr ng h p 0l y; x 0 y; x y 0. Ta c bi ton p d ng: Dy s (xn ) th a mn i u ki n x10 = 3; x11 = 2; xn+1 = |xn | xn1 , n 12. Hy tnh S := x2 + x2 + + x2 . 2001 2002 2010 H ng d n. Xt dy s yn = xn+10 . R rng dy (yn ) tu n hon v i chu k 9. p s S = 87.

Dy s v ng th c
Nh ng bi ton v dy s m c n ch ng minh m t s s h ng no c a n tho mn ng th c, ta hay s d ng n phng php quy nap.

Dy s v m t s tnh ch t V d 23. Cho hai dy s (xn ), (yn ) xc nh nh sau x0 = 0, x1 = 1, xn+1 = 4xn xn1 , n 1 y0 = 1, y1 = 2, yn+1 = 4yn yn1 , n 1
2 Ch ng minh r ng m i n, yn = 3x2 + 1. n

173

H ng d n. Ta dng phng php quy n p. D ki m tra ng th c ng v i n = 0, 1. Gi s ng th c ng v i n t 1 n k, (k N ). Ta s ch ng minh ng th c ng v i n = k + 1. Th t v y, ta c


2 2 2 yk+1 = (4yk yk1 )2 = 4yk 8yk yk1 + yk1 = 3(16x2 8xk xk1 + x2 ) + 1 + 24xk xk1 yk yk1 + 16 k k1 2 = 3xk+1 + 1 + 24xk xk1 yk yk1 + 16.

M 24xk xk1 yk yk1 + 16 = 6(xk+1 + xk1 )xk1 2(yk+1 + yk1 )yk1 + 16 2 = 6xk+1 xk1 + 6x2 2yk+1 yk1 2yk1 + 16 k1 2 2 = 6(x2 1) + 6x2 2yk1 2(yk + 3) + 16 = 0 k k1
2 (do xk+1 xk1 x2 = 1; yk+1 yk1 yk = 3, theo bi ton 2.1.1). pcm k

V d 24. Cho a, A > 0 l nh ng s b t k v dy (an ) xc nh a0 = a, 1 A an+1 = an + , n 0. 2 an Ch ng minh r ng n 1, ta lun c an A a1 A = an + A a1 + A


2n1

(2.24)

H ng d n. Dng quy n p. Hi n nhin ng th c (2.24) c n ch ng minh ng v i n = 1. Gi s (2.24) ng v i n = k, (k N ). Ta s ch ng minh (2.24) ng v i n = k + 1. Th t v y, ta c 1 A ak + A ak+1 A a2 2ak A + A 2 ak k = = a2 + 2ak A + A 1 A ak+1 + A k ak + + A 2 ak 2 2k ak A a1 A = = ak + A a1 + A (theo gi thi t quy n p). pcm

Dy s v m t s tnh ch t V d 25. Cho dy Phibonacci (un ) v i xc nh u1 = 1, u2 = 1 un+2 = un+1 + un , n 1. Ch ng minh cc ng th c sau 1. u1 + u2 + + un = un+2 1, (n 2); 2. un+m = un1 um + un um+1 , (n 2, m 1); 3. u2 un1 un+1=(1)n+1 , (n 2); n 1 5 ); 4. n = un + un1 , (n 2, = 2 5. 2un un+1 = un2 , (n 3); 6. (2un un+1 + 2)2 + (un un+3 )2 = u2 , (n 1); 2n+3 7. un un+1 = un1 un+2 + (1)n , (n 1). H ng d n. Dng quy n p. Ch ng h n, ch ng minh ng th c un+m = un1 um + un um+1 (2) b ng quy n p theo m. V i m = 1 ta c un1 u1 + un u2 = un1 + un = un+1 , v v i m = 2, ta c un1 u2 + un u3 = un1 + 2un = (un1 + un ) + un = un+1 + un = un+2 . ng th c (2) u ng. Gi s v i s m no cc ng th c sau ng un+m = un1 um + un um+1 (3), un+m+1 = un1 um+1 + un um+2 (4), ta s ch ng minh ng th c sau ng un+m+2 = un1 um+2 + un um+3 . Th t v y, c ng v v i v c a (3) v (4), ta nh n c un+m + un+m+1 = un1 (um + um+1 ) + un (um+2 + um+1 ) un+m+2 = un1 um+2 + un um+3 . V d 26. Cho dy s (an ) c xc nh nh sau

174

ak = k 1, k = 1, 2, 3, 4; a2n1 = a2n2 + 2n2 , a2n = a2n5 + 2n , n 3. Ch ng minh r ng v i n N , ta c cc ng th c sau

Dy s v m t s tnh ch t 1. 1 + a2n1 = 2. 1 + a2n = 12 n1 .2 ; 7 17 n1 . .2 7

175

H ng d n. Tr c h t ta ch ng minh cc ng th c sau b ng quy n p 1. 2. 12 n 17 n1 .2 = .2 + 2n1 ; 7 7 12 n2 17 n .2 = .2 + 2n+1 . 7 7

Sau ta ch ng minh cc ng th c c a bi ton cng b ng quy n p. V i n = 1, n = 2, ki m tra tr c ti p, ta th y 2 ng th c trn ng. Gi s hai ng th c trn ng v i hai s t nhin lin ti p n 1, n. Ta s ch ng minh hai ng th c trn ng cho gi tr ti p theo n + 1. T nh ngha c a dy 1 + a2n+1 = 1 + a2n + 2n1 . Ch t i ng th c 17 n1 12 n .2 .2 . Suy ra ng (2) cho gi tr n, ta c 1 + a2n+1 = + 2n1 = 7 7 th c (1) c a bi ton ng v i gi tr n + 1. T nh ngha c a dy 1 + a2n+2 = 1 + a2n3 + 2n+1 . Ch t i ng th c 12 n2 17 n .2 .2 . Suy ra (1) cho gi tr n 1, ta c 1 + a2n+2 = + 2n+1 = 7 7 ng th c (2) c a bi ton ng v i gi tr n + 1. pcm

Dy s v b t ng th c
V i bi ton lin quan n b t ng th c th phng php quy n p v phng php sai phn t ra r t h u hi u. V d 27. Cho dy s (an ) xc nh nh sau a1 = 1 2 a2 n a n+1 = , n 1. 1 an + a2 n Ch ng minh r ng, a1 + a2 + + an < 1, n 1. H ng d n. Dng phng php sai phn. 1 1 Ta c a2 = , a3 = . T cng th c truy h i, ta c 3 7 1 1 1 =1 + . an+1 an a2 n 1 t un = . Ta c u1 = 2; u2 = 3; u3 = 7 v an

Dy s v m t s tnh ch t un+1 = u2 un + 1 un+1 1 = un (un 1) n T suy ra S = a1 + a2 + + an = 1 1 1 + + + u1 u2 un 1 1 1 1 1 1 = + + + u1 1 u2 1 u2 1 u3 1 un 1 un+1 1 1 1 = u1 1 un+1 1 1 <1 =1 un+1 1

176 1 1 1 = . un un 1 un+1 1

(do un tng, u1 = 2 suy ra un > 1, n 1). pcm Nh n xt. Ta c th ch ng minh lim n ai = 1 b ng cch ch ng minh un i=1 1 tng khng b ch n trn, t 0. un+1 1 V d 28. Cho dy s (en ) xc nh nh sau e1 = 2 1 en = 2 + n , n 2. k=2 k! Ch ng minh r ng, en < 3 1 2n1 , n 3.

H ng d n. Dng quy n p. . 231 Gi s ng th c c n ch ng minh tho mn v i n no . Ta s d ng b t ng th c (n + 1)! > 2n v i n > 1 v gi thi t quy n p ta nh n c 1 1 1 1 1 1 < (3 n1 ) + < 3 n1 + n = 3 n . pcm en+1 = en + (n + 1)! 2 (n + 1)! 2 2 2 V d 29. Cho dy s th c dng (an ) tho mn b t ng th c a2 an an+1 , n N. n Ch ng minh r ng, an < 1 , n N. n V i m i n = 3 ta c e3 < 2, 67 < 2, 75 = 3 1

H ng d n. Dng quy n p. D dng ki m tra b t ng th c ng v i n = 1, n = 2. 1 1 Gi s an < , n 2. Xt hm s f (x) = x x2 , tng trn [0; ), m n 2 1 an < nn ta c n 1 1 1 1 1 1 an+1 f (an ) < f ( ) = 2 = 2 < . pcm n n n n + 1 n (n + 1) n+1

Dy s v m t s tnh ch t V d 30. Cho dy s nguyn khng m (an ) tho mn b t ng th c an+m an + am , n, m N. Ch ng minh r ng, an < ma1 + n 1 am , n > m. m

177

H ng d n. Dng quy n p. Tr c h t b ng quy n p theo k ta s ch ng minh n an kam + anmk (), k . m Th t v y, theo gi thi t an = an+(nm) am +anm , suy ra () ng v i k = 1. Gi s () ng v i k. Ta c an kam + anmk = kam + am+nmkm kam + am + anmkm = (k + 1)am + an(k+1)m . Ti p theo, t gi thi t ta c am = a1+(m1) a1 + am1 ma1 . N u n = mk + r, (r = 1, ..., m 1) th t am ma1 , ar ra1 , ta suy ra an kam + anmk = kam + ar = nr am + ar m nm+mr nm r = am + ar = am + +am am + ar m m m nm n am + ma1 ra1 + ra1 = ma1 + 1 am . m m

V d 31. Cho dy s (xn ) xc nh nh sau x1 = 1; xn = 1 , n 2. x1 + x2 + + xn1

Ch ng minh r ng, t n t i n N sao cho x1 + x2 + + xn > 2010. H ng d n. Nh n th y (xn ) l dy dng. Ta c xn+1 = 1 1 xn xn . = = 1 x1 + x2 + + xn1 + xn 1 + x2 n + xn xn

Suy ra (xn ) l dy gi m b ch n d i b i 0. Suy ra xn h i t v a v i a tho a mn a = . Ta c a = 0. 1 + a2 Nh v y, t n t i n0 sao cho n n0 : xn+1 < 1 1 1 < . 2010 x1 + x2 + + xn1 + xn 2010

V y, t n t i n N sao cho x1 + x2 + + xn1 + xn > 2010. pcm Nh n xt: Th c ch t bi ton trn l ch ng minh (xn ) h i t v 0.

Dy s v m t s tnh ch t

178

V d 32. Cho X = (x1 , x2 , ..., xn ), n 4 l dy g m n s khng m v t ng c a chng b ng 1. 1 1. Ch ng minh r ng x1 x2 + x2 x3 + + xn x1 . 4 2. Ch ng minh r ng t n t i m t hon v Y = (y1 , y2 , ..., yn ) c a X sao cho y1 y2 + y2 y3 + + yn y1 H ng d n. Dng quy n p. a) p d ng phng php quy n p v i n ta ch ng minh (x1 + x2 + + xn )2 4(x1 x2 + x2 x3 + + xn x1 ); (2.25) 1 . n

y xi 0, n 4. Th t v y, v i n = 4, b t ng th c (2.25) tng ng v i (x1 x2 + x3 x4 )2 0. ng th c x y ra khi v ch khi x1 + x3 = x2 + x4 . Gi s (2.25) ng v i n = k v i k 4. Ta c n ch ng minh (x1 + x2 + + xk + xk+1 )2 4(x1 x2 + x2 x3 + + xk xk+1 + xk+1 x1 ). (2.26) V t ng 2 v c a (2.26) l vng trn theo ch s , ta c th gi thi t xk+1 xi , i = 1, 2, ..., k. Khi t gi thi t quy n p suy ra (x1 +x2 + +(xk +xk+1 ))2 4(x1 x2 +x2 x3 + +xk1 (xk +xk+1 )+(xk +xk+1 )x1 ). (2.27) B i v x1 x2 + x2 x3 + + xk1 (xk + xk+1 ) + (xk + xk+1 )x1 = (x1 x2 + x2 x3 + + xk xk+1 + xk+1 x1 ) + xk1 xk+1 + xk (x1 xk+1 ) v x1 xk+1 0 nn t (2.27) suy ra (2.26). T v i x1 + x2 + + xn = 1, ta nh n c ngay k t qu . b) V i hon v b t k Y = (y1 , y2 , ..., yn ) c a X ta t SY = y1 y2 + y2 y3 + + yn y1 . G i S l t ng t t c cc SY (tnh theo t t c n! hon v c a X). V i s c nh i v j, i = j s l ng c a nh ng hon v c a X, trong xi ng tr c xj (x p theo vng l p), l n(n 2)!. T y,
n n

S = n(n 2)!
i,j=1,i=j

xi xj = n(n 2)!(1
i=1 n

x2 ) k

n(n 2)!(1

1 1 ( xk )2 ) = n(n 2)!(1 ) = (n 1)!. n k=1 n S 1 = . pcm n! n

Suy ra s nh nh t trong SY khng v t qu

Dy s v m t s tnh ch t V d 33. Cho dy s (xn ) xc nh nh sau: 1 2n 3 x1 = , xn = xn1 , n 2. 2 2n Ch ng minh r ng x1 + x2 + + xn < 1, n 1.

179

H ng d n. Dng sai phn. Xt dy s yn = (2n 1)xn . Ta c 2n 3 (2n 1)(2n 3) yn1 yn = (2n 1) xn1 = .xn1 . 2n 2n 2(n 1) 1 2n 1 yn = .yn1 , n 2. Cng th c trn cng ng cho n = 1 n u ta 2n t y0 = 1. 2n yn Ta l i c yn1 yn = yn yn = = xn , n 1. V v y 2n 1 2n 1 x1 + x2 + + xn = (y0 y1 ) + (y1 y2 ) + + (yn1 yy ) = y0 yn = 1 yn < 1. pcm.

2.8.3

Bi t p

Bi t p c h ng d n gi i
Ph n ny trnh by m t s bi ton lin quan n cc tnh ch t c a dy s c km theo h ng d n gi i chi ti t, c bi t l i gi i s d ng cc phng php nu. Bi t p 14. Cho dy s nguyn dng (an ) th a mn a1 = 1; an 1 + a1 + + an1 , n 2. (2.28)

Ch ng minh r ng v i m i s t nhin c th bi u di n nh t ng c a m t vi s c ch n trong dy s trn. H ng d n. Ta s ch ng minh m i s t nhin N th a mn b t ng th c 0 < N < 1 + a1 + a2 + + an c th bi u di n nh m t t ng c a m t vi s trong dy a1 , a2 , ..., an . Th t v y, ta dng phng php quy n p. V i n = 1 m nh ng v a1 = 1, khi 0 < N < 1 + 1 nn N = 1 = a1 . Gi s m nh ng v i n = k 1, ngha l m i s t nhin N th a mn b t ng th c 0 < N < 1 + a1 + a2 + + ak c th bi u di n nh m t t ng c a m t vi s trong dy a1 , a2 , ..., ak . Ta s ch ng minh n cng ng v i n = k + 1.

Dy s v m t s tnh ch t Ta ch xt tr ng h p sau l 1 + a1 + a2 + + ak < N < 1 + a1 + a2 + + ak + ak+1 .

180

Do (2.28) ta c 0 1 + a1 + a2 + + ak ak+1 N ak+1 1 + a1 + a2 + + ak . N u N ak+1 = 0 th m nh ng v i n = k + 1. N u N ak+1 > 0 th theo gi thi t quy n p N ak+1 c th bi u di n nh t ng c a m t vi s trong a1 , a2 , ..., ak . Khi N bi u di n nh t ng trn v c ng thm ak+1 . pcm Bi t p 15. Cho dy s nguyn t (pn ) th a mn 2 = p1 < p2 < < pn < (2.29)

Ch ng minh r ng gi a hai s p1 + p2 + + pn v p1 + p2 + + pn + pn+1 lun c m t s chnh phng. H ng d n. Tr c h t, ta ch ng minh r ng v i n 7, pn > 2n + 1() b ng phng php quy n p. V i n = 7, p7 = 17 > 2.7 + 1, () ng. Gi s () ng v i n = k, pk > 2k + 1, ta ch ng minh () ng v i n = k + 1. Th t v y, do pn l s l v i n > 1 nn pk+1 pk 2, ngha l pk+1 pk + 2 > 2k + 1 + 2 = 2(k + 1) + 1. Ti p theo, ta ch ng minh v i n, yn > n2 (), v i yn = p1 + p2 + + pn cng b ng quy n p. D ki m tra () ng v i n = 1, ..., 7. Gi s () ng v i n = k 7 ta c yk > k 2 . V i n = k + 1, yk+1 = yk + pk+1 > k 2 + 2k + 1 = (k + 1)2 . G i m2 l s chnh phng l n nh t khng l n hn yn . Theo trn, ta c m = n + k, k > 0. Nh v y, v i n 1, t n t i s k > 0 sao cho (n + k)2 yn < (n + k + 1)2 . Ta ch ng minh pn+1 > 2(n + k) + 1. Gi s ng c l i, ta c pn+1 2(n + k) + 1. Nhng v i n 2, pn+1 pn + 2, suy ra pn 2(n + k) + 1 2 = 2(n + k) 1, pn1 2(n + k) + 1 4 = 2(n + k) 3, .................................................. pnj 2(n + k) + 1 2(j + 1) = 2(n + k) (2j + 1), ........................................................... 3 = p2 2(n + k) + 1 2(n 1) = 2(n + k) (2n 3), 2 = p1 2(n + k) + 1 2n = 2(n + k) (2n 1).

Dy s v m t s tnh ch t C ng t ng v cc b t ng th c trn, ta c yn =p1 + p2 + + pn 2n(n + k) (1 + 3 + + (2n 1)) =2n(n + k) n2 = n2 + 2nk + k 2 k 2 = (n + k)2 k 2 .

181

i u ny mu thu n v i yn (n + k)2 . Nh v y pn+1 > 2(n + k) + 1. Khi yn+1 = yn + pn+ > (n + k)2 + 2(n + k) + 1 = (n + k + 1)2 > yn . Ngha l yn < (n + k + 1)2 < yn+1 . V y (n + k + 1)2 n m gi a yn v yn+1 . Bi t p 16. Cho dy s (an ) xc nh nh sau a1 = 1, a2 = 2, an+2 = 2an+1 an + 2, n 1. Ch ng minh r ng, v i m i m N , am am+1 cng l s h ng c a dy. H ng d n. Tm ra cng th c s h ng t ng qut c a dy s l an = (n 1)2 + 1. Khi am am+1 = [(m 1)2 + 1][m2 + 1] = (m2 + m + 1)2 + 1 = am2 +m+2 . Bi t p 17. Cho dy s (an ) xc nh nh sau a1 = 1, an+1 = an + 2 Ch ng minh r ng an > 1 + ( n 1). 3 H ng d n. an < an+1 , n 1 an 1, n 1. M t khc, 2 1 1 2 an = an1 + = a2 + 2 + 2 a2 + 3, n 2 n1 n1 an1 an1 a2 a2 + 3(n 1) = 3n 2 an 3n 2. n 1 1 1 1 T ta c > . Suy ra an1 3 n1 an = an1 + 1 1 + + an1 a2 an1 1 1 1 1 + + + 1+ . n1 3 1 2 = a1 + 1 1 , n 1. an

1 1 1 + + + 2( n 1) (). n1 1 2 2 V y an > 1 + ( n 1). pcm 3 Ch . Ch ng minh (*) b ng phng php sai phn.

Dy s v m t s tnh ch t Bi t p 18. Cho dy s (an ) xc nh nh sau a1 = 0, |an+1 | = |an + 1|, n 1. n Ch ng minh r ng S = a1 + a2 + + an . 2 H ng d n. Ta gi thi t ta c a2 = (an + 1)2 = a2 + 2an + 1, n 1 n+1 n a2 + a2 + + a2 = a2 + a2 + + a2 + 2S + n 2 3 n+1 1 2 n n 2S + n = a2 a2 = a2 0 S . n+1 1 n+1 2 pcm Bi t p 19. Xt dy s (an ) c

182

a1 = 1, a2 = 2, a3 = 3, a4 = 4, a5 = 5, a6 = 119, an+1 = a1 a2 ...an 1, n 5. Ch ng minh r ng a2 + a2 + + a2 = a1 .a2 ...a70 . 70 2 1 H ng d n. t bn = a1 .a2 ...an a2 a2 a2 = an+1 + 1 a2 a2 a2 . V i n 2 1 n 2 1 n 5 ta c bn+1 = a1 .a2 ...an an+1 a2 a2 a2 1 2 n+1 2 2 = (an+1 + 1)an+1 a1 a2 a2 n+1 = an+1 a2 a2 a2 = bn 1 1 2 n bn+1 = b5 (n 4) b70 = b5 65 = 0. Bi t p 20. Cho dy s (xn ) th a mn i u ki n x1 = 1, xn+1 = n 2 xn + , n 1. xn n2

Ch ng minh r ng n 4 ta lun c [xn ] = n. H ng d n. 49 9.12 49 2 , x4 = + + 2. R rng 4 + < x4 < 4 + 1. 12 49 9.12 4 2 Nh v y, b t ng th c kp n + < xn < n + 1() ng v i n = 4. n 2 Gi s () ng v i n = k, k 4, ngha l k + < xk < k + 1. k Ta s ch ng minh () ng v i n = k + 1, ngha l ch ng minh x2 = 3; x3 = k+1+ 2 < xk+1 < k + 2. k+1

Dy s v m t s tnh ch t Th t v y, theo gi thi t quy n p k2 k2 1 xk < k + 1 > =k1+ v xk k+1 k+1 xk 2 2 k2 + 2 1 xk > k + 2 > = + 3. 3 k k k k k 2 xk 1 2 k 1 1 + 2 >k1+ + + 3 >k1+ T suy ra xk k k+1 k k k+1 2 xk+1 > k + 1 + . k+1 M t khc, cng t gi thi t quy n p ta c xk+1 2 = k+1 k 2 xk k3 + 2 < 2 + xk k k + 21 k2 k 2 + 2k + 2 k 2 =k+ <k k 2 (k 2 + 2) xk+1 < k + 2.

183

Nh v y n +

2 < xn < n + 1, n 4, suy ra n 4 ta lun c [xn ] = n. n

Bi t p 21. Cho dy s (an ) th a mn i u ki n a0 = 1, a1 = 6, an+2 = 6an+1 an , n 0. Ch ng minh r ng 1. a2 6an an+1 + a2 = 1, n N, n n+1 2. n N , k N : an = k(k + 1) . 2

H ng d n. 1) T cng th c truy h i ta c an+2 + an = 6an+1 , n N a2 a2 = 6an+1 (an+2 an ), n N n+2 n a2 6an+1 an+2 + a2 = a2 6an an+1 + a2 n+2 n+1 n+1 n = = a2 6a0 a1 + a2 = 1. 1 0 2) D th y an N , n N. Ta c 8a2 + 1 = 8a2 + a2 6an an+1 + a2 n n n+1 n = a2 6an an+1 + 9a2 n+1 n 2 = (an+1 3an ) . Suy ra 8a2 + 1 l s chnh phng l , ngha l t n t i k N sao cho 8a2 + 1 = n n k(k + 1) (2k + 1)2 hay an = . pcm 2

Dy s v m t s tnh ch t Bi t p 22. Cho dy s t nhin (an ) th a mn i u ki n an = a2 + a2 + a2 , n 3. n1 n2 n3 Ch ng minh r ng n u ak = 2010 th k 3. H ng d n. Ph n ch ng. Gi s k > 3, ta c an N, n v 2010 = ak = a2 + a2 + a2 a2 < 2010 ak1 44 (). k1 k2 k3 k1 M t khc ak1 = a2 + a2 + a2 (do k > 3) k2 k3 k4 2 2 ak1 ak2 + ak3 ak1 + a2 2010 ak1 45 (). k1 () v () mu thu n v i nhau hay gi s sai. V y ta c pcm. Bi t p 23. Cho cc dy s (an ), (bn ), (cn ), (dn ) th a mn i u ki n an+1 = an + bn , bn+1 = bn + cn , cn+1 = cn + dn , dn+1 = dn + an , n 0. Gi s t n t i hai s nguyn dng k, r sao cho ak+r = ak , bk+r = bk , ck+r = ck , dk+r = dk , Ch ng minh r ng a1 = b1 = c1 = d1 = 0. H ng d n. t sn = an + bn + cn + dn . T gi thi t suy ra sk+r = sk ; sn+1 = 2sn , n 0. B ng quy n p ta tm c sn = 2n s0 , n 0. V v y ta c 2k+r s0 = 2r s0 s0 = 0 sn = 0 n 0. L i c n 1 an + cn = (an1 + bn1 ) + (cn1 + dn1 ) = sn1 = 0. (1) t wn = a2 + b2 + c2 + d2 . Ta c wk+r = wk v n n n n wn+1 = (an + bn )2 + (bn + cn )2 + (cn + dn )2 + (dn + an )2 = 2(a2 + b2 + c2 + d2 ) + (an bn + bn cn + cn dn + dn an ) n n n n = 2wn + 2(an + cn )(bn + dn ). Do (1) nn wn+1 = 2wn , v i n 1. Theo quy n p ta c wn = 2n1 w1 , n 1. T 2k+r1 w1 = 2k1 w1 w1 = 0 a1 = b1 = c1 = d1 = 0. pcm Bi t p 24. Cho hai dy s (xn ), (yn ) th a mn i u ki n x0 = y0 = 1, xn+1 = xn + 2 y2 + 2 , yn+1 = n , xn + 1 2yn n 0.

184

Ch ng minh r ng yn = x2n 1 , n 0.

Dy s v m t s tnh ch t

185

H ng d n. Xt hai dy s (an ), (bn ) xc nh xn 2 yn 2 , bn = , n 0. an = xn + 2 yn + 2 1 2 := . T cng th c trn ta ch ng minh c an+1 = Ta c a0 = b0 = 1+ 2 an , bn+1 = b2 . B ng quy n p ta c n an = n+1 , bn = 2 , n 0.
n 1)+1 n

Thay n b i 2n 1 ta c a2n 1 = (2 x2n 1 2 yn 2 = x2n 1 + 2 yn + 2 2 2 2 2 =1 1 x2n 1 + 2 yn + 2 x2n 1 = yn . pcm

= 2 = bn . Khi

Bi t p t gi i
Bi t p 25. Cho x1 , x2 , ...., xn l cc s th c b t k. Ch ng minh r ng: x1 x2 x2 + + + < n. 2 2 2 2 1 + x1 1 + x 1 + x2 1 + x1 + + x2 n 1 1 1 Bi t p 26. Tm ph n nguyn c a t ng S = 1 + + + + . 2 3 100 Bi t p 27. Cho dy s (an ) xc nh nh sau a0 = 1, a1 = a, an+2 = 2an+1 an + 2, n 1. Ch ng minh r ng, v i n u a N th n N t n t i m N sao cho an an+1 = am . Bi t p 28. Cho dy s (an ) xc nh nh sau a1 = a, a > 1, 2011an+1 = a2 + 2010an , n 1. n Ch ng minh r ng, v i n N ta c
n i=1

ai 2011 < . ai+1 1 a1

H ng d n gi i. S d ng phng php sai phn. Bi t p 29. Cho dy s (an ) xc nh nh sau a1 = 3, an+1 = a2 3an + 4, n 1. n Ch ng minh r ng, v i n N ta c
n i=1

1 < 1. ui 1

Dy s v m t s tnh ch t H ng d n gi i. S d ng phng php sai phn. Bi t p 30. Cho dy s (an ) th a mn 0 < a1 < 1, an+1 = Ch ng minh r ng, dy s (an ) b ch n. H ng d n gi i. S d ng phng php sai phn. Bi t p 31. Cho dy s th c (an ) th a mn an+1 = a2 1 n , n 0. 2an a2 n + an , n 1. n2

186

Ch ng minh r ng, dy s (an ) ch a v h n s h ng m, v h n s h ng dng. H ng d n gi i. S d ng phng php php th l ng gic, t a0 = cot t. Bi t p 32. Cho dy s (an ) th a mn a1 = 1.2.3, an = n(n + 1)(n + 2). K hi u Sn = a1 + a2 + an . Ch ng minh r ng 4Sn + 1 l s chnh phng. H ng d n gi i. S d ng phng php sai phn, v i 1 an+1 = ((n + 1)(n + 2)(n + 3)(n + 4) n(n + 1)(n + 2)(n + 3)). 4 Bi t p 33. Cho dy s (an ) th a mn a1 = a2 = a3 = a4 = a5 = 1, an+6 an+1 = an+2 an+5 + an+3 an+4 n 0. Ch ng minh r ng an N, n N . H ng d n gi i. S d ng phng php quy n p. c n + 5 s h ng u tin l s t nhin, ta c n ch ng minh an+6 N b ng cch ch ng minh an+2 an+5 an+3 an+4 ( mod an+1 ). Bi t p 34. Cho dy s (an ) th a mn a1 < 0an+1 = an 1 n 1. an

Ch ng minh r ng t n t i k, 1 k 2010 th a mn ak < 0. H ng d n gi i. Ch ng minh t n t i k, 1 k 1626 th a mn ak < 0. Bi t p 35. Cho dy s (an ) th a mn a1 = 2, a2 = 3, an+1 = 2an1 ho c an+1 = 3an 2an1 , n 2. Ch ng minh r ng khng c s h ng no c a dy n m trong o n t 1600 n 2010.

Dy s v m t s tnh ch t

187

H ng d n gi i. S d ng phng php quy n p ch ng minh an = 2xn +an = 2yn v i xn , yn N v ho c xn = xn1 ; yn = yn1 + 1 ho c xn = xn1 + 1; yn = yn1 . Bi t p 36. Cho dy s (an ) th a mn a0 = 0, a1 = 1, an+2 = 3an+1 2xn , n 1. t yn = x2 + 2n+2 . Ch ng minh r ng yn l s chnh phng l . n H ng d n gi i. Tm s h ng t ng qut c a (xn ). Bi t p 37. Xt cc s dng x1 , x2 , ..., xn th a mn minh r ng 1 1. n 1 + xi xi = 1 . Ch ng xi

H ng d n gi i. t yn =

1 . Khi ta c n ch ng minh n 1 + xi S d ng phng php ph n ch ng.

yi 1.

Bi t p 38. Xt dy s (an ) th a mn an+2 = an+1 an . Gi s t ng c a 2009 s h ng u tin l 2011, t ng c a 2011 s h ng u tin l 2009. H i t ng c a 2020 s h ng u tin l bao nhiu? H ng d n gi i. Ch ng minh dy s trn tu n hon v i chu k 6.

2.8.4

K t lu n

Chuyn trnh by m t s phng php gi i bi ton v dy s c lin quan n tnh ch t, cng v i cc v d minh h a, p d ng chng ch ng minh tnh ch t c a dy s . Trong ph n p d ng ny, ng i vi t c gi i thi u m t s tnh ch t th ng g p v dy s , ch y u minh h a qua cc v d cng h ng d n gi i k t h p v i nh n xt v l i v b n ch t c a bi ton ng i c hi u v n m b t c v n d dng. Cu i cng, ng i vi t a ra cc bi t p lin quan, m t s bi c h ng d n gi i chi ti t m t l n n a ng i c th y r hn v cc bi ton d ng ny, m t s bi c a ra nh h ng phng php gi i nhng khng h ng d n chi ti t ng i c t gi i, rn luy n, thu ho ch k t qu c a mnh khi c chuyn . Qua chuyn ny, ng i vi t th y r ng c n ph i nghin c u su hn n a v cc tnh ch t nu c a dy s , cn r t nhi u cc phng php gi i khc cha c a ra, r t nhi u tnh ch t khc v dy s v n cha c khm ph. Ng i vi t r t mong qu c gi quan tm v b sung thm. Ti li u tham kh o 1. Nguy n Vn M u, Chuyn ch n l c dy s v p d ng, NXB Gio D c, 2008. 2. Nguy n Vn M u, Nguy n Nguy n Vn Ti n, M t s chuyn gi i tch b i d ng h c sinh gi i trung h c ph thng, NXB Gio D c Vi t Nam, 2009.

M t s phng php gi i h phng trnh

188

3. Nguy n H u i n, Phng php quy n p ton h c, NXB Gio D c, 2001. 4. Nguy n Vn Nho, Tuy n t p cc bi ton t nh ng cu c thi t i Trung Qu c, NXB Gio D c, 2003. 5. Andrei Negut, Problems for the Mathematican Olympiad. 6. T p ch ton h c v tu i tr , Tuy n t p 5 nm, NXB Gio D c, 2003. 7. Ti li u t Internet.

2.9

M t s phng php gi i h phng trnh trong cc bi thi h c sinh gi i

Hunh T n Chu Tr ng THPT Chuyn Lng Vn Chnh, Ph Yn

2.9.1

Dng cc php bi n i i s

Bi ton 1. (CH N I TUY N TP H N I 2005) 1 + 1 = (x2 + 3y 2 ) (3x2 + y 2 ) x 2y Gi i h phng trnh: 1 1 = 2 (y 4 x4 ) x 2y L i gi i. K: x = 0, y = 0 2 = x4 + 5y 4 + 10x2 y 2 2 = x5 + 10x3 y 2 + 5xy 4 x H phng trnh 1 = 5x4 y + 10x2 y 3 + y 5 1 = 5x4 + y 4 + 10x2 y 2 y 3 = x5 + 5x4 y + 10x3 y 2 + 10x2 y 3 + 5xy 4 + y 5 1 = x5 5x4 y + 10x3 y 2 10x2 y 3 + 5xy 4 y 5 1+ 53 x= 3 = (x + y)5 x+y = 53 2 xy =1 1 + 5 3 1 = (x y)5 y= 2

Bi ton 2. Gi i h phng trnh: 1 2 x+ y 4 + x 4 x +y 2 x+ y 4y 1 4 x+y L i gi i.

=2 =1

M t s phng php gi i h phng trnh N u x = 0 ho c y = 0 ho c x = y = 0 th h v nghi m. Do i u ki n c a h : x > 0 v y > 0. t x = u, y = v (u, v > 0), h cho tr thnh: 2 1 u 1 + 2u + v = 2 + = 1 (1) 2 + v2 4 u 2 u v 2 1 1 2u + v = 4u + 2v (2) v =1 u2 + v 2 u v 4 u2 + v 2

189

Nhn (1) v (2) v theo v ta c: 2v (u2 + 2v 2 ) u (u2 + 2v 2 ) = 0 hay (2v u) (u2 + 2v 2 ) = 0, suy ra u = 2v. Thay u = 2v vo (1) ta c v = 2 1 + 2 hay v = 4 3 + 2 2 v u = 8 3 + 2 2 , suy ra x = 64 17 + 12 2 ; y = 16 17 + 12 2 . Bi ton 3. Gi i h phng trnh sau: 4 x + 2y 3 x = 1 + 3 3(1) 2 1 y 4 + 2x3 y = 3 3(2) 2 z + y x = log (y x) (3) 3 L i gi i. i u ki n : yx > 0 C ng phng trnh (1) v (2) v theo v ta c 1 1 x +2x x+ +y 4 +2y 3 y+ = 0 4 4
4 3

x, y Xt phng trnh t2 + t

1 1 x +x + y2 + y 2 2 1 3 1 + 3 , 2 2
2

= 0(4)

1 = 0 (*) . Gi s l m t nghi m c a pt (*) 2 1 3 1 4 3 2 = + , 3 = 2 + = , = 2 + 2 2 2 4 3 3y 1 T c l x4 = 2x + , y 3 = , thay vo (1) ta c yx = 3 4 2 1 3 1 + 3 x= ,y = tho mn (1) , (2) v (4) 2 2 V i yx = 3 (tho i u ki n) , thay vo (3) ta c 1 1 z + 3 = log3 3 = z = 3 2 2 V y h phng trnh cho c nghi m l 1 3 1 + 3 12 3 x= ,y = ,z = 2 2 2

M t s phng php gi i h phng trnh

190

Bi ton 4. (CH N I TUY N PTNK- HQG TPHCM 2004) Gi i h phng trnh : u + v = 2(1) ux + vy = 3(2) ux2 + vy 2 = 5(3) ux3 + vy 3 = 9(4) L i gi i. Cch 1. T (1) v = 2 u, th vo (2) : ux + (2 u)y = 3 u(x y) = 3 2y Th v = 2 u vo (3) : ux2 + (2 u) y 2 = 5 u (x2 y 2 ) = 5 2y 2 (3 2y) (x + y) = 5 2y 2 dou (x y) = 3 2y 3(x + y) 2xy = 5 (5) Th v = 2 u vo (4) : ux3 + (2 u) y 3 = 9 u (x3 y 3 ) + 2y 3 = 9 (3 2y) (x2 + xy + y 2 ) + 2y 3 = 9 3(x + y)2 3xy 2xy(x + y) = 9 (6) 3(x + y) 2xy = 5 T (5) v (6): 3(x + y)2 3xy 2xy(x + y) = 9 t x + y = a, xy = b Ta c : 3a 2b = 5 3a2 2ab 3b = 9 a=3 b=2 x+y =3 xy = 2 x=2 y=1 x=1 y=2

Th cc gi tr c a x v y vo hai phng trnh u: * V i x = 2, y = 1 u+v =2 2u + v = 3 * V i x = 1, y = 2 u+v =2 u + 2v = 3 u=1 (x, y, u, v) = (1, 2, 1, 1) v=1 u=1 (x, y, u, v) = (2, 1, 1, 1) v=1

V y h phng trnh c 2 nghi m x, y, u, v l 2, 1, 1, 1 v (1, 2, 1, 1) Cch 2. * Xt x = 0 vy = 3, vy 2 = 5, vy 3 = 9 - N u y = 0 ho c v = 0, h phng trnh v nghi m. y=5 3 : H phng trnh v nghi m - N u y = 0, v = 0 y=9 5

M t s phng php gi i h phng trnh ux + vx = 2x(1 ) ux2 + vxy = 3x(2 ) * V y x = 0, h phng trnh ux3 + vxy 2 = 5x(3 ) ux3 + vy 3 = 9(4 ) L y (1 ) (2) v theo v : v(xy) = 2x3 (a) (2 ) (3) : vy(xy) = 3x5 (b) (3 ) (4) : vy2(xy) = 5x9 (c) - N u v = 0 ho c y = 0, h phng trnh v nghi m - N u x = y, h phng trnh v nghi m Suy ra v = 0, y = 0, x = 0 1 2x 3 L y (a) chia (b) v theo v : = (5) y 3x 5 1 3x 5 L y (b) chia (c) v theo v : = (6) y 5x 9 2x 3 3x 5 T (5) v (6) suy ra: = 3x 5 5x 9 2 (2x 3)(5x 9) = (3x 5) 10x2 33x + 27 = 9x2 30x + 25 x2 3x + 2 = 0 x=1 x=2

191

* Th x = 1 va (5) ta c y = 2 Th x = 1 va (a): v = 1 u = 1 * Th x = 2 ta c: y = 1, v = 1, u = 1 V y h phng trnh c 2 nghi m (x, y, u, v) l 2, 1, 1, 1 v (1, 2, 1, 1) Bi ton 5. (VMO 2010) Gi i h phng trnh : x4 y 4 = 240 x3 2y 3 = 3(x2 4y 2 ) 4(x 8y) L i gi i. Cch 1. Nhn phng trnh th hai v i 8 r i c ng v i phng trnh th nh t, ta c x4 8x3 + 24x2 32x + 16 = y 4 16y 3 + 96y 2 256y + 256 (x 2)4 = (y 4)4 x2=y4x2=4y x=y2x=6y Thay vo phng trnh u, ta c : (1) 8y 3 + 24y 2 32y + 16 = 240 y 3 3y 2 + 4y + 28 = 0 (y + 2)(y 2 5y + 14) = 0. Suy ra y = 2 v x = 4. (2) 24y 3 + 216y 2 864y + 1296 = 240 y 3 9y 2 + 36y 44 = 0 (y 2)(y 2 7y + 22) = 0. Suy ra y = 2 v x = 4. V y h cho c 2 nghi m l (x, y) = (4, 2) v (x, y) = (4, 2).

M t s phng php gi i h phng trnh Cch 2. t y = 2t thay vo phng trnh v vi t l i h d i d ng x4 + 16 = 16(t4 + 16) (1) x3 3x2 + 4x = 16(t3 3t2 + 4t) (2) Nhn cho 2 phng trnh ny, ta c (x4 + 16)(t3 3t2 + 4t) = (t4 + 16)(x3 3x2 + 4x) (3)

192

D th y n u (x, t) l nghi m c a h th xt = 0 nn ta chia hai v c a phng trnh trn cho x2 t2 th c : (x2 + 16 16 4 4 )(t 3 + ) = (t2 + 2 )(x 3 + ) x2 t t x

T y n u t u = x + 4/x v v = t + 4/t th ta c phng trnh (u2 8)(v 3) = (v 2 8)(u 3) u2 v v 2 u 3(u2 v 2 ) + 8(u v) = 0 (u v)(uv 3(u + v) + 8) = 0 (4) T (1) ta suy ra r ng x v t cng d u. Do p d ng b t ng th c AM-GM ta d dng suy ra u, v ho c cng 4 ho c cng 4. Suy ra (u 3) v (v 3) lun l n hn hay b ng 1 ho c lun nh hn hay b ng 7. Suy ra uv 3(u + 3) + 8 = (u 3)(v 3) 1 0. D u b ng ch c th x y ra khi u = v = 4. T l lu n trn v t (2) ta suy ra u = v, t suy ra x = t ho c x = 4/t. Tr ng h p x = t. Thay vo phng trnh (1) ta c t4 + 16 = 16(t4 + 16), v nghi m. Tr ng h p x = 4/t. Thay vo phng trnh (1), ta c : 256/t4 + 16 = 16(t4 + 16) t8 + 15t4 16 = 0 (t4 1)(t4 + 16) = 0 Suy ra t = 1. T ta c cc nghi m (x, y) = (4, 2) v (4, 2). Nh n xt. L i gi i 1 kh ng n g n nhng l 1 t ng khng d ngh ra. N u nh t x = 2u, y = 2v v a v h phng trnh u4 v 4 = 15 2(u3 2v 3 ) = 3(u2 4v 2 ) 2(u 8v) th c l s d nhn th y cc h s nh th c hn. D sao th y l m t t ng khng m i. N c s d ng VMO 2004, b ng B. Th m ch xt v m t m t no th bi VMO 2004 cn kh hn bi nm nay. C th bi VMO 2004 nh sau: Gi i h phng trnh sau : x3 + 3xy 2 = 49 x2 8xy + y 2 = 8y 17x.

M t s phng php gi i h phng trnh

193

Cch gi i p n c a bi ny nh sau: t x + y = u, x y = v th x = (u + v)/2, y = (u v)/2 v h c th a v d ng : u3 + v 3 = 98 3u2 + 5v 2 = 9u 25v Sau nh n phng trnh th hai v i 3 r i c ng v i phng trnh th nh t th c (u 3)3 + (v + 5)3 = 0. R rng cch gi i ny tng ng v i cch gi i th nh t c a VMO 2010. Tuy nhin, bi VMO 2004 cn c 1 cch gi i n gi n hn l nhn phng trnh th hai (c a h ban u) v i 3 r i c ng v i phng trnh th nh t v a v d ng : (x + 1)((x 1)2 + 3(y 4)2) = 0. Bi ton 6. 2 x (y + z)2 = (3x2 + x + 1) y 2 z 2 (1) y 2 (z + x)2 = (4y 2 + y + 1) z 2 x2 (2) (I) Gi i h phng trnh: 2 z (x + y)2 = (5z 2 + z + 1) x2 y 2 (3) L i gi i. Tr ng h p 1: xyz = 0 * N u x = 0, (I) y = 0 hay z = 0 Khi h nh n nghi m (0; 0; z) v (0; y; 0), y, z R * Tng t cho tr ng h p y = 0 hay z = 0 Tr ng h p 2: xyz = 0 Chia hai v cc phng trnh c a (I) cho x2 y 2 z 2 ta c h phng trnh: 1 1 2 1 1 =3+ + 2 z+y x x 2 1 1 1 1 + =4+ + 2 x z y y 1 1 2 1 1 + =5+ + 2 y x z z ta= 1 1 1 ,b = ,c = . H phng trnh tr thnh: x y z 2 (b + c) = 3 + a + a2 (4) (c + a)2 = 4 + b + b2 (5) (a + b)2 = 5 + c + c2 (6)

C ng cc v c a phng trnh, rt g n ta c : (a + b + c)2 (a + b + c) 12 = 0 a+b+c=4 a + b + c = 3

M t s phng php gi i h phng trnh Khi a + b + c = 4 thay v (4), (5), (6) ta tnh c: a = 13 x= 9 9 13 4 3 b= y= 3 4 c = 11 z= 9 9 11 Khi a + b + c = 3 thay v (4), (5), (6) ta tnh c: x = 5 a = 6 5 6 b = 1 y = 1 z = 5 c = 4 5 4 K t lu n: H phng trnh c cc nghi m l : 9 3 9 (x; 0; 0) , (0; y; 0) , (0; 0; z) (x, y, z R), ; ; 13 4 11 5 5 , ; 1; 6 4

194

Bi t p p d ng
Bi t p 1. Gi i h phng trnh: 3 x + y = 3x + 4 2y 3 + z = 6y + 6 3 3z + x = 9z + 8 Bi t p 2. Gi i h phng trnh: 2 x = 2x y 2 y = 2y z z 2 = 2z t 2 t = 2t x Bi t p 3. Cho s nguyn n 3. Gi i h phng trnh : 2x1 5x2 + 3x3 = 0 2x2 5x3 + 3x4 = 0 ............................. 2xn 5x1 + 3x2 = 0 Bi t p 4. (VMO 1996) Gi i h phng trnh : 3x 1 + 1 =2 x+y 7y 1 1 =4 2 x+y

M t s phng php gi i h phng trnh

195

Bi t p 5. Gi i h phng trnh: 2 2 x = a + (y z) y 2 = b + (z x)2 2 z = c + (x y)2 Bi t p 6. Gi i h phng trnh : 3 x + y 3 + x2 (y + z) = xyz + 14 y 3 + z 3 + y 2 (z + x) = xyz 21 3 z + x3 + z 2 (x + y) = xyz + 7 Bi t p 7. Cho a l m t s th c cho tr c. Gi i h phng trnh: 2 2 a1 x1 + ax1 + = x2 2 2 a1 2 x2 + ax2 + = x3 2 ..................................... 2 2 x1000 + ax1000 + a 1 = x1 2 Bi t p 8. Tm b b n s (x, y, u, v) th a mn h phng trnh: 2 x + y 2 + u2 + v 2 = 4 xu + yv = xv yu xyu + yuv + uvx + vxy = 2 xyuv = 1 Bi t p 9. (IRAN 2006) Gi i h phng trnh : x + y + zx = 1 2 1 y + z + xy = 2 z + x + yz = 1 2 Bi t p 10. (VMO 2004 B NG A) Gi i h phng trnh: 3 2 x + x(y z) = 2 y 3 + y(z x)2 = 30 3 z + z(x y)2 = 16

M t s phng php gi i h phng trnh

196

Bi t p 11. (BULGARIAN 2003) Gi i h phng trnh: x + y + z = 3xy x2 + y 2 + z 2 = 3xz 3 x + y 3 + z 3 = 3yz

2.9.2

S d ng tnh n i u c a hm s

Bi ton 1. (OLYMPIC 30 4 2009 L P 11) Gi i h phng trnh : 3 2x + 3x2 18 = y 3 + y 2y 3 + 3y 2 18 = z 3 + z 3 2z + 3z 2 18 = x3 + x L i gi i. t f (t) = 2t3 +3t2 18 v g(t) = t3 +t. Ta c g (t) = 3t2 +1 > 0, t R. Suy ra hm s g(t) ng bi n trn R. f (x) = g (y) f (y) = g (z) Khi h phng trnh c vi t l i: f (z) = g (x) Gi s x = max(x, y, z) th xy xz g (x) g (y) g (x) g (z) g (x) f (x) f (z) g (z) x2 z2

x3 + x 2x3 + 3x2 18 2z 3 + 3z 2 18 z 3 + z

(x 2) (x2 + 5x + 9) 0 (z 2) (z 2 + 5z + 9) 0

T suy ra: 2 z x 2 x = z = 2 Th vo h phng trnh ta c y = 2 Th l i ta th y x = y = z = 2 th a mn h phng trnh. K t lu n: H phng trnh c nghi m duy nh t x = y = z = 2. Bi ton 2. (VMO 2006 B NG A) Gi i h phng trnh : 2 x 2x + 6.log3 (6 y) = x y 2 2y + 6.log3 (6 z) = y 2 z 2z + 6.log3 (6 x) = z L i gi i.

M t s phng php gi i h phng trnh

197

i u ki n : x, y, z < 6. x log3 (6 y) = x2 2x + 6 y log3 (6 z) = H phng trnh y 2 2y + 6 z log (6 x) = 3 z 2 2z + 6 Ta th y hm s f (t) = log3 (6 t) l hm s ngh ch bi n trn (, 6). t 6t Xt hm s g (t) = > 0t c g (t) = t2 2t + 6 2 2t + 6)3 (t (; 6) Suy ra g(t) ng bi n trn kho ng (, 6). T ta c c x = y = z = 3. H phng trnh c nghi m duy nh t x = y = z = 3. Bi ton 3. 1 x2 3 2 x2 + xy + = 2y (1) Gi i h phng trnh: 2 2 2 (x y + 2x) 2x2 y 4x + 1 = 0 (2) L i gi i. i u ki n x = 0 (2) x2 y + 2x 1
2

=0y=

1 2x x2

1 x2 1 2x 1 2x 3 1 2x vo (1) : 2 x2 + + = 2 x2 Thay y = x2 x 2 1 x2 1 2x 1 1 1 2x 1 x2 x2 2 x2 = = 2 x 2 2x2 2x2 2 1x 1 2x 1 x2 1 2x 2 + 2 x = 2 x2 + (3) 2x2 2x2 t t f (t) = 2t + , l hm s ng bi n trn R. 2 (3) f 1 x2 x2 =f 1 2x x2 3 4 1 x2 1 2x = x=2 x2 x2

V y h c nghi m x = 2; y =

Bi ton 4. (VMO 2008) Hy xc nh s nghi m c a h phng trnh ( n x, y) sau: x2 + y 3 = 29 log3 x.log2 y = 1 L i gi i.

M t s phng php gi i h phng trnh K: x, y > 0 t log3 x = a, log2 y = b x = 3a , y = 2b 1 1 9a + 8b = 29 Ta c h m i . Thay b = : 9a + 8 a = 29 ab = 1 a 1 a Xt hm: f (x) = 9 + 8 a 29 1 1 f (x) = ln 9.9 2 ln 8.8 a a 1 1 1 1 2 2 f (x) = ln 9.9a + 2 ln 8.8 a . 2 + 3 . ln 8.8 a > 0 a a a
a

198

M t khc, ta c: f (1) > 0, f

1 2

< 0 f c nghi m 1 , 1 nn f (x) tng trn (a0 , +), 2

Do n c nghi m duy nh t a0 ; a0

gi m trn (, a0 ). V do f n i u trn t ng kho ng y nn nghi m l nghi m duy nh t trn cc kho ng y. V y h cho c hai nghi m. Bi ton 5. Gi i h phng trnh : 3 x 2y 3 2 (x2 3y 2 ) + 3 (x 2y) 1 = 0 y 3 2z 3 2 (y 2 3z 2 ) + 3 (y 2z) 1 = 0 3 z 2x3 2 (z 2 3x2 ) + 3 (z 2x) 1 = 0 L i gi i. 3 x 2x2 + 3x 1 = 2y 3 6y 2 + 6y y 3 2y 2 + 3y 1 = 2z 3 6z 2 + 6z H phng trnh 3 z 2z 2 + 3z 1 = 2x3 6x2 + 6x t f (t) = t3 2t2 + 3t 1, g (t) = 2t3 6t2 + 6t Ta c f (t) = 3t2 4t + 3 > 0, t R, g (t) = 6t2 12t + 6 = 6(t 1)2 0, t R Do f (t), g(t) ng bi n trn R. f (x) = g (y) (1) f (y) = g (z) (2) H phng trnh f (z) = g (x) (3) Gi s (x; y; z) th a mn h phng trnh cho. Khng m t t ng qut, gi s : x y T (1) v (2) suy ra : g (y) g (z) y z T (2) v (3) suy ra : g (z) g (x) z x

M t s phng php gi i h phng trnh Do x = y = z. H phng trnh x=y=z x3 + 4x2 3x 1 = 0

199 x=y=z x3 4x2 + 3x + 1 = 0 (4)

t t=x1x=t+1 (4) : (t + 1)3 4(t + 1)2 + 3 (t + 1) + 1 = 0 t3 t2 2t + 1 = 0 (5) t h (t) = t3 t2 2t + 1, ta c h(t) lin t c trn R. V h (2) = 7 < 0, h (0) = 1 > 0, h (1) = 1 < 0, h (2) = 1 > 0, nn PT: h(t) = 0 c 3 nghi m phn bi t n m trong kho ng (2, 2) t t = 2 cos , (0; ). Khi sin = 0 (5) : 8 cos3 4 cos2 4 cos +1 = 0 4 cos (2 cos2 1)4 1 sin2 + 1=0 4 cos . cos 2 + 4sin2 3 = 0 4 cos . cos 2. sin = 3 sin 4sin3 sin 4 = sin 3 3 5 V i (0; ) ta thu c ; ; . Do t = 2 cos , 7 7 7 3 5 ; ; 7 7 7 3 5 V y h phng trnh c nghi m: x = y = z = 2 cos + 1, ; ; 7 7 7

Bi t p p d ng
Bi t p 1. (OLYMPIC 30 - 4 - 2008) 2x 2 = 3y 3x Gi i h phng trnh : 3y 2 = 3x 2y S: x = y = 0; x = y = 1 Bi t p 2. Gi i h phng trnh : S: x = y = 1 Bi t p 3. (VMO 1994 B NG B) x2 + 3x + ln (2x + 1) = y Gi i h phng trnh: y 2 + 3y + ln (2y + 1) = x S: x = y = 0 Bi t p 4. (VMO 1994 B NG A) x+ y+ x2 2x + 2 = 3y1 + 1 y 2 2y + 2 = 3x1 + 1

M t s phng php gi i h phng trnh Gi i h phng trnh: 3 x + 3x 3 ln (x2 x + 1) = y y 3 + 3y 3 ln (y 2 y + 1) = z 3 z + 3z 3 ln (z 2 z + 1) = x S: x = y = z = 1 Bi t p 5. (VMO 1998) (1 + 42xy ) .512x+y = 1 + 22xy+1 S: Gi i h sau : y 3 + 4x + 1 + ln (y 2 + 2x) = 0 Bi t p 6. Gi i h phng trnh: 3 x + 3x + 3x + 1 5 y = 0 y 3 + 3y + 3y + 1 5 z = 0 3 z + 3z + 3z + 1 5 x = 0 S: x = y = z = 1 Bi t p 7. 3 x 3x2 + 5x + 1 = 4y y 3 3y 2 + 5y + 1 = 4z Gi i h phng trnh: 3 z 3z 2 + 5z + 1 = 4x S: x = y = z = 1; x = y = z = 1 2 Bi t p 8. 30x2 9x2 y 25y = 0 30y 2 9y 2 z 25z = 0 Gi i h phng trnh: 30z 2 9z 2 x 25x = 0 S: x = y = z = 0; x = y = z = 2 Bi t p 9. x 5 = 5 (4y 3) 5y = 5 (4z 3) Gi i h phng trnh : z 5 = 5 (4x 3) S: x = y = z = 1; x = y = z = 2 x=0 y = 1

200

2.9.3

Phng php nh gi

Bi ton 1. (VMO 2006 B NG B) Gi i h phng trnh : 3 x + 3x2 + 2x 5 = y y 3 + 3y 2 + 2y 5 = z 3 z + 3z 2 + 2z 5 = x

M t s phng php gi i h phng trnh (x 1) (x2 + 4x + 6) = y 1 (y 1) (y 2 + 4y + 6) = z 1 L i gi i. Cch 1. H phng trnh (z 1) (z 2 + 4z + 6) = x 1

201

Ta c x2 + 4x + 6 > 0, y 2 + 4y + 6 > 0, z 2 + 4z + 6 > 0, x, y, z R Tr ng h p 1: N u x = 1, th suy ra y = 1, z = 1. H phng trnh c nghi m (1, 1, 1) Tr ng h p 2: N u x = 1, suy ra y = 1, z = 1 Nhn cc PT trong h v theo v ta c : (x 1) (y 1) (z 1) x2 + 4x + 6 x2 + 4x + 6 y 2 + 4y + 6 z 2 + 4z + 6 = (x 1) (y 1) (z 1)

y 2 + 4y + 6

z 2 + 4z + 6 = 1

i u ny khng th x y ra. V y h phng trnh c nghi m duy nh t (x, y, z) = (1, 1, 1) 3 (x + 1) 6 = y + x (1) Cch 2. H phng trnh (y + 1)3 6 = z + y (2) (z + 1)3 6 = x + z (3) m t t ng qut gi s x y v (2) suy ra y + x z + y x z v (3) suy ra y + x x + z y z v (3) suy ra z + y x + z y x c x = y = z x=y=z H phng trnh x3 + 3x2 + 2x 5 = x x=y=z=1 Khng T (1) T (1) T (2) V y ta Bi ton 2. (USA Math Olympiad Progam 1995) Gi i h phng trnh: 3 + 2x2 y x4 y 2 + x4 (1 2x2 ) = y 2 1+ L i gi i. H phng trnh C ng (1) v (2): Ta c 4 (1 x2 y)2 = 2x6 x4 + y 2 (1) 1 + (x y)2 = x3 (x3 x + 2y 2 )

x=y=z (x 1) (x2 + 4x + 5) = 0

1 + (x y)2 = 1 x6 + x4 2x3 y 2 (2) 1 + (x y)2 = x6 2x3 y + y 2 + 1


2

4 (1 x2 y)2

4 (1 x2 y)2 =

1 + (x y)2 + x3 y

+ 1 (3)

4 (1 x2 y)2 2 v

1 + (x y)2 + (x3 y) + 1 2

M t s phng php gi i h phng trnh 1 x2 y = 0 xy =0 Do PT (3) th a mn khi v ch khi : x=y=1 3 x y =0

202

Th x = y = 1 vo h phng trnh ta th y h th a mn. V y h phng trnh c nghi m duy nh t x = y = 1. Bi ton 3. Tm t t c cc c p s th c (x; y) th a mn cc i u ki n sau: y 6 + y 3 + 2x2 = xy x2 y 2 (1) 1 4xy 3 + y 3 + 2x2 + 1 + (2x y)2 (2) 2 L i gi i. i u ki n xy x2 y 2 0 0 xy 1 2 1 1 1 2 2 Ta c xy x y = xy + 2 4 4 1 T (1) v (3) suy ra : y 6 + y 3 + 2x2 (4) 2 C ng (2) v (4) v theo v ta c: 4xy 3 + 1 y 6 + 4x2 + Do 1 1 + (2x y)2 1
2

xy x2 y 2

1 (3) 2

1 + (2x y)2 y 3 2x

1 + (2x y)2 0, (y 3 2x) 0 1 + (2x y)2 = (y 3 2x) = 0 x=0 y=0 1 2 y=1 x=


2

Nn ta c: 1

2x y = 0 y 3 2x = 0

x = 1/2 y = 1

Th l i ta th y ch c c p K t lu n : x = 1/2 y = 1

x = 1/2 th a mn i u ki n bi ton. y = 1

Bi ton 4. Tm t t c cc s th c x, y, z tho h : 3 y + 2 = (3 x) (1) (2z y) (y + 2) = 9 + 4y (2) x2 + z 2 = 4x (3) z 0 (4) L i gi i. Ta nh gi n z. Ta c: (2) y 2 + 2y (3 z) + 9 4z = 0

M t s phng php gi i h phng trnh PT (2) c nghi m y khi v ch khi: y = (3 z)2 (9 4z) 0 z 2 2z 0 z0 (5) z2

203

(3) x2 4x + z 2 = 0 PT (3) c nghi m x khi v ch khi: x = 4 z 2 0 2 z 2 (6) K t h p (4), (5), (6) ta c z = 0 ho c z = 2. Tr ng h p 1: z = 0 th (2) y = 3, (3) x = 0 x = 4 Th vo (1) ta c x = 4; y = 3 tho mn. Tr ng h p 2: z = 2 th (2) y = 1, (3) x = 2 Th vo (1) ta c x = 2; y = 1 tho mn. x=4 x=2 y = 3 v y = 1 Tm l i z=0 z=2 1 + 1 x2 = x 1 + 2 1 y 2 (1) Bi ton 5. Gi i h phng trnh: 1 1 2 + = (2) 1+y 1 + xy 1+x (I) L i gi i. i u ki n: |x| 1, |y| 1 v xy 0. T (1) suy ra 0 x 1. Do 0 y 1. 1 1 1 1 2 + + (3) Ta ch ng minh Ta c 1+x 1+y 1+y 1+x 1 2 1 + (4) Th t v y : (4) 2 + x + y + 2 xy + c 1+x 1+y 1 + xy xy(x + y) 2 + 2(x + y) + 2xy (1 xy)(x + y) 2 xy(1 xy) 0 (1 xy)( x y)2 0 (B t ng th c ng v i m i x, y thu c [0, 1]) 1 1 2 T (3) v (4) suy ra + 1+y 1 + xy 1+x ng th c x y ra khi v ch khi x = y Thay y = x vo (2) ta c 1 + 1 x2 = x 1 + 2 1 x2 (5) t x = sin t, t 0; , phng trnh (5) tr thnh 2 1 + cos t = sin t(1 + 2 cos t) t t t t 2 cos = 2 sin . cos . 1 + 2 1 2sin2 2 2 2 2 4 t= +k t t 2 t 6 3 3 sin 4sin3 = sin 3 = sin 4 2 2 2 2 4 t= +k 2 3
2

M t s phng php gi i h phng trnh 1 t= 6 x= 2 V i t 0; ta c 2 t= x=1 2 1 1 V y h phng trnh c nghi m ; v (1 ;1) 2 2 Bi ton 6. (BA LAN 1997) Gi i h phng trnh sau trong t p s th c : 3 (x2 + y 2 + z 2 ) = 1(1) x2 y 2 + y 2 z 2 + z 2 x2 = xyz(x + y + z)3 (2) L i gi i. * N u x = 0, (2) x2 z 2 = 0 y = 0 z = 0 1 + V i y = 0, (1): 3z 2 = 1 z = 3 1 + V i z = 0, (1): 3y 2 = 1 y = 3 H phng trnh c cc nghi m 1 0; 0; 3 1 1 , 0; ; 0 , ; 0; 0 3 3

204

* N u x, y, z = 0, (2) x + y + z = 0 x2 y 2 + y 2 z 2 + z 2 x 2 (2) xyz(x + y + z) = 0 (x + y + z)2 V i a, b, c R ta c: (a b)2 + (b c)2 + (c a)2 0 a2 + b2 + c2 ab + bc + ca ng th c x y ra khi v ch khi a = b = c. Nh v y: 1 = 3(x2 +y 2 +z 2 ) (x + y + z)2 = x2 y 2 + y 2 z 2 + z 2 x2 xy 2 z + x2 yz + xyz 2 =1 xyz(x + y + z) xyz(x + y + z) 1 3

do cc BT trn ph i x y ra d u b ng nn x = y = z = K t lu n: H phng trnh c cc nghi m 1 1 1 ; ; 3 3 3 1 1 1 , ; ; 3 3 3

1 1 1 , ; 0; 0 , 0; ; 0 , 0; 0; 3 3 3

M t s phng php gi i h phng trnh

205

Bi t p p d ng
Bi t p 1. (CA NA A 2003) Tm t t c cc nghi m th c dng c a h phng trnh: x3 + y 3 + z 3 = x + y + z x2 + y 2 + z 2 = xyz S: H phng trnh v nghi m Bi t p 2. (OLYMPIC ANH 1998) Tm t t c cc nghi m c a h phng trnh trong t p h p cc s th c dng. S: x = y = z = 2. Bi t p 3. (OLYMPIC ANH 1996) Tm t t c cc nghi m th c dng c a h phng trnh: w + x + y + z = 12 wxyz = wx + wy + wz + xy + xz + yz + 27 S: w = x = y = z = 3 Bi t p 4. (UKRAINA 1997) Tm t t c cc nghi m trong t p h p s th c c a h phng trnh: x1 + x2 + ... + x1997 = 1997 x4 + x4 + ... + x4 = x3 + x3 + ... + x3 1997 2 1 1997 2 1 S: x1 = x2 = ... = x1997 = 1 Bi t p 5. (VMO 2009) Gi i h phng trnh : 1 2 1 + = 1 + 2xy 1 + 2x2 1 + 2y 2 x (1 2x) + y (1 2y) = 2 9 9 + 73 9 73 S: x = y = ;x = y = 18 18 Bi t p 6. Cho a l m t s th c dng cho tr c. Tm t t c cc nghi m th c c a h phng trnh: x2 = 1 x1 + a 2 x1 1 a x2 + x3 = 2 x2 ........................... xn = 1 xn1 + a 2 xn1 1 a x1 = xn + 2 xn xy + yz + zx = 12 xyz = 2 + x + y + z

Gi tr l n nh t, nh nh t c a hm s v ng d ng c a n S: x1 = x2 = ... = xn = a; x1 = x2 = ... = xn = a

206

Ti li u tham kh o 1. Nguy n Vn M u, Phng php gi i phng trnh v b t phng trnh, NXB Gio D c, 1993. 2. Nguy n Vn M u, Nguy n Nguy n Vn Ti n, M t s chuyn gi i tch b i d ng h c sinh gi i trung h c ph thng, NXB Gio D c Vi t Nam, 2009. 3. T p ch ton h c v tu i tr , Tuy n t p 5 nm, NXB Gio D c, 2003. 4. Ti li u t Internet.

2.10

Gi tr l n nh t, nh nh t c a hm s v ng d ng c a n
Nguy n Ti n Tu n Tr ng THPT Chuyn Chu Vn An, L ng Sn

2.10.1

Ph n l thuy t

nh ngha: Cho hm s y = f (x) xc nh trn D. Khi +) M c g i l gi tr l n nh t c a hm s trn D. KH: M = max f (x)


xD

tho mn f (x) M, x D T n t i x0 D sao cho M = f (x0 ). +) m c g i l gi tr nh nh t c a hm s trn D. KH: M = min f (x)


xD

tho mn f (x) M, x D T n t i x0 D sao cho M = f (x0 ). Tnh ch t: a) Tnh ch t 1: Gi s f (x) xc nh trn D v A, B l hai t p con c a D(A B). Gi s t n t i max f (x), max f (x), min f (x), min f (x).
xA xB xA xB

Khi , ta c max f (x) max f (x) v min f (x) min f (x).


xA xB xA xB

CM: Gi s max f (x) = f (x0 ) v i x0 A. Do x0 A x0 B (A B) .


xA

Theo nh ngha ta c f (x0 ) max f (x) hay max f (x) max f (x).
xB xA xB

b) Tnh ch t 2: Hm s f (x) xc nh trn D v t n t i max f (x) v min f (x)


xD xD

Gi tr l n nh t, nh nh t c a hm s v ng d ng c a n Khi , ta c: max f (x) = min (f (x))


xD xD

207

v min f (x) = max (f (x)) .


xD xD

c) Tnh ch t 3: Gi s f (x) v g(x) l hai hm s xc nh trn D v f (x) g(x) v i m i x thu c D. Khi max f (x) max g(x)
xD xD

d) Tnh ch t 4: Gi s f (x) xc nh trn D v D = D1 D2 . Gi thi t t n t i max f (x) v min f (x) v i i = 1, n. Khi


xDi xDi

max f (x) = max max f (x), max f (x)


xD xD1 xD2

v min f (x) = min


xD xD1

min f (x), min f (x) .


xD2

e) Tnh ch t 5: Cho cc hm s f1 (x), f2 (x), . . . , fn (x) cng xc nh trn D. t f (x) = f1 (x) + f2 (x) + + fn (x). Gi thi t t n t i max f (x), min f (x), max fi (x), min fi (x)
xD xD xD xD

v i i = 1, n. Khi , ta c max f (x) max f1 (x) + max f2 (x) + + max fn (x).


xD xD xD xD

D u ng th c x y khi v ch khi t n t i x0 thu c D sao cho max fi (x) =


xD

fi (x0 ) v i i = 1, n. min f (x) min f1 (x) + min f2 (x) + + min fn (x)


xD xD xD xD

D u ng th c x y khi v ch khi t n t i x0 thu c D sao cho min fi (x) =


xD

fi (x0 ) v i i = 1, n. f) Tnh ch t 6: Cho cc hm s f1 (x), f2 (x), . . . , fn (x) cng xc nh trn D v fi (x) > 0. t f (x) = f1 (x).f2 (x) . . . fn (x). Gi thi t t n t i max f (x), min f (x), max fi (x), min fi (x)
xD xD xD xD

v i i = 1, n. Khi , ta c max f (x) (max f1 (x))(max f2 (x)) . . . (max fn (x))


xD xD xD xD

min f (x) min f1 (x) min f2 (x) . . . min fn (x)


xD xD xD xD

Gi tr l n nh t, nh nh t c a hm s v ng d ng c a n

208

2.10.2

Ph n bi t p

Bi t p 39. Chuyn 1: Phng php b t ng th c I/ L THUY T: B t ng th c AM-GM: Cho a1 , a2 , a3 , . . . , an l cc s khng m. Khi a1 + a2 + + an n a1 .a2 . . . an n D u ng th c x y ra khi v ch khi a1 = a2 = = an . B t ng th c Cauchy-Bunhiacovski: Cho a1 , a2 , a3 , . . . , an v b1 , b2 , b3 , . . . , bn l 2n s b t k. Khi , a2 + a2 + + a2 1 2 n b2 + b2 + + b2 (a1 b1 + a2 b2 + + an bn )2 1 2 n a1 a2 an = = = . b1 b2 bn (1)

D u ng th c x y ra khi v ch khi II/ Bi t p. B t ng th c AM-GM Bi 1: Cho hm s y= 4 1 x2 +

1x+

1 + x.

Tm gi tr nh nh t c a hm s . Hm s c T p xc nh: D = [1; 1] . V i m i x D, p d ng b t ng th c AM-GM ta c 1x+ 1+x 4 4 4 1 x2 = 1 x. 1 + x (1) 2 1x+1 4 4 1 x = 1 x.1 (2) 2 1+x+1 4 4 1 + x = 1 + x.1 (3) 2 C ng v v i v 3 ng th c trn ta c f (x) 1 + 1 + x + 1 x (4) v i m i x D. D u ng th c x y ra khi v ch khi d u ng th c (1), (2) v (3) cng x y ra. M d u ng th c (1), (2) v (3) x y ra khi v ch khi x = 0. p d ng b t ng th c AM-GM, v i m i x D, ta c: 1x+1 1 x = 1 x.1 2 1+x+1 1 + x = 1 + x.1 2 (5) (6)

Gi tr l n nh t, nh nh t c a hm s v ng d ng c a n Suy ra f (x) 1 + 2x 2+x + =3 x1+ x+11+ 2 2

209

(7)

D u ng th c x y ra khi v ch khi d u ng th c trong (5) v (6) x y ra. D u ng th c trong (5) v (6) x y ra khi v ch khi x = 0. T (4) v (7) suy ra f (x) 3 v i m i x D m f (0) = 3 v 0 D nn max f (x) = 3.
xD

Bi 2: Tm gi tr nh nh t c a hm s x f (x) = + 1 x 2x2 . 2 Hm s c T p xc nh: D = 1; th c AM-GM, ta c: 1 x 2x2 = 1(1 x 2x2 ) 1 + 1 x 2x2 2 x 2x2 = 2 2 1 V i m i x D, p d ng b t ng 2

x x 2 x 2x2 2 Khi f (x) = + 1 x 2x + = 1 x2 1 v i m i 2 2 2 xD D u ng th c x y ra khi v ch khi 2 1 = 1 x 2x 1 x2 = 1 x=0 1 x 1 2 Suy ra max f (x) = 1.


xD

Bi 3: Tm gi tr nh nh t c a hm s f (x, y) =

x y + trn mi n 1y 1x

D = {(x, y) : x > 0, y > 0, x + y = 1} . H ng d n. (66) L y (x, y) D. Khi f (x, y) = x y x y + = + . y x 1y 1x

p d ng b t ng th c AM-GM x + y 2x y f (x, y) + x + y 2( x + y) y + x2 y x Hay f (x, y) x+ y (4)

Gi tr l n nh t, nh nh t c a hm s v ng d ng c a n M t khc 1y 1x 1 1 f (x, y) = + = + ( x + y) y y x x 1 1 T (4) v (5) suy ra 2f (x, y) + hay y x f (x, y) 1 2 1 1 + y x

210

(5)

(6)

1 2 1 Ti p t c p d ng b t ng th c AM-GM cho hai s + 4 xy y x 2 = 2 2 v x + y = 1 (7). x+y 2 1 1 1 1 + T (7) suy ra f (x, y) 2 2= 2 2 y 2 x 1 1 1 1 1 D u = x y ra khi v ch khi = = hay x = y = M y x y 2 x 1 1 1 1 , D v f ; = 2. V y min f (x, y) = 2. (x,y)D 2 2 2 2 Bi 4: Tm cc gi tr nh nh t c a hm s f (x, y), g(x, y), h(x, y) trn mi n D = {(x, y) : x > y > 0}, trong f (x, y) = x + g(x, y) = x + 1 y(x y)

4 (x y)(y + 1)2 1 h(x, y) = x + y(x y)2

H ng d n. L y (x, y) D tu . Khi , theo b t ng th c AM-GM ta c: f (x, y) = x + 1 1 1 = y + (x y) + 3 3 y(x y) =3 y(x y) y(x y) y(x y) x=2 y=1

1 y(x y) C (2, 1) D v f (2, 1) = 3. V y min f (x, y) = 3. D u ng th c x y ra khi v ch khi y = xy =


(x,y)D

L y (x, y) D tu . Khi , theo b t ng th c AM-GM ta c: g(x, y) = x + 4 y+1 y+1 4 + + 1 2 = (x y) + 2 2 (x y)(y + 1) (x y)(y + 1)2 y+1 y+1 4 . . 1=41=3 2 2 (x y)(y + 1)2

4 4 (x y).

Gi tr l n nh t, nh nh t c a hm s v ng d ng c a n D u ng th c x y ra khi v ch khi xy = y+1 4 = 2 (x y)(y + 1)2


(x,y)D

211

x=2 y=1

C (2, 1) D v g(2, 1) = 3. V y min g(x, y) = 3. L y (x, y) D tu . Khi , theo b t ng th c AM-GM ta c: xy xy 1 xy 4 h(x, y) = y+ + + y. 2 4 2 2 2 y(x y) D u ng th c x y ra khi v ch khi xy 1 y= = 2 y(x y)2 3 3, 3 3 2 3 3 3 3, 2 3 3 1 y= 33 2 x=
2

1 4 4 = = =2 2 4 4 2 y(x y)2

D v h

= 2 2. V y min h(x, y) = 2 2.
(x,y)D

Bi 5: Tm gi tr nh nh t c a hm s f (x, y, z) = (xyz + 1) 1 1 1 + + x y z + x y z + + xyz y z x

trn mi n D = {(x, y, z) : x > 0, y > 0, z > 0} H ng d n. z x 1 1 1 y Ta c f (x, y, z) = yz + + xz + + xy + + + + x y z L y z x y x y z (x, y, z) tu thu c D. Khi , p d ng b t ng th c AM-GM, ta c: yz + Khi f (x, y, z) 1 1 1 + + +x+y+z = x y z 1 +x + x 1 +y + y 1 +z z 6 y z x 2y, xz + 2z, xy + 2x z x y

D u ng th c x y ra khi v ch khi x = y = z = 1. V (1, 1, 1) thu c D v f (1, 1, 1) = 6 nn min f (x, y, z) = 6


xD

1 1 1 2+ 2+ x y z v gi tr nh nh t c a hm s g(x, y, z) = x + 16xyz, h(x, y, z) = x + xy + 3 xyz trn mi n D = {(x, y, z) : x > 0, y > 0, z > 0, x + y + z = 1} . Bi 6: Tm gi tr nh nh t c a hm s f (x, y, z) = 2+ H ng d n. (57)

Gi tr l n nh t, nh nh t c a hm s v ng d ng c a n L y (x, y, z) D tu . p d ng b t ng th c AM-GM ta c: 2+ 2+ 2+ yz 1 x+y+z y z =1+1+ =1+1+1+ + 55 2 x x x x x xz 1 x+y+z x z =1+1+ =1+1+1+ + 55 2 y y y y y x+y+z x y xy 1 =1+1+ =1+1+1+ + 55 2 z z z z z

212

Nhn v v i v c a ba b t ng th c ta c, f (x, y, z) 125 v i m i (x, y, z) thu c D. y z x z x y D u = x y ra khi v ch khi = = = = = = 1 x = y = x x y y z z 1 z= 3 1 1 1 1 1 1 M ; ; D v f ; ; = 125. V y min f (x, y, z) = 125. (x,y,z)D 3 3 3 3 3 3 L y (x, y, z) D v i x + y + z = 1 khi g(x, y, z) = 1 (y + z) + 16yz(1 y z) = 1 + 16yz (y + z)(1 + 16yz) p d ng b t ng th c AM-GM ta c y +z 2 yz v 1+16yz 8 yz (y + z)(1 + 16yz) 16yz Khi g(x, y, z) 1 + 16yz 16yz = 1 D u = x y ra khi v ch khi y=z x= 1 16yz = 1 2 x+y+z =1 y=z= 1 4 x > 0, y > 0, z > 0 M 1 1 1 , , 2 4 4 D v g 1 1 1 , , 2 4 4 = 1. V y max = g
(x,y,z)D

1 1 1 , , 2 4 4

= 1.

B t ng th c Cauchy-Bunhiacovski x y z + + x+1 y+1 z+1 trn mi n D = {(x, y, z) : x > 0, y > 0, z > 0, x + y + z = 1} . H ng d n. L y (x, y, z) thu c D. Khi , ta c Bi 7: Tm gi tr l n nh t c a hm s f (x, y, z) = f (x, y, z) = 1 1 1 + + x+1 y+1 z+1 (1) (1) p d ng b t ng th c Cauchy-Bunhiacovski cho hai c p s 1 = 3 1 , x+1 1 , y+1 1 z+1 1 1 1 + 1 + 1 x+1 y+1 z+1

Gi tr l n nh t, nh nh t c a hm s v ng d ng c a n v x + 1, y + 1, z + 1 c 1 1 1 + + x+1 y+1 z+1

213

((x + 1) + (y + 1) + (z + 1)) (1 + 1 + 1)2 = 9 (2)

M x + y + z = 1 t (2) suy ra 1 1 1 + + x+1 y+1 z+1 K t h p (1) v (3) suy ra f (x, y, z) 3 f 1 1 1 , , 3 3 3 4 = . 3 9 4 1 1 1 , , 3 3 3 (3)

9 4 = c 4 3

D v

4 V y max f (x, y, z) = . xD 3 Bi 8: Tm gi tr l n nh t c a hm s f (x, y, z) = y(z x) trn mi n D = {(x, y, z) : x2 + z 2 = 1, y 2 + 2y(x + y) = 6} . H ng d n. Ta c f (x, y, z) = y(zx) = z(2x + y) + (x)(2z + y) p d ng b t ng th c Cauchy-Bunhiacovski cho hai c p s (z, (x)) v (2x + y, 2z + y) ta c z 2 + (x)2 (2x + y)2 + (2z + y)2

[z(2x + y) + (x)(2z + y)]2 = [y(z x)]2 M x2 + z 2 = 1 v (2x + y)2 + (2z + y)2 = 2y 2 + 4y(x + z) + 4(x2 + y 2 ) = 16 v y 2 + 2y(x + z) = 6 Khi [y(z x)]2 1.16 = 16 y(z x) 4 hay f (x, y, z) 4. D u ng th c x y ra khi v ch khi x = 3 x2 + z 2 = 1 2 10 y + 2y(x + z) = 6 y = 10 2x + y z = z= 1 x 2z + y 10 C 3 1 , 10, 10 10 D, f 3 1 , 10, 10 10 = 4. V y max f (x, y, z) = 4.
xD

Bi 9 Tm gi tr nh nh t c a hm s f (x, y) = trn mi n D = {(x, y) : 0 < x < 1, 0 < y < 1} . H ng d n. V i m i (x, y) thu c D ta c f (x, y) =

x2 y2 1 + + +x+y 1x 1y x+y

x2 y2 1 1 1 1 +1+x+ +1+y+ 2 = + + 2 (1) 1x 1y x+y 1x 1y x+y

Gi tr l n nh t, nh nh t c a hm s v ng d ng c a n p d ng b t ng th c Cauchy-Bunhiacovski cho hai c p s 1 , 1x v ( 1 x, 1 y, x + y). Ta c 1 , 1y 1 x+y

214

1 1 1 + + ((1 x) + (1 y) + (x + y)) (1 + 1 + 1)2 = 9 1x 1y 1z 1 1 9 1 + + (2) 1x 1y x+y 2 T (1) v (2) f (x, y) 9 5 2 = . D u ng th c x y ra khi v ch khi 2 2

1 1 = 1x x+y x=y = 1 1 1 3 = 1y x+y C 1 1 , 3 3 D, f 1 1 , 3 3 5 5 = . V y min f (x, y) = . xD 2 2

Bi 8: Tm gi tr l n nh t v gi tr nh nh t c a hm s f (x, y, z) = x+y +z 4 trn mi n D = (x, y, z) : x(x 1) + y(y 1) + z(z 1) 3 H ng d n. L y (x, y, z) thu c D. Khi x(x 1) + y(y 1) + z(z 1) 4 3(x2 + y 2 + z 2 ) 3(x + y + z) + 4 3

p d ng b t ng th Cauchy-Bunhiacovski cho hai c p s (1, 1, 1) v (x, y, z) ta c: 3(x2 + y 2 + z 2 ) (x + y + z)2

Bi 10: Tm gi tr nh nh t v gi tr l n nh t c a hm s f (x, z) = y, 3 3 3 4 4 y + 4 z trn x + y + z v gi tr l n nh t c a hm s g(x, y, z) = x + mi n D = {(x, y, z) : x 0, y 0, z 0, x + y + z = 1} H ng d n. Xt hm s f (x, y, z) = x3 +y 3 +z 3 trn D = {(x, y, z) : x 0, y 0, z 0, x + y + z = 1} . L y (x, y, z) thu c D. p d ng b t ng th c Cauchy-Bunhiacovski cho hai c p s x x, y y, z z v x, y, z Ta c f (x, y, z) (x2 + y 2 + z 2 )2 (1)

Gi tr l n nh t, nh nh t c a hm s v ng d ng c a n do x + y + z = 1.

215

p d ng b t ng th c Cauchy-Bunhiacovski cho c p s (1, 1, 1) v (x, y, z) Ta c 1 (2) 3(x2 + y 2 + z 2 ) (x + y + z)2 x2 + y 2 + z 2 3 1 T (1) v (2) suy ra f (x, y, z) . 9 1 1 1 1 C , , D u ng th c x y ra khi v ch khi x = y = z = 3 3 3 3 1 1 1 1 1 D, f , , = . V y min f (x, y, z) = . xD 3 3 3 9 9 L y (x, y, z) thu D. p d ng b t ng th c Cauchy-Bunhiacovski cho c 4 4 hai c p s x, 4 y, z v (1, 1, 1). Ta c 3( x + y + z) 4 x + 4 y + 4 z
2

(3)

L i p d ng b t ng th c Cauchy-Bunhiacovski cho hai c p s x, y, z v (1, 1, 1) Ta c 3(x + y + z) ( x + y + z)2 x + y + z 3 (4) T (3) v (4) suy ra g 2 (x, y, z) 3 3 = 27 hay g(x, y, z) 4 27. D u ng th c x y ra d u ng th c (3) v (4) cng x y ra hay x = 1 y=z= . 3 1 1 1 1 1 1 3 C , , D, g , , = = 4 27. V y max f (x, y, z) = 4 27 4 xD 3 3 3 3 3 3 3 Chuyn 2: Phng php mi n gi tr hm s Phng php: Xt bi ton tm gi tr l n nh t, gi tr nh nh t c a hm s f (x) trn m t mi n D cho tr c. B 1: G i y0 l m t gi tr tu c a hm s f (x) trn D. f (x) = y0 B 2: Gi i i u ki n h phng trnh ( n x): xD B 3: Bi n i a h phng trnh v d ng: y0 . B 4: V y0 l gi tr b t k trn D. a ra k t lu n. Bi t p. Bi 1: Tm gi tr l n nh t v gi tr nh nh t c a hm s f (x) = 2x2 + 7x + 23 x2 + 2x + 10

trn ton tr c s . T p xc nh: R. G i y0 l m t gi tr c a hm s . Khi , phng trnh 2x2 + 7x + 23 = y0 (1) c nghi m. x2 + 2x + 10

Gi tr l n nh t, nh nh t c a hm s v ng d ng c a n V x2 + 2x + 10 > 0 nn

216

(1) 2x2 +7x+23 = y0 (x2 +2x+10) (y0 2)x2 +(2y0 7)x+10y0 23 = 0 (2) c nghi m Tr ng h p 1: y0 = 2 phng trnh (2) tr thnh 3x 3 = 0 x = 1 (1) c nghi m. Tr ng h p 2: y0 = 0, khi phng trnh (2) c nghi m khi v ch khi 0 = (2y0 7)2 4(y0 2)(10y0 23) 0 3 5 2 9y0 16y0 + 15 0 y0 , y0 = 2 2 2 V y0 l m t gi tr tu c a hm s y = f (x), nn 5 max f (x) = , xR 2 min f (x) =
xR

3 2

Bi 2: Tm gi tr l n nh t v gi tr nh nh t c a hm s x2 + 4 2x + 3 y= x2 + 1 /S: 1 y0 5. Bi 3: (98 191) Tm gi tr l n nh t v gi tr nh nh t c a hm s y= 1 4 153 1+ 2x2 + 10x + 3 3x2 + 2x + 1

/ S:

y0

153 4

Bi 4: Tm gi tr l n nh t v gi tr nh nh t c a hm s f (x, y) = x2 + y 2 2 trn m t mi n D = (x, y) : (x2 y 2 + 1) + 4x2 y 2 x2 y 2 = 0 . H ng d n. G i t0 l m t gi tr tu c a hm s f (x, y) trn D. Khi , h phng trnh x2 + y 2 = t0 (1) 2 2 2 2 2 2 2 (x y + 1) + 4x y x y = 0 (2) c nghi m T (2) (x2 + y 2 )2 3(x2 + y 2 ) + 1 + 4x2 = 0 th (1) vo c phng trnh t2 3t0 + 1 + 4x2 = 0 (3) 0 3 5 h c nghi m th (3) c nghi m t2 3t0 + 1 0 t0 0 2 3+ 5 2

Gi tr l n nh t, nh nh t c a hm s v ng d ng c a n Khi , (3) c nghi m x2 = mn c nghi m.

217

t2 + t0 + 1 t2 + 3t0 1 0 th vo (2) y 2 = 0 tho 4 4

3+ 5 M t0 l m t gi tr tu c a f (x, y) trn D nn max f (x, y) = (x,y)D 2 3 5 . v min f (x, y) = (x,y)D 2 Bi 5: Tm gi tr l n nh t v gi tr nh nh t c a hm s f (x, y) = x + 2y + 1 x2 + y 2 + 7

trn mi n D = {(x, y) : x + y = 1} . H ng d n. G i t0 l m t gi tr tu c a hm s f (x, y) trn D. Khi , h phng trnh x + 2y + 1 = t0 (1) x2 + y 2 + 7 x+y =1 (2) c nghi m T (2) c x = 1 y th vo (1) y+2= v x2 + y 2 + 7 > 0. 2t0 y 2 (2t0 + 1)y + 8t0 2 = 0 (3) 1 y 2 + y 2 + 7 t0

h c nghi m th (3) c nghi m Tr ng h p 1: t0 = 0 khi , (3) tr thnh y 2 = 0 y = 2, x = 3. Phng trnh c nghi m. Tr ng h p 2: t0 = 0 c phng trnh b c hai, phng trnh c nghi m = (2t0 + 1)2 8t0 (8t0 2) 0 60t2 20t0 1 0 5 2 10 5 + 2 10 0 t0 , t0 = 0 30 20 5 2 10 5 + 2 10 K t h p (3) c nghi m khi t0 30 20

5 + 2 10 V t0 l m t gi tr b t k c a f (x, y) trn D nn max f (x, y) = (x,y)D 30 5 10 v min f (x, y) = . (x,y)D 30 Bi 6: Tm gi tr l n nh t v gi tr nh nh t c a hm s f (x, y) = trn D = {(x, y) : x2 + y 2 > 0} . x2 (x 4y)2 x2 + 4y 2

Gi tr l n nh t, nh nh t c a hm s v ng d ng c a n

218

H ng d n. Xt D1 = {(x, y) : x = 0, y = 0} v D2 = {(x, y) : y = 0}. Khi D = D1 D2 . N u (x, y) D1 th f (x, y) = 0. V y max f (x, y) = min f (x, y) = 0.
xD1 xD1

(x/2y) (x/2y 2)2 N u (x, y) D2 . Khi , ta c f (x, y) = (x/2y)2 + 1 t2 (t 2)2 4t 4 x = 2 t t = , c hm s F (t) = 2y t2 + 1 t +1 Khi max f (x, y) = max F (t),
(x,y)D2 tR

min f (x, y) = min F (t)


(x,y)D2 tR

G i l m t gi tr b t k c a hm s F (t). 4t 4 Khi , phng trnh 2 = t2 4t + + 4 = 0 (1) c nghi m t +1 Tr ng h p 1: V i = 0 phng trnh (1) c nghi m t = 1. Tr ng h p 2: V i = 0 phng trnh (1) c nghi m = 4 ( + 4) 0 2 2 2 2 + 2 2, = 0 K t h p (1) c nghi m khi 22 2 2+2 2 Suy ra max f (x, y) = (x,y)D2 max F (t) = 2 + 2 2 v min f (x, y) = min F (t) = 2 2 2
tR (x,y)D2 tR

Vy max f (x, y) = max max f (x, y), max f (x, y)


xD xD1 xD2

= max 0, 2 + 2 2 = 2+2 2 = min 0, 2 2 2 = 22 2

min f (x, y) = min


xD

xD1

min f (x, y), min f (x, y)


xD2

Bi 7: Tm gi tr l n nh t v gi tr nh nh t c a hm s f (x) = 3 + 4x2 + 3x4 (1 + x2 )2

H ng d n. G i y0 l m t gi tr tu c a f (x). 3 + 4x2 + 3x4 Khi , phng trnh = y0 c nghi m (1 + x2 )2 (y0 3)x4 + 2(y0 2)x2 + y0 3 = 0 (1) c nghi m xt hai tr ng h p Tr ng h p 1: y0 = 3 khi (1) tr thnh x2 = 0. V y (1) c nghi m. (1)

Gi tr l n nh t, nh nh t c a hm s v ng d ng c a n Tr ng h p 2: (1) c nghi m khi v ch khi h t0 (y0 3)t2 + 2(y0 2) + y0 3 = 0 (2)

219

(1) c nghi m (2) c nghi m t 0 m (2) c P = 1 > 0 (2) c 2 nghi m cng d u. Khi , (2) c nghi m 5 0 t0 3 S0 2 5 t0 3. 2

K t h p hai tr ng h p (1) c nghi m khi

x2 + px + q Bi 8: Cho hm s f (x) = . Tm p, q max f (x) = 9, min f (x) = xR xR x2 + 1 1 H ng d n. G i y0 l m t gi tr tu c a hm s f (x). x2 + px + q = y0 (1) c nghi m. Khi , phng trnh x2 + 1 Ta c (1) (y0 1)x2 px + (y0 q) = 0 (2). (1) c nghi m th (2) c nghi m, xt 2 tr ng h p Tr ng h p 1: y0 = 1 th (2) c nghi m khi p = 0 ho c p = 0 v q = 1. Tr ng h p 2: y0 = 1 th (2) c nghi m khi = p2 4(y0 1)(y0 q) 0
2 4y0 4(q + 1)y0 (p2 4q) 0 y1 y0 y2

Ta c y1 1 y2 v v i y0 = 1 c = p2 0 q K t h p hai tr ng h p ta c (1) c nghi m l y1 y0 y2 . Khi , tac min f (x) = y1 , max f (x) = y2


xR xR

Theo vit ta c y1 + y2 = q + 1 = 8 p2 4q y1 .y2 = = 9 4

q=7 p = 8

12x(x + a) tm a nguyn khc 0 sao cho i l ng x2 + 36 max f (x) cng l s nguyn. Bi 9: Cho hm s y =
xR

H ng d n. G i y0 l m t gi tr tu c a hm s f (x). 12x(x + a) Khi , phng trnh = y0 (1) c nghi m x2 + 36 T (1) ta c (y0 12)x2 + 12ax + 36y0 = 0 (2) (1) c nghi m th (2) c nghi m. (2) c nghi m xt hai tr ng h p

Gi tr l n nh t, nh nh t c a hm s v ng d ng c a n Tr ng h p 1: y0 = 12 c (2) tr thnh x = 0. V y (2) c nghi m Tr ng h p 2: y0 = 12 th (2) c nghi m

220

2 = 36a2 36y0 (y0 12) 0 y0 12y0 a2 0 6 36 + a2 y0 6 + 36 + a2 Nh n th y y0 = 0 th = a2 < 0 a do v y 6 36 + a2 0 6 + 36 + a2 K t h p hai tr ng h p (1) c nghi m th 6 36 + a2 y0 6 + 36 + a2 V y max f (x) = 6 + 36 + a2 . xR Tm a nguyn khc 0 36 + a2 = k (3) nguyn dng. N u a > 0 tho mn (3) th a cng tho mn 3, xt a > 0 khi , (3) 36 = k 2 a2 = (k a)(k + a) V k + a > 0 suy ra k a > 0 v k + a v k a l s nguyn. Suy ra ka=2 a = 8. k + a = 18

Chuyn 3: Phng php s d ng o hm c a hm s . Phng php: Cho hm s y = f (x) xc nh trn [a; b] B 1: Gi i phng trnh f (x) = 0 tm cc nghi m x1 , x2 , x3 , . . . , xn trong [a; b]. B 2: Tnh cc s f (a), f (b), f (x1 ), f (x2 ), . . . , f (xn ). B 3: K t lu n gi tr l n nh t l s l n nh t, gi tr nh nh t l s nh nh t trong cc gi tr trn. BI T P: Bi 1: Tm gi tr nh nh t v gi tr nh nh t c a hm s y = sin20 x + cos20 x + cos2 x + = cos20 x + sin20 x hm s 2 2 y = sin20 x + cos20 x tu n hon v i chu k T = . Do v y, ta ch c n tm gi tr 2 l n nh t v gi tr nh nh t c a hm s trn 1 chu k 0; . 2 19 19 Ta c y = 20sin x. cos x 20cos x. sin x. Xt y = 0 x=0 sin x = 0 cos x = 0 x = 2 cos x = sin x x= 4 H ng d n. Do sin2 x + B ng bi n thin:

Gi tr l n nh t, nh nh t c a hm s v ng d ng c a n K t lu n: max y = 1 khi x = 0; x =
xR

221

1 , min y = khi x = 2 xR 512 4

Chuyn 4: Phng php chi u bi n thin c a hm s Phng php: D a vo tnh ng bi n v ngh ch bi n c a hm s b c nh t v b c hai. L p b ng bi n thin c a hm f (x) c n tm gi tr l n nh t, gi tr nh nh t trn D cho tr c. Tnh ch t 1 +) Hm s y = ax + b ng bi n khi a > 0 v ngh ch bi n khi a < 0. +) Hm s y = ax2 + bx + c : b b N u a > 0 : ng bi n khi x > , ngh ch bi n khi x < . N u a < 0 : 2a 2a b b ng bi n khi x < , ngh ch bi n khi x > . 2a 2a Tnh ch t 2: +) N u f (x) l hm ng bi n trn D th f (x) l hm ngh ch bi n trn D. 1 ngh ch +) N u f (x) l hm ng bi n trn D v f (x) > 0 th hm f (x) bi n trn D. Tnh ch t 3: +) N u f (x) v g(x) l hai hm ng bi n trn D th hm f (x) + g(x) cng ng bi n trn D. +) N u f (x), g(x) l hai hm ng bi n trn D v f (x) > 0, g(x) > 0 trn D th hm f (x).g(x) cng ng bi n trn D. BI T P Bi 1: Tm gi tr l n nh t v gi tr nh nh t c a hm s y = |2x 4| + |3x + 6| |x 3| + |x + 1| xt trn mi n D = {x : 3 x 4}. H ng d n. L p b ng bi n thin c a hm s trn D Bi 2: Tm gi tr l n nh t v gi tr nh nh t c a hm s y = |x2 3x + 2| |2x + 4| + |x2 7x + 12| trn mi n D = {x : 3 x 5} Bi 3: Tm gi tr l n nh t v gi tr nh nh t c a hm s f (x, y, z) = x + y + z + xy + yz + zx trn mi n D = {(x, y, z) : x2 + y 2 + z 2 = 1} Bi 4: Tm gi tr l n nh t v gi tr nh nh t c a hm s f (x) = x4 4x3 x2 + 10x 3 trn mi n D = {x : 1 x 4} Bi 5: Tm gi tr l n nh t v gi tr nh nh t c a hm s f (x, y) = (x + 1 1 + trn mi n D = {(x, y) : 1 x 3, 1 y 2} y) x y

Gi tr l n nh t, nh nh t c a hm s v ng d ng c a n Bi 6: Tm gi tr l n 2nh t v gi tr nh nh t c a hm s y = 6 x 18 + 3x x trn mi n D = {x : 3 x 6}

222 3+x+

Bi 7: Cho hm s f (x) = 4x2 4ax+a2 2a xt trn mi n D = {x : 2 x 0}. Tm a hm s c min f (x) = 2.


xD

Bi 8: Cho phng trnh 2x2 + 2(m + 2)x + m3 + 4m + 3 = 0. Khi phng trnh c nghi m x1 , x2 xt i l ng A = x1 + x2 + 3x1 x2 . Tm gi tr l n nh t v gi tr nh nh t c a A. Bi 9: Tm gi tr l n nh t v gi tr nh nh t c a hm s f (x) = x2 +mx+1 trn mi n 1 x 1. Bi n lu n k t qu theo m. Bi 10: Cho hm s f (x) = x2 + |x a|. Tm a min f (x) > 1.
xR

Chuyn 5: ng d ng c a gi tr l n nh t v gi tr nh nh t trong vi c gi i v bi n lu n phng trnh v b t phng trnh. Phng php: Bi n i phng trnh v d ng f (x) = d(m). Tm gi tr l n nh t v gi tr nh nh t c a hm s trn I v so snh cc gi tr l n nh t v gi tr nh nh t v i a. T suy ra nghi m c a phng trnh.

2.10.3

Bi t p

Bi 1: a. Tm m phng trnh x + 2x2 + 1 = m c nghi m. b. Tm m b t phng trnh x + 2x2 + 1 > m x R 2x H ng d n. Xt hm s y = x + 2x2 + 1 y = 1 + Xt 2x2 + 1 1 x<0 y = 0 2x2 + 1 = 2x B ng bi n 2 2 x = 2x + 1 = 4x 2 thin: 1 Phng trnh f (x) = m c nghi m khi v ch khi m . 2 1 B t phng trnh f (x) > m c nghi m v i m i m khi min f (x) = > m xR 2 Bi 2: Tm m phng trnh nghi m. 2x+ 2+x (2 x)(2 + x) = m c

Bi 3: Tm m b t phng trnh mx4 4x + m 0 x R. H ng d n. B t phng trnh m(x4 + 1) 4x f (x) = C f (x) = B ng bi n thin: 4(1 3x4 ) 1 2 = 0 x = 4 3 (1 + x4 ) x4 4x m. +1

Gi tr l n nh t, nh nh t c a hm s v ng d ng c a n V y b t phng trnh nghi m ng m i x th max f (x) = f


xR

223

1 4 3

4 27 m

You might also like